2014-2018高考英语试题分项版解析专题15社会生活说理议论类含解析

申明敬告: 本站不保证该用户上传的文档完整性,不预览、不比对内容而直接下载产生的反悔问题本站不予受理。

文档介绍

2014-2018高考英语试题分项版解析专题15社会生活说理议论类含解析

‎ 专题15 社会生活、说理议论类 ‎2019年高考题 C【2019·全国I】‎ Languages have been coming and going for thousands of years, but in recent times there has been less coming and a lot more going. When the the world was still populated by hunter-gatherers,small, tightly knit (联系)groups developed their own patterns of speech independent of each other.Some language experts believe that 10,000 years ago, when the world had just five to ten million people, they spoke perhaps 12,000 languages between them.‎ Soon afterwards, many of those people started settling down to become farmers, and their languages too became more settled and fewer in number. In recent centres, trade,industrialisation. the development of the nation-state and the spread of universal compulsory education. Especially glbalisation and better communications in the past few decades,all have caused many Languages to disappear,and dominant languages such as English.Spanish and Chinese are increasingly taking over.‎ At present, the world has about 6 800 languages. The distribution of these languages is hugely uneven. The general rule is that mild zones have relatively few languages. Often spoken by many people while hot. wet zones have lots, often spoken by small numbers.Europe has only around 200 Languages: the Americas about 1,000. Africa 2 400; and Asia and the Pacific perhaps 3,200, of which Papua New Guinea alone accounts for well over 800. The median number (中位数)of speakers is a mere 6.000, which means that half the worlds languages are spoken by fewer people than that.‎ Already well over 400 of the total of, 6,800 languages are close to extinction(消亡), with only a few elderly speakers left. Pick, at random, Busuu in Cameroon (eight remaining speakers),Chiapaneco in Mexico(150). Lipan Apache in the United States(two or three)or Wadjigu in Australia (one, with a question-mark): none of these seems to have much chance of survival.‎ ‎28. What can we infer about languages in huntergatherer times?‎ A. They developed very fast. B. They were large in number.‎ C. They had similar patters. D. They were closely connected ‎29. Which of the following best explains"dominant " underlined in paragraph 2?‎ A. Complex. B. Advanced.‎ C. Powerful. D. Modem.‎ ‎30. How many languages are spoken by less than 6, 000 people at present?‎ A. About 6 800 B. About 3 400‎ C. About 2.400 D. About 1-200‎ ‎31. What is the min idea of the text?‎ A. New languages will be created.‎ B. Peoples lifestyles are reflected in languages C. Human development results in fewer languages D. Geography determines language evolution.‎ ‎【答案】28. B 29. C 30. B 31. C ‎【解析】本文是一篇议论文。文章讲述了随着社会的发展人类语言越来越少及其原因。‎ ‎29. 猜测词义题。根据文章第二段中的dominant languages such as English, Spanish and Chinese are increasingly taking over.可知,英语、西班牙语和汉语正在替代其他语言。由此推知dominant languages意为:强有力的语言。故选C。‎ ‎30. 细节理解题。根据文章第三段中的At present, the world has about 6,800 languages.和The median number (中位数) of speakers is a mere 6,000, which means that half the world’s languages are spoken by fewer people than that. 可知,目前世界上大约有6800种语言,但是讲的人数少于6000人的占一半即3400.故选B。‎ ‎31. 主旨要义题。根据文章第一段中的主题句Languages have been coming and going for thousands of years, but in recent times there has been less coming and a lot more going.可知,语言的产生和消失进行了几千年,但最近时代语言产生的少,消失的太多。故选C。‎ 点睛:长难句解读 In recent centuries, trade, industrialisation, the development of the nation-state and the spread of universal compulsory education, especially globalisation and better communications in the past few decades, all have caused many languages to disappear, and dominant languages such as English, Spanish and Chinese are increasingly taking over.‎ 分析:and连接两个并列句,前面的介词短语in. . .作时间状语,all和前面的名词短语是同位语。‎ 句意:在最近的几个世纪,贸易、工业化、民族国家的发展和全球义务教育的蔓延,尤其是过去几十年来,全球化和更好的交流,这些都是导致许多语言消失的原因。而且像英语、西班牙语和汉语这些强大的语言正在不断地占据统治地位。 ‎ B【2019·全国II】‎ Many of us love July because it’s the month when nature’s berries and stone fruits are in abundance. These colourful and sweet jewels form British Columbia’s fields are little powerhouses of nutritional protection.‎ Of the common berries, strawberries are highest in vitamin C, although, because of their seeds, raspberries contain a little more protein (蛋白质), iron and zinc (not that fruits have much protein). Blueberries are particularly high in antioxidants (抗氧化物质). The yellow and orange stone fruits such as peaches are high in the carotenoids we turn into vitamin A and which are antioxidants. As for cherries (樱桃), they are so delicious who cares? However, they are rich in vitamin C.‎ When combined with berries of slices of other fruits, frozen bananas make an excellent base for thick, cooling fruit shakes and low fat “ice cream”. For this purpose, select ripe bananas for freezing as they are much sweeter. Remove the skin and place them in plastic bags or containers and freeze. If you like, a squeeze of fresh lemon juice on the bananas will prevent them turning brown. Frozen bananas will last several weeks, depending on their ripeness and the temperature of the freezer.‎ If you have a juicer, you can simply feed in frozen bananas and some berries or sliced fruit. Out comes a “soft-serve” creamy dessert, to be eaten right away. This makes a fun activity for a children’s party; they love feeding the fruit and frozen bananas into the top of the machine and watching the ice cream come out below.‎ ‎24. What does the author seem to like about cherries?‎ A. They contain protein. B. They are high in vitamin A.‎ C. They have a pleasant taste. D. They are rich in antioxidants.‎ ‎25. Why is fresh lemon juice used in freezing bananas?‎ A. To make them smell better. B. To keep their colour.‎ C. To speed up their ripening. D. To improve their nutrition.‎ ‎26. What is “a juicer” in the last paragraph?‎ A. A dessert. B. A drink.‎ C. A container. D. A machine.‎ ‎27. From which is the text probably taken?‎ A. A biology textbook. B. A health magazine.‎ C. A research paper. D. A travel brochure.‎ ‎【答案】24. C 25. B 26. D 27. B ‎【解析】本文是一篇日常生活类说明文。文中讲述了人们热爱水果丰盛的7月,这时候各种水果营养丰富且含有对人体有益的微量元素,尤其是香蕉。我们可以利用它做一些孩子喜欢的甜点或冰淇淋。‎ ‎24. 细节理解题。题干问的是,作者喜欢樱桃什么。根据第二段中As for cherries (樱桃), they are so delicious who cares?(至于樱桃,因为它们很好吃谁在乎呢?)可知,作者在乎的是它的美味。故选C。‎ ‎25. 细节理解题。根据第三段中的If you like, a squeeze of fresh lemon juice on the bananas will prevent them turning brown.可知,往香蕉上滴新鲜的柠檬汁是为了防止香蕉变成褐色,故新鲜的柠檬汁是被用来保持香蕉的颜色的。故选B。‎ ‎26. 词义猜测题。根据最后一段中they love feeding the fruit and frozen bananas into the top of the machine and watching the ice cream come out below可知,孩子们喜欢把一些水果和冷冻的香蕉放入到这台机器的上部,然后看到冰激凌从下面出来。故可以推出a juicer就是一台机器。故选D。‎ ‎27. 文章出处题。文章首先指出七月是水果盛产的季节,并指出各种水果富含的营养,最后一段指出我们可以用a juicer为孩子们做一些甜点和冰激凌,故最可能是从健康杂志上摘取的文章。A项意为:生物教科书;B项意为:一本健康杂志;C项意为:一篇研究论文;D项意为:一本旅游手册。故选B。‎ 点睛:做词义猜测题时我们要注意观察,划线单词的构成结构。如本题背划线的单词是juicer,我们都知道“er”是单词的后缀。它既可以表示人,又可以表示物。如:cleaner既可以表示清洁工,又可以表示清洁工具。根据下文中的意思,可以判断出本题中的a juicer是表示物。‎ C【2019·全国II】‎ ‎ Teens and younger children are reading a lot less for fun, according to a Common Sense Media report published Monday.‎ ‎ While the decline over the past decade is steep for teen readers, some data in the report shows that reading remains a big part of many children’s lives, and indicates how parents might help encourage more reading.‎ ‎ According to the report’s key findings, “the proportion (比例) who say they ‘hardly ever’ read for fun has gone from 8 percent of 13-year-olds and 9 percent of 17-year-olds in 1984 to 22 percent and 27 percent respectively today.”‎ ‎ The report data shows that pleasure reading levels for younger children, ages 2—8, remain largely the same. But the amount of time spent in reading each session has declined, from closer ‎ to an hour or more to closer to a half hour per session.‎ ‎ When it comes to technology and reading, the report does little to counsel(建议) parents looking for data about the effect of e-readers and tablets on reading. It does point out that many parents still limit electronic reading, mainly due to concerns about increased screen time.‎ ‎ The most hopeful data shared in the report shows clear evidence of parents serving as examples and important guides for their kids when it comes to reading. Data shows that kids and teens who do read frequently, compared to infrequent readers, have more books in the home, more books purchased for them, parents who read more often, and parents who set aside time for them to read.‎ ‎ As the end of school approaches, and school vacation reading lists loom(逼近) ahead, parents might take this chance to step in and make their own summer reading list and plan a family trip to the library or bookstore.‎ ‎28. What is the Common Sense Media report probably about?‎ A. Children’s reading habits.‎ B. Quality of children’s books.‎ C. Children’s after-class activities.‎ D. Parent-child relationships.‎ ‎29. Where can you find the data that best supports "children are reading a lot less for fun"?‎ A. In paragraph 2. B. In paragraph 3.‎ C. In paragraph 4. D. In paragraph 5.‎ ‎30. Why do many parents limit electronic reading?‎ A. E-books are of poor quality.‎ B. It could be a waste of time.‎ C. It may harm children’s health.‎ D. E-readers are expensive.‎ ‎31. How should parents encourage their children to read more?‎ A. Act as role models for them.‎ B. Ask then to write book reports.‎ C. Set up reading groups for them.‎ D. Talk with their reading class teachers.‎ ‎【答案】28. A 29. B 30. C 31. A ‎【解析】据本周一公布的a Common Sense Media report报告显示,青少年和年幼的孩子们读书的乐趣大大减少。文中从阅读的乐趣,阅读的时间,阅读方式和父母对孩子阅读的影响等角度展示了该报告的内容。‎ ‎28. 推理判断题。题干问的是这篇报道可能是关于什么内容。根据整篇文章,我们可以看出这篇报道讲述了孩子们阅读的乐趣,孩子们阅读的时间,孩子们阅读方式和父母对孩子阅读的影响。A项意为:孩子们的阅读习惯;B项意为:孩子们所读书籍的质量;C项意为:孩子们的课后活动;D项意为:父母与孩子的关系。故选A。‎ ‎29. 推理判断题。根据第三段中的the proportion (比例) who say they ‘hardly ever’ read for fun has gone from 8 percent of 13-year-olds and 9 percent of 17-year-olds in 1984 to 22 percent and 27 percent respectively today.可知,很少为乐趣而阅读的人的比例已经分别从1984年的13岁的8%和17岁的9%上升到现在的22%和27%。也就是说,为乐趣而读书的人越来越少了。故选B。‎ ‎31. 细节理解题。根据倒数第二段The most hopeful data shared in the report shows clear evidence of parents serving as examples and important guides for their kids when it comes to reading.可知,有明显的证据表明,父母为孩子们做榜样并为孩子们作重要指导能培养孩子的阅读习惯。也即是说,父母可以通过给孩子们作榜样来鼓励孩子们阅读的。故选A。‎ 点睛:作此类型的题时,一定要读懂第一段。第一段读懂了,整篇文章大意也就迎刃而解了。明白了大意,第28题我们也不难看出答案,本文讲的肯定不是质量,不是活动也不是关系。而是习惯,即孩子们的阅读习惯。‎ B【2019·全国I】‎ Good Morning Britain’s Susanna Reid is used to grilling guests on the sofa every morning, but she is cooking up a storm in her latest role — showing families how to prepare delicious and nutritious meals on a tight budget.‎ In Save Money: Good Food, she visits a different home each week and with the help of chef Matt Tebbutt offers top tips on how to reduce food waste, while preparing recipes for under £5 per family a day. And the Good Morning Britain presenter says she’s been able to put a lot of what she’s leant into practice in her own home, preparing meals for sons, Sam,14, Finn,13, and Jack, 11.‎ ‎"We love Mexican churros, so I buy them on my phone from my local Mexican takeaway restaurant," she explains. "I pay £5 for a portion(一份), but Matt makes them for 26p a portion, because they are flour, water, sugar and oil. Everybody can buy takeaway food, but sometimes we’re not aware how cheaply we can make this food ourselves. "‎ The eight-part series(系列节自), Save Money: Good Food, follows in the footsteps of ITV’s Save Money: Good Health, which gave viewers advice on how to get value from the vast range of health products on the market.‎ With food our biggest weekly household expense, Susanna and Matt spend time with a different family each week. In tonight’s Easter special they come to the aid of a family in need of some delicious inspiration on a budget. The team transforms the family’s long weekend of celebration with less expensive but still tasty recipes.‎ ‎24. What do we know about Susanna Reid?‎ A. She enjoys embarrassing her guests. B. She has started a new programme.‎ C. She dislikes working early in the morning. D. She has had a light budget for her family.‎ ‎25. How does Matt Tebbutt help Susanna?‎ A. He buys cooking materials for her. B. He prepares food for her kids.‎ C. He assists her in cooking matters. D. He invites guest families for her.‎ ‎26. What does the author intend to do in paragraph 4?‎ A. Summarize the previous paragraphs. B. Provide some advice for the readers.‎ C. Add some background information. D. Introduce a new topic for discussion.‎ ‎27. What can be a suitable title for the text?‎ A. Keeping Fit by Eating Smart B. Balancing Our Daily Diet C. Making yourself a Perfect Chef D. Cooking Well for Less ‎【答案】24. B 25. C 26. C 27. D ‎【解析】本文是一篇说明文。文章介绍了一档英国系列电视节目,给观众介绍如何减少食物浪费以及如何以较少的预算做出美味佳肴。‎ ‎24. 细节理解题。根据文章第一段知道Good Morning Britain’s Susanna Reid is used to grilling guests on the sofa every morning, but she is cooking up a storm in her latest role可知,她开辟了一个新的节目。故选B。‎ ‎25. 细节理解题。根据文章第二段中的In Save Money: Good Food, she visits a different home each week and with the help of chef Matt Tebbutt offers top tips on how to reduce food waste, while preparing recipes for under £5 per family a day.可知, Susanna 在Matt Tebbutt的帮助下,提供如何减少食物浪费同时给每日生活费低于5英镑的每个家庭准备食谱。故选C。解题关键词:同义词表达with the help of和help。‎ ‎26. 写作意图题。根据文章第四段中的which gave viewers advice on how to get value from the vast range of health products on the market.可知,Save Money: Good Food节目是Save Money: Good Health节目之后,给观众一些建议:如何从众多的市场上的健康产品中获取价值。故选C。‎ ‎27. 主旨要义题。根据文章的整体内容可知,文章作者一直在讲如何用较少的钱做出好的食物。根据文章中的prepare delicious and nutritious meals on a tight budget. 在资金紧张的情况下,准备可口且有营养的饭菜; how to reduce food waste, while preparing recipes for under £5 per family a day. 如何减少食物浪费同时给每日生活费低于5英镑的每个家庭准备食谱; how cheaply we can make this food ourselves. 我们自己做这种食物有多便宜; less expensive but still tasty recipes.不贵可仍然可口的食谱。可以推知D正确。‎ 点睛:本篇文章学生感到比较难的是26题,很多学生不知道作者在说什么,不明白作者的意图,感到很茫然。根据文章中的Save Money: Good Food, follows in the footsteps of ITV's Save Money: Good Health,…可知作者是为读者介绍了Save Money这个节目是怎么来的,即是背景信息。‎ D【2019·全国II】‎ We’ve all been there: in a lift, in line at the bank or on an airplane, surrounded by people who are, like us, deeply focused on their smartphones or, worse, struggling with the uncomfortable silence. ‎ ‎ What’s the problem? It’s possible that we all have compromised conversational intelligence. It’s more likely that none of us start a conversation because it’s awkward and challenging, or we think it’s annoying and unnecessary. But the next time you find yourself among strangers, consider that small talk is worth the trouble. Experts say it’s an invaluable social practice that results in big benefits.‎ ‎ Dismissing small talk as unimportant is easy, but we can’t forget that deep relationships wouldn’t ‎ even exist if it weren’t for casual conversation. Small talk is the grease(润滑剂) for social communication, says Bernardo Carducci, director of the Shyness Research Institute at Indiana University Southeast. "Almost every great love story and each big business deal begins with small talk," he explains. "The key to successful small talk is learning how to connect with others, not just communicate with them."‎ In a 2019 study, Elizabeth Dunn, associate professor of psychology at UBC, invited people on their way into a coffee shop. One group was asked to seek out an interaction(互动) with its ‎ waiter; the other, to speak only when necessary. The results showed that those who chatted with their server reported significantly higher positive feelings and a better coffee shop experience. "It’s not that talking to the waiter is better than talking to your husband," says Dunn. "But interactions with peripheral(边缘的) members of our social network matter for our well-being also."‎ Dunn believes that people who reach out to strangers feel a significantly greater sense of belonging, a bond with others. Carducci believes developing such a sense of belonging starts with small talk. "Small talk is the basis of good manners," he says.‎ ‎32. What phenomenon is described in the first paragraph?‎ A. Addiction to smartphones.‎ B. Inappropriate behaviours in public places.‎ C. Absence of communication between strangers.‎ D. Impatience with slow service.‎ ‎33. What is important for successful small talk according to Carducci?‎ A. Showing good manners. B. Relating to other people.‎ C. Focusing on a topic. D. Making business deals.‎ ‎34. What does the coffee-shop study suggest about small talk?‎ A. It improves family relationships. B. It raises people’s confidence.‎ C. It matters as much as a formal talk. D. It makes people feel good.‎ ‎35. What is the best title for the text?‎ A. Conversation Counts B. Ways of Making Small Talk C. Benefits of Small Talk D. Uncomfortable Silence ‎【答案】32. C 33. B 34. D 35. C ‎【解析】这是一篇议论文。在当今社会,人们在公共场合或沉迷于智能手机,或与不舒服的沉默抗争,陌生人之间缺乏沟通。但人与人之间是需要适当的交谈闲聊的,闲聊是人际关系社会交往必不可少的部分,而且也有很多好处。‎ ‎33. 推理判断题。题干问得是对于Carducci来说,成功的闲聊中重要的是什么。根据第三段最后一句“The key to successful small talk is learning how to connect with others, not just communicate with ‎ them”(成功闲聊的关键是学习如何与他们交流,而不仅仅是与他们沟通。)由此推断C符合题意。A项意为:表现出良好的礼貌。B项意为:与他人有关的。C项意为:专注于一个话题。D项意为:做商业交易。故选B项。‎ ‎34. 推理判断题。题干问的是:咖啡店的研究对闲聊有什么建议。根据第四段的调查结果可知,那些与服务员聊天的人,有显著的积极情绪和更好的咖啡店体验。由此可知,D项符合题意。A项意为:闲聊改善了家庭关系。B项意为:闲聊提高了人们的信心。C项意为:闲聊和正式谈话一样重要。D项意为:闲聊让人感觉很好。故选D项。‎ ‎35. 主旨大意题。整篇文章刚开始介绍了社会的现象(公共场合人们沉迷于智能手机,陌生人之间缺乏沟通交流),接着分析了这一问题的原因,接下来有专家对闲聊进行了研究,最后得出结论,闲聊都有什么样的好处。A项意为:谈话很重要。B项意为:闲聊的方法。C项意为:闲聊的好处。D项意为:不舒服的沉默。故选C项。‎ 点睛:我们在做阅读理解时,尽量找出每段的主讲内容(可能是一句话或是一个短语),这样有利于整篇文章的把握,有利于做文章大意题。如35题,很显然本文讲的不是重要性也不是方法更不是沉默,所以ABD都可以轻松排除。‎ B【2019·全国III】‎ Cities usually have a good reason for being where they are, like a nearby port or river. People settle in these places because they are easy to get to and naturally suited to communications and trade. New York City, for example, is near a large harbour at the mouth of the Hudson River. Over 300 years its population grew gradually from 800 people to 8 million. But not all cities develop slowly over a long period of time. Boom towns grow from nothing almost overnight. In 1896, Dawson, Canada, was unmapped wilderness(荒野). But gold was discovered there in 1897, and two years later, it was one of the largest cities in the West, with a population of 30,000.‎ Dawson did not have any of the natural conveniences of cities like London or Paris. People went there for gold. They travelled over snow-covered mountains and sailed hundreds of miles up icy rivers. The path to Dawson was covered with thirty feet of wet snow that could fall without warming. An avalanche(雪崩) once closed the path, killing 63 people. For many who made it to Dawson, however, the rewards were worth the difficult trip. Of the first 20,000 people who dug for gold, 4,000 got rich. About 100 of these stayed rich men for the rest of their lives.‎ But no matter how rich they were, Dawson was never comfortable. Necessities like food and wood were very expensive. But soon, the gold that Dawson depended on had all been found. The city was crowded with disappointed people with no interest in settling down, and when they heard there were new gold discoveries in Alaska, they left Dawson City as quickly as they had come. Today,‎ ‎ people still come and go — to see where the Canadian gold rush happened. Tourism is now the chief industry of Dawson City — its present population is 762.‎ ‎24. What attracted the early settlers to New York City?‎ A. Its business culture.‎ B. Its small population.‎ C. Its geographical position.‎ D. Its favourable climate.‎ ‎25. What do we know about those who first dug for gold in Dawson?‎ A. Two-thirds of them stayed there.‎ B. One out of five people got rich.‎ C. Almost everyone gave up.‎ D. Half of them died.‎ ‎26. What was the main reason for many people to leave Dawson?‎ A. They found the city too crowded.‎ B. They wanted to try their luck elsewhere.‎ C. They were unable to stand the winter.‎ D. They were short of food.‎ ‎27. What is the text mainly about?‎ A. The rise and fall of a city.‎ B. The gold rush in Canada.‎ C. Journeys into the wilderness.‎ D. Tourism in Dawson.‎ ‎【答案】24. C 25. B 26. B 27. A ‎【解析】这是一篇说明文。文章主要介绍了Dawson这座城市的发展原因、过程与现状。‎ ‎24. 细节理解题。根据文章第一段中Cities usually have a good reason for being where they are, like a nearby port or river. People settle in these places because they are easy to get to and naturally suited to communications and trade. New York City, for example, is near a large harbour at the mouth of the Hudson River.可知,人们选择在河边或港口设城是因为交通方便,便于做生意。而纽约就是在哈德森河口附近的一个大港口,故纽约吸引早期移民的原因是它的地理位置,故C正确。‎ ‎25. 细节理解题。根据文章第二段最后一句Of the first 20,000 people who dug for gold, 4,000 got ‎ rich. About 100 of these stayed rich men for the rest of their lives.可知,在最初挖黄金的两万人中有4000人变富有,所以是五分之一的人变富了,故B正确。‎ ‎26. 细节理解题。根据文章最后一段中and when they heard there were new gold discoveries in Alaska, they left Dawson City as quickly as they had come.可知,人们离开Dawson的主要原因是听说在Alaska发现了黄金,也就是他们要去别的地方寻找发财的机会。故B正确。‎ ‎27. 主旨大意题。第一段简要介绍城市发展的原因,引出Dawson这一城市的兴起,第二段介绍了该城市兴起的原因,第三段介绍人们选择离开该城市的原因及现在的状况,所以全文围绕Dawson这个城市的发展起伏。故A正确。‎ ‎【点睛】主旨大意题的解题技巧:一篇文章都是围绕着一个中心思想或一个话题展开的,因此,有的文章中最明显的特点之一是有一个反复出现的中心词,即高频词,也叫做主题词。抓住了它,便容易抓住文章的中心。通读全文可知,文章主要围绕Dawson的发展展开,Dawson反复出现,再根据每一段的段首句可以总结出,文章第一段通过城市发展的原因引出Dawson,第二段介绍Dawson发展的原因,第三段介绍人们选择离开该城市的原因及现在的状况,从而总结出文章的主旨大意。‎ C【2019·浙江】‎ As cultural symbols go, the American car is quite young. The Model T Ford was built at the Piquette Plant in Michigan a century ago, with the first rolling off the assembly line(装配线) on September 27, 1908. Only eleven cars were produced the next month. But eventually Henry Ford would build fifteen million of them.‎ ‎ Modern America was born on the road, behind a wheel. The car shaped some of the most lasting aspects of American culture: the roadside diner, the billboard, the motel, even the hamburger. For most of the last century, the car represented what it meant to be American—going forward at high speed to find new worlds. The road novel, the road movie, these are the most typical American ideas, born of abundant petrol, cheap cars and a never-ending interstate highway system, the largest public works project in history.‎ ‎ In 1928 Herbert Hoover imagined an America with “a chicken in every pot and a car in every garage.” Since then, this society has moved onward, never looking back, as the car transformed America from a farm-based society into an industrial ‎ The cars that drove the American Dream have helped to create a global ecological disaster. In America the demand for oil has grown by 22 percent since 1990.‎ ‎ The problems of excessive(过度的)energy consumption, climate change and population growth ‎ have been described in a book by the American writer Thomas L. Friedman. He fears the worst, but hopes for the best.‎ ‎ Friedman points out that the green economy(经济)is a chance to keep American strength. “The ability to design, build and export green technologies for producing clean water, clean air and healthy and abundant food is going to be the currency of power in the new century.”‎ ‎28. Why is hamburger mentioned in paragraph 2?‎ A. To explain Americans’ love for travelling by car.‎ B. To show the influence of cars on American culture.‎ C. To stress the popularity of fast food with Americans.‎ D. To praise the effectiveness of America’s road system.‎ ‎29. What has the use of cars in America led to?‎ A. Decline of economy. B. Environmental problems.‎ C. A shortage of oil supply. D. A farm-based society.‎ ‎30. What is Friedman’s attitude towards America’s future?‎ A. Ambiguous. B. Doubtful. C. Hopeful. D. Tolera ‎【答案】28. B 29. B 30. C ‎【解析】文章讲述了汽车在美国经济和文化上的重要作用,也指出了汽车带来的环境问题。‎ ‎28. 推理判断题。根据第二段Modern America was born on the road, behind a wheel. The car shaped some of the most lasting aspects of American culture: the roadside diner, the billboard, the motel, even the hamburger.可知现代美国诞生于公路和汽车,汽车塑造了美国文化最持久的一些方面。Hamburger就是汽车塑造的美国文化的一个方面。用这个例子是在说明汽车对美国文化的影响,故选B。‎ ‎【点睛】‎ 细节理解题具体方法:‎ ‎1.略读材料,大概了解原文,掌握中心或主旨。‎ ‎2.按文章的体裁,作者写作的组织模式及有关的信息词。如for example, first, second…等预测应该到何处寻找自己所需要的事实。‎ ‎3.将精力放在寻找你所需要的细节上。快速通篇跳读,全文扫视,找到细节出处,待找到含细节句子时,放慢速度,仔细核对比较内容,直至找到答案。如小题1要求分析第二段提到hamburger的作用,根据第二段Modern America was born on the road, behind a wheel. The car shaped some of the most lasting aspects of American culture: the roadside diner, the billboard, the motel, even the ‎ hamburger.可知现代美国诞生于公路和汽车,汽车塑造了美国文化最持久的一些方面。Hamburger就是汽车塑造的美国文化的一个方面。用这个例子是在说明汽车对美国文化的影响。‎ D【2019·天津】‎ Give yourself a test. Which way is the wind blowing? How many kinds of wildflowers can be seen from your front door? If your awareness is as sharp as it could be, you’ll have no trouble answering these questions.‎ Most of us observed much more as children than we do as adults. A child’s day is filled with fascination, newness and wonder. Curiosity gave us all a natural awareness. But distinctions that were sharp to us as children become unclear; we are numb(麻木的)to new stimulation(刺激), new ideas. Relearning the art of seeing the world around us is quite simple, although it takes practice and requires breaking some bad habits.‎ The first step in awakening senses is to stop predicting what we are going to see and feel before it occurs. This blocks awareness. One chilly night when I was hiking in the Rocky Mountains with some students, I mentioned that we were going to cross a mountain stream. The students began complaining about how cold it would be. We reached the stream, and they unwillingly walked ahead. They were almost knee-deep when they realized it was a hot spring. Later they all admitted they’d felt cold water at first.‎ Another block to awareness is the obsession(痴迷) many of us have with naming things. I saw bird watchers who spotted a bird, immediately looked it up in field guides, and said, a "ruby-crowned kinglet" and checked it off. They no longer paid attention to the bird and never learned what it was doing.‎ The pressures of "time" and "destination" are further blocks to awareness. I encountered many hikers who were headed to a distant camp-ground with just enough time to get there before dark. It seldom occurred to them to wander a bit, to take a moment to see what’s around them. I asked them what they’d seen. "Oh, a few birds," they said. They seemed bent on their destinations.‎ Nature seems to unfold to people who watch and wait. Next time you take a walk, no matter where it is, take in all the sights, sounds and sensations. Wander in this frame of mind and you will open a new dimension to your life.‎ ‎51. According to Paragraph 2, compared with adults, children are more ____________.‎ A. anxious to do wonders B. sensitive to others’ feelings C. likely to develop unpleasant habits D. eager to explore the world around them ‎52. What idea does the author convey in Paragraph 3?‎ A. To avoid jumping to conclusions.‎ B. To stop complaining all the time.‎ C. To follow the teacher’s advice.‎ D. To admit mistakes honestly.‎ ‎53. The bird watchers’ behavior shows that they __________.‎ A. are very patient in their observation B. are really fascinated by nature C. care only about the names of birds D. question the accuracy of the field guides ‎54. Why do the hikers take no notice of the surroundings during the journey?‎ A. The natural beauty isn’t attractive to them.‎ B. They focus on arriving at the camp in time.‎ C. The forest in the dark is dangerous for them.‎ D. They are keen to see rare birds at the destination.‎ ‎55. In the passage, the author intends to tell us we should __________.‎ A. fill our senses to feel the wonders of the world B. get rid of some bad habits in our daily life C. open our mind to new things and ideas D. try our best to protect nature ‎【答案】51. D 52. A 53. C 54. B 55. A ‎【解析】本文是一篇散文。我们有多久没有仔细观察我们周围的世界了。作者通过此文要告诉我们:放慢脚步,带着我们所有的感官来感受周围世界的奇妙。‎ ‎51. 推理判断题。根据文章第二段Most of us observed much more as children than we do as adults.可知,与成人相比较,孩子观察得更多,从而可以推断出孩子更急于探索他们周围的世界。故选D。‎ ‎52.‎ ‎ 推理判断题。根据文章第三段作者叙述在一个寒冷的夜晚,作者和学生徒步旅行穿过一条小溪的时候,学生们抱怨水太冷而不愿往前走,结果事实上那是一个温泉。作者举这样一个事例是为了向读者传递这样的观念:避免过早下结论。故选A。‎ ‎53. 细节理解题。根据文章第四段全段及首句Another block to awareness is the obsession(痴迷) many of us have with naming things.可知,鸟观察者发现鸟后只关心鸟的名字,并不关心它在做什么。故选C。‎ ‎54. 细节理解题。根据文章第五段I encountered many hikers who were headed to a distant camp-ground with just enough time to get there before dark. It seldom occurred to them to wander a bit, to take a moment to see what’s around them.可知,徒步旅行者只关心能够及时到达目的地,而很少关心周围的事物。故选B。‎ ‎55. 推理判断题。文章作者想要通过此文要告诉我们:大自然只展现给那些善于观察和等待的人,带着我们所有的感官来感受周围世界的奇妙。‎ ‎【点睛】本文是一篇散文阅读。整个文章行文自由,不拘一格,对考生来说阅读有一定的难度。但我们只需要抓住贯穿全文的主线:大自然只展现给那些善于观察和等待的人,就不难理解全文大意了。文章多考查细节理解题和推理判断题,考生需要根据文中事实和线索作出判断和推理。‎ D【2019·北京】‎ Preparing Cities for Robot Cars The possibility of self-driving robot cars has often seemed like a futurist’s dream, years away from materializing in the real world. Well, the future is apparently now. The California Department of Motor Vehicles began giving permits in April for companies to test truly self-driving cars on public roads. The state also cleared the way for companies to sell or rent out self-driving cars, and for companies to operate driverless taxi services. California, it should be noted, isn’t leading the way here. Companies have been testing their vehicles in cities across the country. It’s hard to predict when driverless cars will be everywhere on our roads. But however long it takes, the technology has the potential to change our transportation systems and our cities, for better or for worse, depending on how the transformation is regulated.‎ While much of the debate so far has been focused on the safety of driverless cars(and rightfully so), policymakers also should be talking about how self-driving vehicles can help reduce traffic jams, cut emissions(排放) and offer more convenient, affordable mobility options. The arrival of driverless vehicles is a chance to make sure that those vehicles are environmentally friendly and more shared.‎ Do we want to copy — or even worsen — the traffic of today with driverless cars? Imagine ‎ a future where most adults own individual self-driving vehicles. They tolerate long, slow journeys to and from work on packed highways because they can work, entertain themselves or sleep on the ride, which encourages urban spread. They take their driverless car to an appointment and set the empty vehicle to circle the building to avoid paying for parking. Instead of walking a few blocks to pick up a child or the dry cleaning, they send the self-driving minibus. The convenience even leads fewer people to take public transport — an unwelcome side effect researchers have already found in ride-hailing(叫车) services.‎ A study from the University of California at Davis suggested that replacing petrol-powered private cars worldwide with electric, self-driving and shared systems could reduce carbon emissions from transportation 80% and cut the cost of transportation infrastructure(基础设施) and operations 40% by 2050. Fewer emissions and cheaper travel sound pretty appealing. The first commercially available driverless cars will almost certainly be fielded by ride-hailing services, considering the cost of self-driving technology as well as liability and maintenance issues(责任与维护问题). But driverless car ownership could increase as the prices drop and more people become comfortable with the technology.‎ ‎ Policymakers should start thinking now about how to make sure the appearance of driverless vehicles doesn’t extend the worst aspects of the car-controlled transportation system we have today. The coming technological advancement presents a chance for cities and states to develop transportation systems designed to move more people, and more affordably. The car of the future is coming. We just have to plan for it.‎ ‎47. According to the author, attention should be paid to how driverless cars can __________.‎ A. help deal with transportation-related problems B. provide better services to customers C. cause damage to our environment D. make some people lose jobs ‎48. As for driverless cars, what is the author’s major concern?‎ A. Safety. B. Side effects.‎ C. Affordability. D. Management.‎ ‎49. What does the underlined word "fielded" in Paragraph 4 probably mean?‎ A. Employed. B. Replaced.‎ C. Shared. D. Reduced.‎ ‎50. What is the author’s attitude to the future of self-driving cars?‎ A. Doubtful. B. Positive.‎ C. Disapproving. D. Sympathetic.‎ C7[2019•北京卷] ‎ D ‎【要点综述】 本文是一篇社会生活类文章。文章围绕无人驾驶汽车展开,展示了无人驾驶汽车的美好前景以及其必将到来的趋势,呼吁政府采取措施,使无人驾驶汽车为人们的生活带来更多积极的影响。‎ ‎47. A 细节理解题。根据文章第一段中的“But however long it takes, the technology has the potential to change our transportation systems and our cities, for better or for worse, depending on how the transformation is regulated”以及后文中作者所描述的这些汽车带来的有利影响可知,作者认为我们应该注意如何让这些汽车来帮助我们解决与交通有关的问题。故选A。‎ ‎49. A 词义猜测题。联系上下文,“考虑到无人驾驶技术的可靠性和责任与维护问题,第一批商用无人驾驶汽车几乎会被全部投放到打车行业”。故选A。‎ ‎50. B 写作意图题。由文章的最后一段以及文章很大篇幅都在讲无人驾驶汽车的优点可知,作者对无人驾驶汽车怀有积极态度。故选B。‎ B【2019·江苏】‎ ‎ In the 1760s, Mathurin Roze opened a series of shops that boasted(享有)a special meat soup called consomme. Although the main attraction was the soup, Roze's chain shops also set a new standard for dining out, which helped to establish Roze as the inventor of the modern restaurant.‎ ‎ Today, scholars have generated large amounts of instructive research about restaurants. Take visual hints that influence what we eat: diners served themselves about 20 percent more pasta(意大利面食)when their plates matched their food. When a dark-colored cake was served on a black plate rather than a white one, customers recognized it as sweeter and more tasty.‎ ‎ Lighting matters, too. When Berlin restaurant customers ate in darkness, they couldn't tell how much they'd had: those given extra-large shares ate more than everyone else, but were none the wiser—they didn’t feel fuller, and they were just as ready for dessert.‎ ‎ Time is money, but that principle means different things for different types of restaurants.‎ ‎ Unlike fast-food places. fine dining shops prefer customers to stay longer and spend. One way to encourage customers to stay and order that extra round: put on some Mozart(莫扎特).When classical, rather than pop, music was playing, diners spent more. Fast music hurried diners out.‎ Particular scents also have an effect: diners who got the scent of lavender(薰衣草)stayed longer and spent more than those who smelled lemon, or no scent.‎ ‎ Meanwhile, things that you might expect to discourage spending—"bad" tables, crowding.‎ high prices — don't necessarily. Diners at bad tables — next to the kitchen door, say — spent nearly as much as others but soon fled. It can be concluded that restaurant keepers need not "be overly concerned about ‘bad' tables," given that they're profitable. As for crowds, a Hong Kong study found that they increased a restaurant's reputation, suggesting great food at fair prices. And doubling a buffet's price led customers to say that its pizza was 11 percent tastier.‎ ‎58. The underlined phrase "none the wiser" in paragraph 3 most probably implies that the ‎ ‎ customers were .‎ ‎ A. not aware of eating more than usual ‎ ‎ B. not willing to share food with others ‎ C. not conscious of the food quality ‎ ‎ D. not fond of the food provided ‎59. How could a fine dining shop make more profit?‎ ‎ A. playing classical music.‎ ‎ B. Introducing lemon scent.‎ ‎ C. Making the light brighter,‎ ‎ D. Using plates of larger size.‎ ‎60. What does the last paragraph talk about?‎ ‎ A. Tips to attract more customers.‎ ‎ B. Problems restaurants are faced with.‎ ‎ C. Ways to improve restaurants' reputation.‎ ‎ D. Common misunderstandings about restaurants.‎ ‎【文章大意】本文是 一篇议论文。论述了现代餐馆面临的经营困境和解决方案,通过对比快餐店和正规餐馆提出,现代餐饮业可以通过味道(比如薰衣草相比柠檬更能刺激消费者的食欲)、灯光的明暗(比如暗的灯光更能够刺激顾客食欲)等吸引顾客。 ‎ ‎58.A 词义猜测题。根据该句中“…they didn’t feel fuller, and they were just as ready for dessert”可知,他们没有感觉更饱,想要再吃一点甜点;据此可以判断,划线词表示“他们没有意识到比平时吃得多”,故选A项。‎ ‎59.A 细节理解题。根据第四段第三、四句“One way to encourage customers to stay and order that extra round: put on some Mozart (莫扎特). When classical, rather than pop, music was playing, diners spent more.”可知, 在餐馆播放古典音乐能延长顾客在餐馆的时间,从而促进餐馆赢得更多的利润,故选A项。‎ ‎60.D 段意归纳题。通读尾段可知,该段第一句“Meanwhile, things that you might expect to discourage spending — “bad tables, crowding, high prices — don’t necessarily”为该段的主题段;结合全段内容可知,该段主要论述了人们对餐馆的常见误解,故选D项。‎ C【2019·江苏】‎ ‎ If you want to disturb the car industry, you'd better have a few billion dollars: Mom-and-pop carmakers are unlikely to beat the biggest car companies. But in agriculture, small farmers can get the best of the major players. By connecting directly with customers, and by responding quickly to changes in the markets as well as in the ecosystems(生态系统), small farmers can keep one step ahead of the big guys. As the co-founder of the National Young Farmers Coalition (NYFC, 美国青年农会)and a family farmer myself. I have a front-row seat to the innovations among small farmers that are transforming the industry.‎ ‎ For example, take the Quick Cut Greens Harvester, a tool developed just a couple of years ago by a young farmer, Jonathan Dysinger, in Tennessee, with a small loan from a local Slow Money group. It enables small-scale farmers to harvest 175 pounds of green vegetables per hour—a huge improvement over harvesting just a few dozen pounds by hand—suddenly making it possible for the little guys to compete with large farms of California. Before the tool came out, small farmers couldn't touch the price per pound offered by California farms. But now, with the combination of a better price point and a generally fresher product, they can stay in business.‎ ‎ The sustainable success of small farmers, though, won't happen without fundamental changes to the industry. One crucial factor is secure access to land. Competition from investors. developers, and established large farmers makes owning one's own land unattainable for many new farmers.‎ From 2019 to 2019, agricultural land values doubled, and they continue to rise in many regions.‎ ‎ Another challenge for more than a million of the most qualified farm workers and managers is a non-existent path to citizenship — the greatest barrier to building a farm of their own. With farmers over the age of 65 outnumbering(多于)farmers younger than 35 by six to one, and with two-thirds of the nation's farmland in need of a new farmer, we must clear the path for talented people willing to grow the nation's food.‎ ‎ There are solutions that could light a path toward a more sustainable and fair farm economy, but farmers can't clumsily put them together before us. We at the NYFC need broad support as we urge Congress to increase farmland conservation, as we push for immigration reform, and as we seek policies that will ensure the success of a diverse and ambitious next generation of farms from all backgrounds. With a new farm bill to be debated in Congress, consumers must take a stand with young farmers.‎ ‎61. The author mentions car industry at the beginning of the passage to introduce .‎ ‎ A. the progress made in car industry ‎ ‎ B. a special feature of agriculture ‎ C. a trend of development in agriculture ‎ ‎ D. the importance of investing in car industry ‎62. What does the author want to illustrate with the example in paragraph 2?‎ ‎ A. Loans to small local farmers are necessary.‎ ‎ B. Technology is vital for agricultural development.‎ ‎ C. Competition between small and big farms is fierce ‎ D. Small farmers may gain some advantages over big ones.‎ ‎63. What is the difficulty for those new famers?‎ ‎ A. To gain more financial aid.‎ ‎ B. To hire good farm managers.‎ ‎ C. To have fans of their own.‎ ‎ D. To win old farmers’ support.‎ ‎64. What should farmers do for a more sustainable and fair farm economy?‎ ‎ A. Seek support beyond NYFC.‎ ‎ B. Expand farmland conservation.‎ ‎ C. Become members of NYFC.‎ ‎ D. Invest more to improve technology.‎ ‎【文章大意】文章主要阐述了美国的小农场主所持有的优势,同时也分析了目前美国农业所面临的问题,比如美国农业目前很多人不愿意卖地,且60岁以上的农民数目远远高出年轻农民。‎ ‎62. D推理判断题。文章第二段是通过事例来证明第一段“…small farmers can keep one step ahead of the big guys.”小农场也可能会超过大型农场可知,小农场也可能比大农场先占领先机,故选D项。‎ ‎63. C 细节理解题。根据第三段“Competition from investors, developers, and established large farmers makes owning one’s own land unattainable for many new farmers.” 来自投资者、开发商和老牌大农场主的竞争使得许多新农民无法拥有自己的土地可知,对于新型的农民来说要拥有自己的土地,即自己的农场还是困难重重的,故选C项。‎ ‎64. A细节理解题。根据第四段“We at the NYFC need broad support as we urge Congress to increase farmland conservation, as we push for immigration reform, and as we seek policies that will ensure the success of a diverse and ambitious next generation of farmers from all backgrounds. With a new farm bill to be debated in Congress, consumers must take a stand with young farmers.”可知,除了在敦促国会增加耕地保护的同时,在推动移民改革的同时,在寻求确保不同背景、雄心勃勃的下一代农民成功的政策时,农民应该为一个更可持续、更公平的农业经济寻求更多的支持,故选A项。 ‎ ‎2019年高考题 ‎ [2019·全国卷Ⅰ]‎ C Some of the world's most famous musicians recently gathered in Paris and New Orleans to celebrate the first annual International Jazz Day. UNESCO(United Nations Educational, Scientific and Cultural Organization) recently set April 30 as a day to raise awareness of jazz music, its significance, and its potential as a unifying(联合) voice across cultures.‎ Despite the celebrations, though, in the US the jazz audience continues to shrink and grow older, and the music has failed to connect with younger generations.‎ It's Jason Moran's job to help change that. As the Kennedy Centre's artistic adviser for jazz, Moran hopes to widen the audience for jazz, make the music more accessible, and preserve its history and culture.‎ ‎“Jazz seems like it's not really a part of the American appetite,” Moran tells National Public Radio's reporter Neal Conan. “What I'm hoping to accomplish is that my generation and ‎ younger start to reconsider and understand that jazz is not black and white anymore. It's actually colour, and it's actually digital.”‎ Moran says one of the problems with jazz today is that the entertainment aspect of the music has been lost. “The music can't be presented today the way it was in 1908 or 1958. It has to continue to move, because the way the world works is not the same,” says Moran.‎ Last year, Moran worked on a project that arranged Fats Waller's music for a dance party, “just to kind of put it back in the mind that Waller is dance music as much as it is concert music,” says Moran. “For me, it's the recontextualization. In music, where does the emotion(情感) lie? Are we, as humans, gaining any insight(感悟) on how to talk about ourselves and how something as abstract as a Charlie Parker record gets us into a dialogue about our emotions and our thoughts? Sometimes we lose sight that the music has a wider context,” says Moran, “so I want to continue those dialogues. Those are the things I want to foster.”‎ ‎28.Why did UNESCO set April 30 as International Jazz Day?‎ A.To remember the birth of jazz.‎ B.To protect cultural diversity.‎ C.To encourage people to study music.‎ D.To recognize the value of jazz.‎ ‎29.What does the underlined word “that” in Paragraph 3 refer to?‎ A.Jazz becoming more accessible.‎ B.The production of jazz growing faster.‎ C.Jazz being less popular with the young.‎ D.The jazz audience becoming larger. ‎ ‎30.What can we infer about Moran's opinion on jazz?‎ A.It will disappear gradually.‎ B.It remains black and white.‎ C.It should keep up with the times.‎ D.It changes every 50 years.‎ ‎31.Which of the following can be the best title for the text?‎ A.Exploring the future of jazz B.The rise and fall of jazz C.The story of a jazz musician D.Celebrating the Jazz Day ‎【文章大意】 本文是一篇说明文。文章讲述了为提高人们对于爵士乐的重视程度,联合国教科文组织把4月30号定为国际爵士乐日。然而这一行为还是没能挽救爵士乐。Jason Moran认为时代在进步,为了将老一代的人和年轻一代的人联接起来,爵士乐应该也随之进步。‎ ‎28. D 细节理解题。根据文章第一段中的“UNESCO (United Nations Educational, Scientific and Cultural Organization) recently set April 30 as a day to raise awareness of jazz music, its significance, and its potential as a unifying(联合) voice across cultures.”可知,联合国教科文组织把4月30号定为国际爵士乐日是为了让人们重视爵士乐,意识到它的重要性以及它作为联接各文化的纽带的潜在功能,也就是为了让人们意识到爵士乐的价值。故选D。‎ ‎29. C 代词指代题。根据前文“Despite the celebrations, though, in the US the jazz audience continues to shrink and grow older, and the music has failed to connect with younger generations.”可知,尽管联合国教科文组织为爵士乐设定了纪念日,但美国的爵士乐听众依然在减少,并且年龄在老化。爵士乐没能将年轻一代人与老一代人联接起来。再结合“It's Jason Moran's job to help change that.”可知,Jason Moran的工作就是帮助改变上述情况,所以可推测that指代的是前文中爵士乐在年轻一代人中失去吸引力的现象。故选C。‎ ‎30. C 推理判断题。根据文章第五段中的“‘The music can't be presented today the way it was in 1908 or 1958. It has to continue to move, because the way the world works is not the same…’”可知,Moran认为爵士乐不能以1908或1958年的方式来呈现了,因为世界运转的方式不同,所以爵士乐必须继续前进才行。这说明随着时代的发展,爵士乐也要跟上时代才不会被年轻一代所抛弃。故选C。‎ ‎31. A 主旨大意题。通读全文可知本文的主要内容是联合国教科文组织为提高人们对爵士乐的重视设立爵士乐日,但实际收效甚微。有人认为爵士乐应随着时代的进步而进步,否则将失去吸引力。本文主要探索爵士乐的未来,故选A。‎ ‎【名师点睛】‎ 通常情况下,文章的标题具有概括性、针对性、醒目性的特点。概括性就是指标题应在最大程度上覆盖全文,囊括文章的主要内容,体现文章的主题。针对性是对标题外延的一种界定,标题要直接指向文章的主要特点。而醒目性是为了吸引读者的注意力,唤起读者对文章阅读的兴趣。考试中标题选择的题型一般是基础标题的概括性进行设题。解答这一类题型,考生可以从以下几点着手:‎ 一、在阅读文章时,要注意文章中反复出现或强调的信息,寻找与文章大多数内容相关的信息,找出覆盖全文的核心词汇,看选项内容是否切中文章的中心论题,也就是要看选项内容与作者的写作目的是否一致。‎ 二、了解文章主要论题的关键是找到全文的主题句。主题句通常在文章首段,但是也可能在文章的末段、在文章的某一段落,或者分散在文章的各个段落中。如果每个段落都有主题句,那么把各段落的主题句的中心思想集中起来,即为全文的主题句。‎ 三、逆向思维法 如果考生觉得文章大意难以总结,不妨采取逆向思维,从四个选项下手。也就是思考四个选项:如果我是作者,遇到这样的标题,文章应该写些什么内容?‎ 例如第31题,考生可采用逆向思维法,思考四个选项分别该怎么写。如果是A选项Exploring the Future of jazz,那么文章应该会讲到爵士乐的现状,以及就未来发展提出一些看法或者建议。如果是B选项The Rise and Fall of jazz.为文章标题,那么文章至少涉及两个方面,也就是爵士乐的繁荣时期和衰败时期。但回到这篇文章,很明显并没有描述前者。至于衰败时期,文中只提到爵士乐在年轻一代人中不那么受欢迎,“衰败”就属于过度推测了。如果是C选项The Story of a jazz Musician.,那么文章讲的应该是一个音乐家。这不符合本文的描述对象,所以可以排除。至于D选项Celebrating the Jazz Day.“庆祝爵士日”,文中既没有提到庆祝方式,也没有讲到庆祝地点之类的相关事物,唯一与之相关的是在第一段UNESCO设立爵士日。所以D选项也不是文章大意的体现。综合比较,A选项是文章标题的最佳选择。‎ ‎ [2019·全国卷Ⅱ]‎ D When a leafy plant is under attack,it doesn't sit quietly. Back in 1983,two scientists,Jack Schultz and Ian Baldwin,reported that young maple trees getting bitten by insects send out a particular smell that neighbouring plants can get. These chemicals come from the injured parts of the plant and seem to be an alarm. What the plants pump through the air is a mixture of chemicals known as volatile organic compounds,VOCs for short.‎ Scientists have found that all kinds of plants give out VOCs when being attacked. It's a plant's way of crying out. But is anyone listening?Apparently. Because we can watch the neighbours react.‎ Some plants pump out smelly chemicals to keep insects away. But others do double duty. They pump out perfumes designed to attract different insects who are natural enemies to the attackers. Once they arrive, the tables are turned. The attacker who was lunching now becomes lunch.‎ In study after study,it appears that these chemical conversations help the neighbours. The damage is usually more serious on the first plant,but the neighbours,relatively speaking,stay safer because they heard the alarm and knew what to do.‎ Does this mean that plants talk to each other? Scientists don't know. Maybe the first plant just made a cry of pain or was sending a message to its own branches, and so, in effect, was talking to itself. Perhaps the neighbours just happened to “overhear” the cry. So information was exchanged, but it wasn't a true, intentional back and forth.‎ Charles Darwin, over 150 years ago, imagined a world far busier, noisier and more intimate (亲密的) than the world we can see and hear. Our senses are weak. There's a whole lot going on.‎ ‎32.What does a plant do when it is under attack? ‎ A.It makes noises. ‎ B.It gets help from other plants.‎ C.It stands quietly. ‎ D.It sends out certain chemicals.‎ ‎33.What does the author mean by “the tables are turned” in Paragraph 3?‎ A.The attackers get attacked.‎ B.The insects gather under the table.‎ C.The plants get ready to fight back.‎ D.The perfumes attract natural enemies.‎ ‎34.Scientists find from their studies that plants can ________.‎ A.predict natural disasters ‎ B.protect themselves against insects C.talk to one another intentionally ‎ D.help their neighbours when necessary ‎35.What can we infer from the last paragraph?‎ A.The world is changing faster than ever.‎ B.People have stronger senses than before.‎ C.The world is more complex than it seems.‎ D.People in Darwin's time were more imaginative.‎ ‎【文章大意】 本文是一篇有关植被自我保护的研究报告。文章通过两位科学家的研究发现植被也有奇特的自我保护和协作抗击昆虫的方式。‎ ‎32. D 细节理解题。根据第一段第二句“…send out a particular smell…”可知是发出某种气味。‎ ‎33. A 词义猜测题。根据第三段最后一句“The attacker who was lunching now becomes lunch(进食者却成了别人的午餐).”可知进攻者变成了被进攻者。由此推断“the tables are turned”意思是“局面反转”。‎ ‎35. C 推理判断题。150年前的达尔文猜测(imagine)自然界比我们表面上认识的世界要喧闹得多,由该段倒数第二句“Our senses are weak.”得出结论:人们对自然界的认识感知是肤浅的。另外该文通过描述叶子自我保护的过程让我们更深入地了解了自然界的复杂性,世界其实比我们知道的要复杂得多。 ‎ ‎【名师点睛】‎ 本文33题属于词义猜测题。我们可以根据后句“Once they arrive,the tables are turned.The attacker who was lunching now becomes lunch”中的lunch,说明攻击者会成为午餐以及“When a leafy plant is under attack ,it doesn’t sit quietly植物受到攻击时,不会坐以待毙”,故the tables are turned意为“The plants get ready to fight back.植物准备反击”。本题属于典型的理由上下文语境推测。‎ 近几年高考阅读中猜测词义考查方法多样化,其中根据上下文语境推测将会越来越多。根据上下文语境作出判断:有时短文中出现一个需猜测其意义的词或短语,下面接着出现其定义域解释或例子,这就是判断该词或短语意义的主要依据。‎ 考点:考查记叙文阅读。‎ ‎ [2019·全国卷Ⅲ]‎ C After years of heated debate, grey wolves were reintroduced to Yellowstone National Park. Fourteen wolves were caught in Canada and transported to the park. By last year, the Yellowstone wolf population had grown to more than 170 wolves.‎ Grey wolves once were seen here and there in the Yellowstone area and much of the continental United States, but they were gradually displaced by human development. By the 1920s, wolves had practically disappeared from the Yellowstone area. They went farther north into the deep forests of Canada, where there were fewer humans around.‎ The disappearance of the wolves had many unexpected results. Deer and elk populations—major food sources (来源) for the wolf—grew rapidly. These animals consumed large amounts of vegetation (植被), which reduced plant diversity in the park. In the absence of wolves, coyote populations also grew quickly. The coyotes killed a large percentage of the park's red foxes, and completely drove away the park's beavers.‎ As early as 1966,biologists asked the government to consider reintroducing wolves to Yellowstone Park. They hoped that wolves would be able to control the elk and coyote problems. Many farmers opposed the plan because they feared that wolves would kill their farm animals or pets.‎ The government spent nearly 30 years coming up with a plan to reintroduce the wolves. The US Fish and Wildlife Service carefully monitors and manages the wolf packs in Yellowstone. Today,the debate continues over how well the grey wolf is fitting in at Yellowstone. Elk,deer,and coyote populations are down,while beavers and red foxes have made a comeback. The Yellowstone wolf project has been a valuable experiment to help biologists decide whether to reintroduce wolves to other parts of the country as well.‎ ‎28.What is the text mainly about?‎ A.Wildlife research in the United States.‎ B.Plant diversity in the Yellowstone area.‎ C.The conflict between farmers and grey wolves.‎ D.The reintroduction of wolves to Yellowstone Park.‎ ‎29.What does the underlined word “displaced” in Paragraph 2 mean?‎ A.Tested. B.Separated. ‎ C.Forced out. D.Tracked down.‎ ‎30.What did the disappearance of grey wolves bring about?‎ A.Damage to local ecology. ‎ B.A decline in the park's income.‎ C.Preservation of vegetation. ‎ D.An increase in the variety of animals.‎ ‎31.What is the author's attitude towards the Yellowstone wolf project?‎ A.Doubtful. B.Positive. ‎ C.Disapproving. D.Uncaring.‎ ‎【文章大意】 生物链中一个环节出了问题便可能会对生态造成灾难性的破坏。黄石公园的人类活动导致灰狼几近消失,幸亏当地政府采取措施引进灰狼,亡羊补牢,才使公园逐渐恢复了生态平衡。‎ ‎28. D 主旨大意题。纵观全文可以看出,文章介绍了由于人类的活动,著名的黄石 国家公园的灰狼几乎灭绝,这造成了严重的生态灾难,比如作为其食物的鹿等大量繁殖,破坏植被等。为改善这一状况,政府决定重新引进灰狼。果然,这些措施对于生态的恢复取得了很好的效果。由此可知答案。‎ ‎29. C 词义猜测题。由前文的but可知,该词的意思和上文的“…once were seen here and there…”意思相反,故选C项。下文的“…disappeared from the Yellowstone area.”也有提示。test测试;separate分开;force out迫使离开,把……赶走;track down追捕到,追查出。‎ ‎30. A 细节理解题。题干关键词为“the disappearance of grey wolves”,由此可定位到文章的第三段,第一句为主题句,告诉我们灰狼的消失带来了很多意想不到的后果,下文进行了详细的介绍:作为灰狼食物的动物迅速增加,导致植被遭到严重破坏等。这些都是对当地生态的破坏。由此可知答案。‎ ‎31. B 观点态度题。题干关键词为“the Yellowstone wolf project”,由此定位信息到文章的最后一句,由该句中的valuable可以看出,作者认为这个项目是一个有价值的实验,由此可以推断,作者对这一项目的态度是积极的。doubtful怀疑的;disapproving不赞成的;uncaring不关心的。‎ ‎【名师点睛】词义猜测题是高考英语阅读理解中常见的题型,一般考查对生词的猜测、熟词生义词的猜测、短语的猜测、代词的指代以及句子的理解。要求考生掌握构词法,还要学会利用上下文语境、利用定义、解释、举例和对比转折等线索来进行判断推测。而构词法也是一种猜词的办法,比如通过前缀:dis-, un-,im-等等,如本题中的displace。‎ ‎ [2019·江苏卷]‎ C A new commodity brings about a highly profitable,fastgrowing industry,urging antitrust (反垄断) regulators to step in to check those who control its flow. A century ago, the resource in question was oil. Now similar concerns are being raised by the giants (巨头) that deal in data, the oil of the digital age. The most valuable firms are Google,Amazon, Facebook and Microsoft. All look unstoppable.‎ Such situations have led to calls for the tech giants to be broken up. But size alone is not a crime. The giants' success has benefited consumers. Few want to live without search engines or a quick delivery. Far from charging consumers high prices, many of these services are free (users pay, in effect, by handing over yet more data). And the appearance of newborn giants suggests that newcomers can make waves, too.‎ But there is cause for concern. The Internet has made data abundant, allpresent and far more valuable, changing the nature of data and competition. Google initially used the data collected from users to target advertising better. But recently it has discovered that data can be turned into new services: translation and visual recognition, to be sold to other companies. Internet ‎ companies' control of data gives them enormous power. So they have a “God's eye view” of activities in their own markets and beyond.‎ This nature of data makes the antitrust measures of the past less useful. Breaking up firms like Google into five small ones would not stop remaking themselves: in time, one of them would become great again. A rethink is required—and as a new approach starts to become apparent, two ideas stand out.‎ The first is that antitrust authorities need to move from the industrial age into the 21st century. When considering a merger (兼并),for example, they have traditionally used size to determine when to step in. They now need to take into account the extent of firms' data assets (资产) when assessing the impact of deals. The purchase price could also be a signal that an established company is buying a newborn threat. When this takes place, especially when a newborn company has no revenue to speak of, the regulators should raise red flags.‎ The second principle is to loosen the control that providers of online services have over data and give more to those who supply them. Companies could be forced to reveal to consumers what information they hold and how much money they make from it. Governments could order the sharing of certain kinds of data, with users' consent.‎ Restarting antitrust for the information age will not be easy. But if governments don't want a data economy controlled by a few giants, they must act soon. ‎ ‎61.Why is there a call to break up giants?‎ A.They have controlled the data market.‎ B.They collect enormous private data.‎ C.They no longer provide free services.‎ D.They dismissed some newborn giants.‎ ‎62.What does the technological innovation in Paragraph 3 indicate?‎ A.Data giants' technology is very expensive.‎ B.Google's idea is popular among data firms.‎ C.Data can strengthen giants' controlling position.‎ D.Data can be turned into new services or products. ‎ ‎63.By paying attention to firms' data assets, antitrust regulators could ________.‎ A.kill a new threat B.avoid the size trap C.favour bigger firms D.charge higher prices ‎64.What is the purpose of loosening the giants' control of data?‎ A.Big companies could relieve data security pressure.‎ B.Governments could relieve their financial pressure.‎ C.Consumers could better protect their privacy.‎ D.Small companies could get more opportunities.‎ ‎【文章大意】 本文是一篇议论文。文章主要论述了数据信息垄断企业的发展导致的后果以及反垄断的措施、对反对数据垄断的未来展望。‎ ‎61. A 细节理解题。根据第一段第三句“Now similar concerns are being raised by the giants (巨头) that deal in data, the oil of the digital age.”和第二段第一句“Such situations have led to calls for the tech giants to be broken up.”可知,人们呼吁打破垄断的原因是数据信息垄断机构控制了数据市场,故选A项。注意B项中的private data,文中并没有提到私人数据,此题易误选B项。‎ ‎63. B 推理判断题。根据第五段第二、三句“When considering a merger (兼并),for example, they have traditionally used size to determine when to step in. They now need to take into account the extent of firms' data assets (资产) when assessing the impact of deals.”可知,传统意义上的反垄断组织用公司的大小来决定何时介入,而现在评估时则要考虑公司的数据资产规模。据此可知,关注数据评估,旨在避免陷入传统的“以公司大小”评估的陷阱,故选B项。‎ ‎64. D 推理判断题。根据第五段中的“The purchase price could also be a signal that an established company is buying a newborn threat. When this takes place, especially when a newborn company has no revenue to speak of, the regulators should raise red flags.”以及对第六段的理解可知,反垄断部门应该采取行动,使几大因特网巨头放松对数据的控制,使新生的小公司得到更多的发展机会,故选D项。‎ ‎【名师点睛】‎ 推理判断题 ‎ 高考英语阅读理解所涉及的考点比较多,比如考查常识理解、细节理解、语义理解、主旨理解、推理判断等。但比较而言,难度最大而且最容易出错的是推理判断题。因为推理判断题通常要求考生不仅要理解原文的文字信息,而且还要进行一定的判断和推理,并以此推断出文章的隐含意义。推理判断题属于主观性很强的高层次阅读理解题,做这类题目时,要严格依据作者所陈述的细节、事实以及作者的措辞、态度和语气,找出能够表露作者思想倾向和感情色彩的词语,然后利用自己已获得的相关知识进行推理判断,从而得出符合逻辑的结论。此时应特别注意:当问及作者的看法、意图与态度时,不要误认为是在问“你”的想法。‎ 考点一 考查因果关系的推断 因果关系的推断是高考英语推断题中考得比较多的一类。因果关系的推断有时只涉及一个或几个细节,有时涉及一个或几个段落,有时甚至涉及全文。一般说来,推断因果关系所涉及的内容越多,难度也就越大,同学们越要仔细。‎ 考点二 考查作者观点的推断 推断作者观点也是高考英语阅读中一个比较常见的考点。这类考题的命题特点是:命题者要求考生根据阅读材料中的某些事实推断出作者所持的观点和态度。同学们在做这类题时,一定要注意作者在文中的措辞,尤其是表达感情色彩的形容词、副词、动词及所举的例子(如有直接引语,也要特别注意),才能推断出作者的弦外之音。‎ 此类试题的干扰项通常具有以下特点:或是社会的一种普遍倾向,或是与本文无关的观点,或是与作者相反的看法等。‎ 考点三 考查作者意图的推断 这里说的“作者意图”至少包括两个方面:一是指作者直接在文中透露出的意图,如提及某个细节的意图,列举某个例子的意图,等等;二是指作者写这篇文章的意图或所想达到的目的。同学们做这类题时一定要结合文章主题进行推断,也就是说作者的意图原则上应与文章主题一致,而不能与之相悖。如:‎ 考点四 考查读者对象的推断 推断读者对象的考题在高考中出现得不多,只在部分省、市的考题中偶尔出现。这类考题的特点是:命题者要求先读一篇材料,然后要求考生根据所读材料推断出该材料的读者对象是什么。同学们做这类题要注意的是,先读懂文章大意,然后比较所给四个选项的人群特点,再寻找其相关点,最后得出一个合情合理的答案。例如第64题。‎ 考点五 考查隐含意义的推断 对隐含意义的推测是阅读理解中难度较大的一类考点。所谓推断隐含意义,就是要求考生在理解原文表面文字信息的基础上,作出合乎情理的推理与判断,从而得出文章的隐含意义和深层意义。对隐含意义的推测属于主观性较强的高层次阅读理解,同学们在做这类题目时,一定要严格依据作者所陈述的细节、事实以及作者的措辞、态度和语气,找出能够表露作者思想倾向和感情色彩的词句,然后利用自己的相关知识进行推理判断,从而得出符合逻辑的结论。例如第72题。‎ 考点六 考查特定结论的推断 简单地说,高考英语对阅读理解的考查主要是两大类:一类是细节理解;一类是推理判断。推理判断题除前面分析的因果关系的推断、作者观点的推断、作者意图的推断、读者对象的推断、文章出处的推断、隐含意义的推断等之外,还有许多结论性的推断,即要求考生根据所读文章对某种结论作出推断。因这类考题涉及面比较广,出题形式比较灵活,所以一并归入“特定结论的推断”。‎ ‎ [2019·全国卷Ⅲ]‎ B Minutes after the last movie ended yesterday at the Plaza Theatre, employees were busy sweeping up popcorns and gathering coke cups. It was a scene that had been repeated many times in the theatre's 75year history. This time, however, the cleanup was a little different. As one group of workers carried out the rubbish, another group began removing seats and other theatre equipment in preparation for the building's end.‎ The film classic The Last Picture Show was the last movie shown in the old theatre. Though the movie is 30 years old, most of the 250 seats were filled with tearyeyed audience wanting to say goodbye to the old building. Theatre owner Ed Bradford said he chose the movie because it seemed appropriate. The movie is set in a small town where the only movie theatre is preparing to close down.‎ Bradford said that large modern theatres in the city made it impossible for the Plaza to compete. He added that the theatre's location(位置) was also a reason. “This used to be the centre of town,” he said. “Now the area is mostly office buildings and warehouses.”‎ Last week some city officials suggested the city might be interested in turning the old theatre into a museum and public meeting place. However, these plans were abandoned because of financial problems. Bradford sold the building and land to a local development firm, which plans to build a shopping complex on the land where the theatre is located.‎ The theatre audience said goodbye as Bradford locked the doors for the last time. After 75 years the Plaza Theatre has shown its last movie. The theatre will be missed.‎ ‎24.In what way was yesterday's cleanup at the Plaza special?‎ A.It made room for new equipment.‎ B.It signalled the closedown of the theatre.‎ C.It was done with the help of the audience.‎ D.It marked the 75th anniversary of the theatre.‎ ‎25.Why was The Last Picture Show put on?‎ A.It was an alltime classic. ‎ B.It was about the history of the town.‎ C.The audience requested it. ‎ D.The theatre owner found it suitable.‎ ‎26.What will probably happen to the building?‎ A.It will be repaired. ‎ B.It will be turned into a museum.‎ C.It will be knocked down. ‎ D.It will be sold to the city government. ‎ ‎27.What can we infer about the audience?‎ A.They are disappointed with Bradford. ‎ B.They are sad to part with the old theatre. ‎ C.They are supportive of the city officials. ‎ D.They are eager to have a shopping centre. ‎ ‎【文章大意】 Plaza Theatre,一座带着城市记忆的剧院,由于种种原因,要面临关门歇业。临关门前,老板选了一部合适的老电影播放。伤心的观众眼含不舍的泪水。‎ ‎24. B 细节理解题。由第一段最后一句“…another group began removing seats and other theatre equipment in preparation for the building's end.”可知,打扫完卫生后,这家剧院就要关门了。故选B项。‎ ‎27. B 推理判断题。由第二段的“…most of the 250 seats were filled with tearyeyed audience wanting to say goodbye to the old building.”可以看出,来看最后一场电影的观众是很伤心的。tearyeyed 意为“泪眼汪汪的”。‎ ‎ [2019·天津卷]‎ A Suppose you're in a rush, feeling tired, not paying attention to your screen, and you send an email that could get you in trouble.‎ Realisation will probably set in seconds after you've clicked “send”. You freeze in horror and burn with shame.‎ What to do? Here are four common email accidents, and how to recover.‎ Clicking “send” too soon Don't waste your time trying to find out if the receiver has read it yet. Write another email as swiftly as you can and send it with a brief title explaining that this is the correct version and the previous version should be ignored.‎ Writing the wrong name The sooner you notice, the better. Respond quickly and briefly, apologising for your mistake. Keep the tone measured: don't handle it too lightly, as people can be offended, especially if your error suggests a misunderstanding of their culture(i.e. incorrect ordering of Chinese names).‎ Clicking “reply all” unintentionally You accidentally reveal(透露)to the entire company what menu choices you would prefer at the staff Christmas dinner, or what holiday you'd like to take. In this instance, the best solution is to send a quick, lighthearted apology to explain your awkwardness. But it can quickly rise to something worse, when everyone starts hitting “reply all” to join in a long and unpleasant conversation. In this instance, step away from your keyboard to allow everyone to calm down.‎ Sending an offensive message to its subject The most awkward email mistake is usually committed in anger. You write an unkind message about someone, intending to send it to a friend, but accidentally send it to the person you're discussing. In that case, ask to speak in person as soon as possible and say sorry. Explain your frustrations calmly and sensibly—see it as an opportunity to clear up any difficulties you may have with this person.‎ ‎36.After realising an email accident, you are likely to feel ________.‎ A.curious B.tired ‎ C.awful D.funny ‎ ‎37.If you have written the wrong name in an email, it is best to ________.‎ A.apologise in a serious manner B.tell the receiver to ignore the error C.learn to write the name correctly D.send a short notice to everyone ‎38.What should you do when an unpleasant conversation is started by your “reply all” email?‎ A.Try offering other choices.‎ B.Avoid further involvement.‎ C.Meet other staff members.‎ D.Make a lighthearted apology.‎ ‎39.How should you deal with the problem caused by an offensive email?‎ A.By promising not to offend the receiver again.‎ B.By seeking support from the receiver's friends.‎ C.By asking the receiver to control his anger.‎ D.By talking to the receiver facetoface.‎ ‎40.What is the passage mainly about?‎ A.Defining email errors.‎ B.Reducing email mistakes.‎ C.Handling email accidents.‎ D.Improving email writing.‎ ‎【文章大意】 本文是一篇说明文,主要介绍了四种常见的误发邮件的情况以及弥补办法。‎ ‎36. C 细节理解题。根据第二段中的“You freeze in horror and burn with shame.”可知当你发邮件出现失误时,你会感到害怕。‎ ‎37. A 细节理解题。根据Writing the wrong name下的“Respond quickly and briefly, apologising for your mistake.”以及“…don't handle it too lightly…”可知要用严肃的方式向对方道歉。‎ ‎38. B 细节理解题。根据Clicking “reply all” unintentionally下的最后一句“…step away from your keyboard to allow everyone to calm down.”可知要尽快避免进一步参与。‎ ‎39. D 细节理解题。根据Sending an offensive message to its subject下的第三句“In that case, ask to speak in person as soon as possible and say sorry.”可知要与收到邮件的人面对面地交流。‎ ‎40. C 主旨大意题。根据第三段中“Here are four common email accidents, and how to recover.”可知本文讲的是如何处理错发邮件的情况。‎ ‎【名师点睛】这是一篇说明文,题目比较简单的,考查细节题为主。可以使用信息筛选法做题。为了节省时间,可以用先看题目,带着题目在文中寻读的方法找到主题句,理解并找出正确答案。如36题,题目问:在你意识到你的邮件问题时你可能会感到_____,而文章中第二段就是题干信息,后面就是答案,答案是很明显的。‎ ‎ [2019·天津卷]‎ C This month, Germany's transport minister, Alexander Dobrindt, proposed the first set of rules for autonomous vehicles(自主驾驶车辆). They would define the driver's role in such cars and govern how such cars perform in crashes where lives might be lost. ‎ The proposal attempts to deal with what some call the “death valley” of autonomous vehicles: the grey area between semiautonomous and fully driverless cars that could delay the driverless future.‎ Dobrindt wants three things: that a car always chooses property(财产) damage over personal injury; that it never distinguishes between humans based on age or race; and that if a human removes his or her hands from the driving wheel—to check email, say—the car's maker is responsible if there is a crash.‎ ‎“The change to the road traffic law will permit fully automatic driving,” says Dobrindt. It will put fully driverless cars on an equal legal footing to human drivers, he says.‎ Who is responsible for the operation of such vehicles is not clear among car makers, consumers and lawyers. “The liability(法律责任) issue is the biggest one of them all,” says Natasha Merat at the University of Leeds, UK.‎ An assumption behind UK insurance for driverless cars, introduced earlier this year, insists that a human “be watchful and monitoring the road” at every moment.‎ But that is not what many people have in mind when thinking of driverless cars. “When you say ‘driverless cars’, people expect driverless cars,”Merat says. “You know—no driver.”‎ Because of the confusion, Merat thinks some car makers will wait until vehicles can be fully automated without human operation.‎ Driverless cars may end up being a form of public transport rather than vehicles you own, says Ryan Calo at Stanford University, California. That is happening in the UK and Singapore, where governmentprovided driverless vehicles are being launched.‎ That would go down poorly in the US, however. “The idea that the government would take over driverless cars and treat them as a public good would get absolutely nowhere here,” says Calo.‎ ‎46.What does the phrase “death valley” in Paragraph 2 refer to?‎ A.A place where cars often break down.‎ B.A case where passing a law is impossible.‎ C.An area where no driving is permitted. ‎ D.A situation where drivers' role is not clear.‎ ‎47.The proposal put forward by Dobrindt aims to ________.‎ A.stop people from breaking traffic rules B.help promote fully automatic driving C.protect drivers of all ages and races D.prevent serious property damage ‎48.What do consumers think of the operation of driverless cars?‎ A.It should get the attention of insurance companies.‎ B.It should be the main concern of law makers.‎ C.It should not cause deadly traffic accidents.‎ D.It should involve no human responsibility.‎ ‎49.Driverless vehicles in public transport see no bright future in ________.‎ A.Singapore B.the UK C.the US D.Germany ‎50.What could be the best title for the passage?‎ A.Autonomous driving: Whose liability?‎ B.Fully automatic cars: a new breakthrough C.Autonomous vehicles: Driver removed!‎ D.Driverless cars: root of road accidents ‎【文章大意】 本文是一篇说明文。文章主要说的是德国的交通部长提议对自主驾驶车辆制定一套规则,以确定在这样的车辆中司机所起的作用、这样的车辆在事故中所承担的责任以及它的前景。‎ ‎46. D 词义猜测题。根据第一段中的“要明确司机的责任”以及第二段该短语后的“…the grey area between semiautonomous and fully driverless cars that could delay the driverless future.”可知dead valley在此处的意思是“司机责任不明确的一种情况”。‎ ‎47. B 推理判断题。根据第四段中的“‘The change to the road traffic law will permit fully ‎ automatic driving…’”以及“It will put fully driverless cars on an equal legal footing to human drivers…”可知交通部长提出这个提议的目的是推销自动驾驶车辆。‎ ‎48. D 细节理解题。根据第七段中的最后一句“‘You know—no driver.’”可知人们对自主驾驶车辆的认识就是它不涉及人的责任。‎ ‎【名师点睛】这篇文章内容和每个人生活密切相关,讲的是谁对这种新型交通工具:无人驾驶的全自动化的车辆负责各国意见不一,学生应该有兴趣阅读,涉及到科技前沿,文章篇幅适中。考查比较全面,除了理解细节题,还需要适当的推理,还有词义辨析类的题目,对学生能力考查比较全面。‎ 做词义辨析类的题目,要仔细阅读这个词所在的前后的句子,发现有没有关联的同义词,反义词或是这个词的定义,分析这篇阅读的46题,要求对含有这个短语的整个句子的正确理解以及标点符号的作用。‎ ‎ [2019·天津卷]‎ D I read somewhere that we spend a full third of our lives waiting. But where are we doing all of this waiting, and what does it mean to an impatient society like ours? To understand the issue, let's take a look at three types of “waits”.‎ The very purest form of waiting is the WatchedPot Wait. It is without doubt the most annoying of all. Take filling up the kitchen sink(洗碗池) as an example. There is absolutely nothing you can do while this is going on but keep both eyes fixed on the sink until it's full. During these waits, the brain slips away from the body and wanders about until the water runs over the edge of the counter and onto your socks. This kind of wait makes the waiter helpless and mindless. ‎ A cousin to the WatchedPot Wait is the Forced Wait. This one requires a bit of discipline. Properly preparing packaged noodle soup requires a Forced Wait. Directions are very specific. “Bring three cups of water to boil, add mix, simmer three minutes, remove from heat, let stand five minutes.” I have my doubts that anyone has actually followed the procedures strictly. After all, Forced Waiting requires patience. ‎ Perhaps the most powerful type of waiting is the LuckyBreak Wait. This type of wait is unusual in that it is for the most part voluntary. Unlike the Forced Wait, which is also voluntary, waiting for your lucky break does not necessarily mean that it will happen. ‎ Turning one's life into a waiting game requires faith and hope, and is strictly for the optimists among us. On the surface it seems as ridiculous as following the directions on soup mixes, but the LuckyBreak Wait well serves those who are willing to do it. As long as one doesn't ‎ come to rely on it, wishing for a few good things to happen never hurts anybody. ‎ We certainly do spend a good deal of our time waiting. The next time you're standing at the sink waiting for it to fill while cooking noodle soup that you'll have to eat until a large bag of cash falls out of the sky, don't be desperate. You're probably just as busy as the next guy.‎ ‎51.While doing a WatchedPot Wait, we tend to ________.‎ A.keep ourselves busy B.get absentminded C.grow anxious D.stay focused ‎52.What is the difference between the Forced Wait and the WatchedPot Wait?‎ A.The Forced Wait requires some selfcontrol.‎ B.The Forced Wait makes people passive.‎ C.The WatchedPot Wait needs directions.‎ D.The WatchedPot Wait engages body and brain. ‎ ‎53.What can we learn about the LuckyBreak Wait?‎ A.It is less voluntary than the Forced Wait.‎ B.It doesn't always bring the desired result. ‎ C.It is more fruitful than the Forced Wait.‎ D.It doesn't give people faith and hope.‎ ‎54.What does the author advise us to do the next time we are waiting?‎ A.Take it seriously.‎ B.Don't rely on others.‎ C.Do something else.‎ D.Don't lose heart.‎ ‎55.The author supports his view by ________.‎ A.exploring various causes of “waits”‎ B.describing detailed processes of “waits”‎ C.analysing different categories of “waits”‎ D.revealing frustrating consequences of “waits”‎ ‎【文章大意】 本文论述了我们人生中经常遇到的三种等待。‎ ‎51. B 细节理解题。根据第二段中的第五句“During these waits, the brain slips away from the body and wanders about until the water runs over the edge of the counter and onto your socks.”可知我们在进行WatchedPot Wait时很容易走神,心不在焉。‎ ‎52. A 推理判断题。根据第三段中的第二句“This one requires a bit of discipline.”可知这种等待需要一点纪律,即需要自控。‎ ‎53. B 推理判断题。根据第四段中的最后一句“…waiting for your lucky break does not necessarily mean that it will happen.”可知等待你的好运气未必就意味着它能发生,因此LuckyBreak Wait并不总是能带来预期的结果。‎ ‎54. D 推理判断题。根据最后一段中的倒数第二句“…don't be desperate.”可知作者建议我们下次等待时,不要失去信心。‎ ‎55. C 篇章结构题。文章的结构是总—分。文章第一段提出有三种类型的等待,然后下面几段内容一直在分析这三种等待,因此作者是通过分析不同的类型的等待来支持他自己的观点的。‎ ‎【名师点睛】这篇文章内容是非常贴近生活的,篇幅适中较长,要求学生快速阅读材料,在掌握文章大意的前提下答题。考查题型以理解细节题为主,还要学生结合文章作出推理判断,这些不是某个句子可以体现的,要对文章有整体把握。 ‎ 这篇文章最后一题就是推理判断题。学生对于55题推理判断题能很好的把握,根据文章中第二三四段的首句即主题句就可以正确理解不难得出答案。‎ ‎2019年高考题 ‎1.【2019·全国新课标I】B Grandparents Answer a Call ‎ As a third generation native of Brownsville, Texas, Mildred Garza never pleased move away,. Even when her daughter and son asked her to move to San Antonio to help their children, she politely refused . Only after a year of friendly discussion did Ms Gaf finally say yes. That was four years ago. Today all three generations regard the move to a success, giving them a closer relationship than they would have had in separate cities.‎ ‎ No statistics show the number of grandparents like Garza who are moving closer to the children and grandchildren. Yet there is evidence suggesting that the trend is growing. Even President Obama’s mother-in-law, Marian Robinson, has agreed to leave Chicago and into the White House to help care for her granddaughters. According to a study grandparents com. 83 percent of the people said Mrs. Robinson ‘s decision will influence the grandparents in the American family.‎ ‎ Two-thirds believe more families will follow the example of Obama’s family.‎ ‎ “in the 1960s we were all a little wild and couldn’t get away from home far enough fast enough to prove we could do it on our own,” says Christine Crosby, publisher of grate magazine for grandparents .We now realize how important family is and how important”” to be near them, especially when you’re raining children.”‎ ‎ Moving is not for everyone. Almost every grandparent wants to be with his or her grandchildren and is willing to make sacrifices, but sometimes it is wiser to say no and visit frequently instead. Having your grandchildren far away is hard, especially knowing your adult child is struggling, but giving up the life you know may be harder.‎ ‎25. Why was Garza’s move a success?‎ A.It strengthened her family ties. ‎ B.It improved her living conditions.‎ C.It enabled her make more friends.‎ D.It helped her know more new places.‎ ‎26.What was the reaction of the public to Mrs. Robinson’s decision?‎ A.17% expressed their support for it.‎ B.Few people responded sympathetically.‎ C.83% believed it had a bad influence.‎ D.The majority thought it was a trend.‎ ‎27. What did Crosby say about people in the 1960s?‎ A.They were unsure of raise more children.‎ B.They were eager to raise more children.‎ C.They wanted to live away from their parents.‎ D.They bad little respect for their grandparent.‎ ‎28. What does the author suggest the grandparents do in the lasr paragraph?‎ ‎ A. Make decisions in the best interests' of their own ‎ B. Ask their children to pay more visits to them ‎ C. Sacrifice for their struggling children ‎ ‎ D. Get to know themselves better ‎【答案】‎ ‎25-28. A D C A ‎【解析】‎ 试题分析:本文是一篇议论文。主要讲述目前社会上很多老年人愿意搬到离子女近的地方居住的一种趋势。同时作者也建议老人们要做出适合自己的选择,不要仅仅为孩子考虑而牺牲了自己习惯了的生活方式。‎ ‎25. A细节理解题。根据首段末句可知,Mildred Garza老人搬到离孩子近的地方居住让他们的关系更密切,一家人认为这是一个成功的事情。故选A 。‎ 考点:社会生活类短文阅读 ‎【技巧点拨】推理判断题属于主观题,是层次较高的题目。此类题要求通过文章中的文字信息,上下文逻辑关系及事物的发展变化等已知的信息,推断出作者暗含的意思。解题关键要注意表明作者观点、立场的动词、名词、形容词和副词等。28题中则根据it is wiser to say no推断出作者的立场。‎ ‎2.【2019·全国新课标I】D The meaning of silence varies among cultural groups. Silences may be thoughtful, or they may be empty when a person has nothing to say. A silence in a conversation may also show stubbornness, or worry. Silence may be viewed by some cultural groups as extremely uncomfortable; therefore attempts may be made to fill every gap(间隙)with conversation. Persons in other cultural groups value silence and view it as necessary for understanding a ‎ person's needs. Many Native Americans value silence and feel it is a basic part of communicating among people, just as some ‎ traditional Chinese and Thai persons do. Therefore, when a person from one of these cultures is speaking and ‎ suddenly stops, what maybe implied(暗示) is that the person wants the listener to consider what has been said before continuing. In these cultures, silence is a call for reflection.‎ Other cultures may use silence in other ways, particularly when dealing with conflicts among people or in relationships of people with different amounts of power. For example, Russian, French, and Spanish persons may use silence to show agreement between parties about the topic under discussion. However, Mexicans may use silence when instructions are given by a person in authority rather than be rude to that person by arguing with him or her. In still another use, persons in ‎ Asian cultures may view silence as a sign of respect, particularly to an elder or a person in authority.‎ Nurses and other care-givers need to be aware of the possible meanings of silence when they come across the personal anxiety their patients may be experiencing. Nurses should recognize their own personal and cultural construction of silence so that a patient’s silence is not interrupted too early or allowed to go on unnecessarily. A nurse who understands the healing(治愈) value of silence can use this understanding to assist in the care of patients from their own and from other cultures.‎ 32. What does the author say about silence in conversations?‎ A. It implies anger.‎ B. It promotes friendship.‎ C. It is culture-specific.‎ D. It is content-based.‎ 33. Which of the following people might regard silence as a call for careful thought?‎ A. The Chinese.‎ B. The French.‎ C. The Mexicans.‎ D. The Russians.‎ 34. What does the author advise nurses to do about silence?‎ A. Let it continue as the patient pleases.‎ B. Break it while treating patients.‎ C. Evaluate its harm to patients.‎ D. Make use of its healing effects.‎ 35. What may be the best title for the text?‎ A. Sound and Silence B. What It Means to Be Silent C. Silence to Native Americans D. Speech Is Silver; Silence Is Gold ‎【答案】‎ ‎32-35.C A D B ‎【解析】‎ 试题分析:本文是一篇说明文, 主要介绍了沉默在不同文化背景下的不同内涵。在有些时候,人们利用沉默来解决人们之间的冲突,而在有些时候沉默则被认为表示顽固或者是担忧。‎ ‎32.C细节理解题。根据文章首段首句The meaning of silence varies among cultural groups 可知,作者认为沉默是有文化特性的。‎ ‎33.A 细节理解题。根据第二段中 what may be implied is that the person wants the listener to consider what has been said before continuing可知中国人认为谈话时人们的沉默是在思考谈话内容。‎ ‎34.D细节理解题。根据末段末句A nurse who understands the healing value of silence can use this understanding to assist in the care of patients 可知,作者建议护士要利用沉默的好处来护理病人。故选D 。‎ ‎35.B主旨要义题。文章主要介绍了在不同文化背景下谈话期间的沉默的不同含义,由此判断B选项可以概括文章内容。故选B 。‎ 考点:社会生活类短文阅读 ‎3.【2019·全国新课标II】D A new collection of photos brings an unsuccessful Antarctic voyage back to life.‎ Frank Hurley’s pictures would be outstanding----undoubtedly first-rate photo-journalism---if they had been made last week. In fact, they were shot from 1914 through 1916, most of them after a disastrous shipwreck(海滩), by a cameraman who had no reasonable expectation of survival. Many of the images were stored in an ice chest, under freezing water, in the damaged wooden ship.‎ ‎ The ship was the Endurance, a small, tight, Norwegian-built three-master that was intended to take Sir Ernest Shackleton and a small crew of seamen and scientists, 27 men in all, to the southernmost shore of Antarctica’s Weddell Sea. From that point Shackleton wanted to force a passage by dog sled(雪橇) across the continent. The journey was intended to achieve more than what Captain Robert Falcon Scott had done. Captain Scott had reached the South Pole early in 1912 but had died with his four companions on the march back.‎ ‎ As writer Caroline Alexander makes clear in her forceful and well-researched story The Endurance, adventuring was even then a thoroughly commercial effort. Scott’s last journey, completed as be lay in a tent dying of cold and hunger, caught the world’s imagination, and a film made in his honor drew crowds. Shackleton, a onetime British merchant-navy officer who had got to within 100 miles of the South Pole in 1908, started a business before his 1914 voyage to make money from ‎ movie and still photography. Frank Hurley, a confident and gifted Australian photographer who knew the Antarctic, was hired to make the images, most of which have never before been published.‎ ‎13. What do we know about the photos taken by Hurley?‎ ‎ A. They were made last week ‎ B. They showed undersea sceneries ‎ C. They were found by a cameraman ‎ D. They recorded a disastrous adventure ‎14. Who reached the South Pole first according to the text?‎ ‎ A. Frank Hurley B. Ernest Shackleton ‎ C. Robert Falcon Scott D. Caroline Alexander ‎15. What does Alexander think was the purpose of the 1914 voyage?‎ ‎ A. Artistic creation B. Scientific research ‎ C. Money making D. Treasure hunting ‎【答案】13. D 14. C 15. C ‎ ‎【解析】‎ 试题分析:本文属于记叙文。讲述了Frank的图片记录了一次不成功的航海活动,文章介绍了与这次航海活动相关的具体内容。‎ ‎33.D 细节理解题。根据文章第二段第二句“they were shot from 1914 through 1916, most of them after a disastrous shipwreck....”可知这些照片记录的是1914年左右发生的一次海难事故。故D项正确。‎ ‎【名师点睛】‎ ‎ 本文三题均考查了细节题。在完成细节题时,要特别注意排除干扰项。如【小题13】中的A项“They were made last week”属于“颠倒黑白”类干扰项,根据第二段第一句“Frank Hurley’s pictures......undoubtedly first-rate photo-journalism---if they had been made last week.”可知“如果这些照片是上周所拍,那么就会是一流的。”实际上这些照片是1914年左右拍摄的。说明A项明显错误。‎ 干扰项的设置方法除了“颠倒黑白”之外,通常还有以下几种方式: ‎ ‎1. 张冠李戴。命题者把文章作者的观点与他人的观点混淆起来,题干问的是作者的观点,选项中出现的却是他人的观点;或者题干问的是他人的观点,却把作者的观点放到选项中去。 ‎ ‎2. 偷梁换柱。干扰项用了与原文相似的句型结构和大部分相似的词汇,却在不易引人注意的地方换了几个词汇,造成句意的改变。 ‎ ‎3. ‎ 无中生有。干扰项往往是生活的基本常识和普遍接受的观点,但在原文中并无相关的信息支持点,这种选项的设置往往与问题的设问毫不相干。 ‎ ‎4. 以偏概全。考生在做猜测文章中心思想、给文章添加标题或判断推理题时,往往会犯以偏概全的错误。产生这类错误的原因是考生受思维定势的影响或考虑不周,以局部代替整体。其具体表现为合理关联与不合理关联、准确概括与不准确概括之间的错位。不合理关联就是表层理解与深层理解相混淆。表层理解是对文章中客观事实的感知和记忆,往往是文章直接表述的结论;深层理解则是对文章中的客观事实进行逻辑推理、总结或概括后得出的结论。不准确概括是指不能准确地按题目要求概括或提取文中的表层或深层信息。‎ 考点:考查记叙文阅读 ‎4.【2019·全国新课标III】B ‎ On one of her trips to New York several years ago, Eudora Welty decided to take a couple of New York friends out to dinner. They settled in at a comfortable East Side cafe and within minutes, another customer was approaching their table.‎ ‎ “Hey, aren’t you from Mississippi?” the elegant, white-haired writer remembered being asked by the stranger. “I’m from Mississippi too.”‎ ‎ Without a second thought, the woman joined the Welty party. When her dinner partner showed up, she also pulled up a chair.‎ ‎ “They began telling me all the news of Mississippi,” Welty said. “I didn’t know what my New York friends were thinking.”‎ ‎ Taxis on a rainy New York night are rarer than sunshine. By the time the group got up to leave, it was pouring outside. Welty’s new friends immediately sent a waiter to find a cab. Heading back downtown toward her hotel, her big-city friends were amazed at the turn of events that had changed their Big Apple dinner into a Mississippi.‎ ‎ “My friends said: ‘Now we believe your stories,’” Welty added. “And I said: ‘Now you know. These are the people that make me write them.’”‎ ‎ Sitting on a sofa in her room, Welty, a slim figure in a simple gray dress, looked pleased with this explanation.‎ ‎ “I don’t make them up,” she said of the characters in her fiction these last 50 or so years. “I don’t have to.”‎ ‎ Beauticians, bartenders, piano players and people with purple hats, Welty’s people come from afternoons spent visiting with old friends, from walks through the streets of her native Jackson,‎ ‎ Miss., from conversations overheard on a bus. It annoys Welty that, at 78, her left ear has now given out. Sometimes, sitting on a bus or a train, she hears only a fragment(片段) of a particularly interesting story.‎ ‎5.What happened when Welty was with her friends at the cafe?‎ A. Two strangers joined her.‎ B. Her childhood friends came in.‎ ‎ C. A heavy rain ruined the dinner.‎ D. Some people held a party there.‎ ‎6. The underlined word “them” in Paragraph 6 refers to Welty’s .‎ A. readers B. parties C. friends D. stories ‎7. What can we learn about the characters in Welty’s fiction?‎ ‎ A. They live in big cities.‎ ‎ B. They are mostly women.‎ ‎ C. They come from real life.‎ ‎ D. They are pleasure seekers.‎ ‎【答案】‎ ‎5. A ‎6. D ‎7. C ‎【解析】‎ 试题分析:文章介绍了一位女作家请纽约的朋友吃饭时发生的故事。Welty是一位年纪比较大的作家,她来自密西西比。Welty的作品都是来自于现实的生活。‎ ‎5. A细节理解题。根据第一段“another customer was approaching their table”和第三段“the woman joined the Welty party. When her dinner partner showed up, she also pulled up a chair”可知,先后有两个陌生人(一位女士及其同伴)加入了Welty他们的聚会,故选A。‎ ‎6. D猜测词义题。划线的them指代前面提到的人或物,根据“Now we believe your stories”可知,them指代的是Welty写的小说里面的故事,听了Welty和两个陌生人的有关密西西比的谈话之后,Welty的朋友相信了Welty小说里的故事都是来源于生活,故选D。‎ ‎7. C推理判断题。根据“I don’t make them up”和“Welty’s people come from afternoons spent visiting with old friends, from walks through the streets of her native Jackson, Miss., from ‎ conversations overheard on a bus.”可知,Welty小说里的人物并非虚构的,他们都来源于现实的生活,故选C。‎ 考点:故事类短文阅读 ‎【名师点睛】‎ 猜词技巧:阅读理解的测试中经常有猜测词、短语、习语意义的题目,这些词、短语、习语要么是生词,要么是熟词新义,单靠平时积累是不够的,还要掌握一定的做题技巧。总结如下: (1)根据构词法(转化、合成、派生)进行判断。 (2) 根据文中的定义、解释猜生词 ;利用事例或解释猜生词;利用重复解释的信息猜生词。 (3)根据上下文的指代关系进行选择:文章中的代词it,that,he,him或them可以指上文提到的人或物,其中it和that还可以指一件事。第6题就是属于对指代关系进行判断。 (4)根据转折或对比关系进行判断:根据上下句的连接词,如but,however,otherwise等就可以看到前后句在意义上的差别,从而依据某一句的含义,来确定另一句的含义。 (5)根据因果关系进行判断:俗话说,“有因必有果,有果必有因”。根据原因可以预测结果,根据结果也可以找出原因。 (6)根据同位关系进行判断:阅读中有时出现新词、难词,后面就跟着一个同位语,对前面的词进行解释,有时这种解释也用连词“or”连接。 (7)利用标点符号和提示词猜测词义。还可以表示转折、对比或不相干的意义。破折号表示解释说明。 常见问题形式有: (1)The word “…” in Line … means/can be best replaced by … (2)As used in the passage, the phrase “…” suggests… (3)From the passage, we can infer that the word/phrase /the sentence “…” is/refers to /means… (4)The word “…” is closest in meaning to …‎ ‎5.【2019·全国新课标III】C ‎ If you are a fruit grower—or would like to become one—take advantage of Apple Day to see what’s around.‎ ‎ It’s called Apple Day but in practice it’s more like Apple Month. The day itself is on October 21, but since it has ‎ caught on, events now spread out over most of October around Britain.‎ ‎ Visiting an apple event is a good chance to see, and often taste, a wide variety of apples.‎ ‎ To people who are used to the limited choice of apples such as Golden Delicious and Royal Gala in supermarkets, it can be quite an eye opener to see the range of classical apples still in existence, such as Decio which was grown by the Romans. Although it doesn’t taste of anything special, it’s still worth a try, as is the knobbly(多疙瘩的) Cat’s Head which is more of a curiosity than anything else.‎ ‎ There are also varieties developed to suit specific local conditions. One of the very best varieties for eating quality is Orleans Reinette, but you’ll need a warm, sheltered place with perfect soil to grow it, so it’s a pipe dream for most apple lovers who fall for it.‎ ‎ At the events, you can meet expert growers and discuss which ones will best suit your conditions, and because these are family affairs, children are well catered for with apple-themed fun and games.‎ ‎ Apple Days are being held at all sorts of places with an interest in fruit, including stately gardens and commercial orchards(果园). If you want to have a real orchard experience, try visiting the National Fruit Collection at Brogdale, near Faversham in Kent.‎ ‎8. What can people do at the apple events?‎ ‎ A. Attend experts’ lectures. B. Visit fruit-loving families.‎ ‎ C. Plant fruit trees in an orchard. D. Taste many kinds of apples.‎ ‎9. What can we learn about Decio?‎ A. It is a new variety. B. It has a strange look.‎ ‎ C. It is rarely seen now. D. It has a special taste.‎ ‎10. What does the underlined phrase “a pipe dream” in Paragraph 3mean?‎ ‎ A. A practical idea. B. A vain hope.‎ C.A brilliant plan. D. A selfish desire.‎ ‎11. What is the author’s purpose in writing the text?‎ A. To show how to grow apples.‎ B .To introduce an apple festival.‎ C. To help people select apples.‎ D. To promote apple research.‎ ‎【答案】‎ ‎8. D ‎9. C ‎10. B ‎11. B ‎【解析】‎ 试题分析:作者向人们介绍了一个节日--Apple Day。由于这个节日翡翠受欢迎,现在已经演变成“苹果月”了。在英国,人们在十月份庆祝该节日,持续大约一个月的时间。‎ ‎11. B写作意图题。根据第一段“if you are a fruit grower—or would like to become one—take advantage of Apple Day to see what’s around. ”和最后一段“If you want to have a real orchard experience, try visiting the National Fruit Collection at Brogdale, near Faversham in Kent.”可知,作者向读者介绍了一个节日—Apple Day,并推荐人们参加这个节日,故选B。‎ 考点:生活故事类短文阅读 ‎【名师点睛】‎ 故事类文章是高考阅读理解常选材料之一。这类文章常通过叙述突出人物性格特征或讲述个人经历与感悟,以及逸闻趣事。文章的要素有时间、地点、人物、事件等。有些是按事件发展的经过为主线叙述的,在叙述的过程中有详有略;有些是按时间的顺序叙述的,有顺叙、倒叙等。 解题技巧: 1、高考选用的此类文章常常为幽默类或情感类故事,其目的或阐明某种观点,或抨击某种陋习,或赞扬某种品德,所以要关注作者表达的个人感悟,提出的个人意见,或故事中出其不意的变化,而这些往往体现文章中心或写作意图,属于必考点。例如第11题,考查写作意图。作者向读者介绍了一个节日—Apple Day,并推荐人们参加有关的活动。 2、该类文章语言流畅、有趣,命题往往从故事的情节、人物或事件之间的关系、作者的意图和态度、故事的前因和后果等方面入手,考查学生对细节的辨认能力以及推理判断能力。这类文章同学们常有文章易懂,题目不易做的感觉。要注意培养自己快速阅读的习惯。切不可在个别难懂的词句上停留太久,耽误时间。 3、对整篇短文内容有了一定的了解后,要马上看短文后的问题,带着问题去选择或判断答案。要确定所需查找的信息范围,并注意所查找信息的特点。例如:如果问题或选项涉及到人名、地名,就应该找首字母大写的单词;如果问题或选项涉及时间、日期、数字,就应该寻找具体的数据。‎ ‎6.【2019·全国新课标III】D ‎ Bad news sells. If it bleeds, it leads. No news is good news, and good news is no news. Those are the classic rules for the evening broadcasts and the morning papers. But now that information is being spread and monitored(监控) in different ways, researchers are discovering new rules. By ‎ tracking people’s e-mails and online posts, scientists have found that good news can spread faster and farther than disasters and sob stories.‎ ‎ “The ‘if it bleeds’ rule works for mass media,” says Jonah Berger, a scholar at the University of Pennsylvania. “They want your eyeballs and don’t care how you’re feeling. But when you share a story with your friends, you care a lot more how they react. You don’t want them to think of you as a Debbie Downer.”‎ ‎ Researchers analyzing word-of-mouth communication—e-mails, Web posts and reviews, face-to-face conversations—found that it tended to be more positive than negative(消极的), but that didn’t necessarily mean people preferred positive news. Was positive news shared more often simply because people experienced more good things than bad things? To test for that possibility, Dr. Berger looked at how people spread a particular set of news stories: thousands of articles on The New York Times’ website. He and a Penn colleague analyzed the “most e-mailed” list for six months. One of his first findings was that articles in the science section were much more likely to make the list than non-science articles. He found that science amazed Times’ readers and made them want to share this positive feeling with others.‎ ‎ Readers also tended to share articles that were exciting or funny, or that inspired negative feelings like anger or anxiety, but not articles that left them merely sad. They needed to be aroused(激发) one way or the other, and they preferred good news to bad. The more positive an article, the more likely it was to be shared, as Dr. Berger explains in his new book, “Contagious: Why Things Catch On.” ‎ ‎12 . What do the classic rules mentioned in the text apply to?‎ A. News reports. B. Research papers.‎ C. Private e-mails. D. Daily conversations.‎ ‎13. What can we infer about people like Debbie Downer?‎ A. They’re socially inactive.‎ B. They’re good at telling stories.‎ C. They’re inconsiderate of others.‎ D. They’re careful with their words.‎ ‎14. Which tended to be the most e-mailed according to Dr. Berger’s research?‎ A . Sports new. B. Science articles.‎ C. Personal accounts. D. Financial reviews.‎ ‎15 . What can be a suitable title for the text?‎ A. Sad Stories Travel Far and Wide B .Online News Attracts More People C. Reading Habits Change with the Times D. Good News Beats Bad on Social Networks ‎【答案】‎ ‎12. A ‎13. C ‎14. B ‎15. D ‎【解析】‎ 试题分析:人们常说“没有消息就是最好的消息”,类似的传统说法只适合于大众媒体。在网络普及的时代,好消息在网络上的传播速度比坏消息要快很多。‎ ‎14. B细节理解题。根据第三段“articles in the science section were much more likely to make the list than non-science articles”可知,科技类的文章比非科技类的更有可能被人们讨论,故选B。‎ ‎15. D选择最佳标题。根据第一段“By tracking people’s e-mails and online posts, scientists have found that good news can spread faster and farther than disasters and sob stories.”可知,好消息在网络上传播得更快,影响更深远;说明文章主要讲的是好消息通过网络的传播,故选D。‎ 考点:风俗文化类短文阅读 ‎【名师点睛】‎ 主旨大意题主要考查学生把握全文主题和理解中心思想的能力。根据多年的备考及高考实践,这类题目考察的范围是:基本论点、文章标题、主题或段落大意等。它要求考生在理解全文的基础上能较好地运用概括、判断、归纳、推理等逻辑思维方法,对文章进行高度概括或总结,属于高层次题。 选择“主题”旨在考查考生是否掌握了所读文章的主要内容或主旨,通常用词、短语或句子来概括。常见的提问方式有: 1. What is the main / general idea of this text? 2. What is mainly discussed in this passage? 3. What is the text mainly about? 4. This text mainly tells us ‎ ________. 5. This passage mainly deals with _________. 6. The main idea of this passage may be best expressed as_________. 选择“标题”则是让考生给所读的文章选择一个合适的标题。通常标题由一个名词或名词短语充当,用词简短、精练。常见的提问方式有: 1. What would be the best title for the text? 2. Which of the following is the best / most suitable title for this text? 3. The best / most suitable title for this text would be ________. 4. The topic of this passage is _________. 不管是选择“主题”还是选择“标题”,实质上都是要求考生从整体上理解语篇内容,找出贯穿语篇的主线;不管是何种体裁的文章,总是围绕一个主题来展开的。在试题设计上,3个干扰项内容一般在文章里或多或少都有涉及,但并非主要内容,需要注意甄别。‎ ‎7.【2019·北京】A Dear Alfred, ‎ ‎ I want to tell you how important your help is to my life.‎ ‎ Growing up, I had people telling me I was too slow, though, with an IQ of 150 at 17, I’m anything but stupid. The fact was that I was found to have ADIID(注意力缺陷多动障碍). Anxious all the time, I was unable to keep focused for more than an hour at a time.‎ ‎ However, when something did interest me, I could become absorbed. In high school, I became curious about the computer, and built my first website. Moreover, I completed the senior course of Computer Basics, plus five relevant pre-college courses.‎ ‎ While I was exploring my curiosity, my disease got worse. I wanted to go to college after high school, but couldn’t . So, I was killing my time at home until June 2019 when I discovered the online computer courses of your training center.‎ ‎ Since then, I have taken courses like Data Science and Advanced Mathematics. Currently, I’m learning your Probability course. I have hundreds of printer paper, covered in self-written notes from your video. This has given me a purpose.‎ ‎ Last year, I spent all my time looking for a job where, without dealing with the public , I could work alone, but still have a team to talk to. Luckily, I discovered the job—Data Analyst—this month and have been going full steam ahead. I want to prove that I can teach myself a respectful profession, without going to college, and be just as good as, if not better than,‎ ‎ my competitors.‎ ‎ Thank you. You’ve given me hope that I can follow my heart. For the first time, I feel good about myself because I’m doing something, not because someone told me I was doing good. I feel whole.‎ ‎ This is why you’re saving my life.‎ Yours,‎ Tanis ‎56.why did’t Tanis go to college after high school?‎ A.She had learned enough about computer science B. She had more difficulty keeping foucesed C.She preferred taking online courses D.She was too slow to learn ‎57.AS for the working environment,Tains prefers____.‎ A .working by herself B.dealing with the public C.competing against others D.staying with ADHD students ‎58.Tanis wrote this letter in order to_____.‎ A.explain why she was interested in the computer B.share the ideas she had for her profession ‎ C .show how grateful she was to the center D.describe the courses she had taken so far ‎【答案】‎ ‎56.B ‎57.A ‎58.C ‎【解析】‎ 试题分析:这是一封书信。作者患有多动症,因为注意力不能集中,作者无法上大学。作者在家通过网络自学了计算机知识,并找到了心仪的工作。作者认为自己能够和同龄人做得一样好。‎ ‎56.B细节理解题。根据第四段“my disease got worse. I wanted to go to college after high ‎ school, but couldn’t ”可知,“我”的多动症加重了,无法上大学,故选B。‎ ‎57.A细节理解题。根据倒数第三段“without dealing with the public”排除B;根据“I could work alone, but still have a team to talk to. ”可知,“我”不想处理复杂的人际关系,想单独工作,故选A。‎ ‎58.C写作意图题。根据倒数第二段“Thank you. You’ve given me hope that I can follow my heart. For the first time, I feel good about myself”可知,作者写信的目的是表达感谢,故选C。‎ 考点:人物故事类短文阅读 ‎ 【名师点睛】‎ 细节理解题的类型:‎ ‎1、直接信息题 对此类题型,考生可以首先从问题中找到关键词,然后以此为线索,运用略读及查阅的技巧在文中迅速寻找这一细节,找到后再把这一部分内容仔细阅读一遍,仔细比较所给选项与文中细节的细微区别,在准确理解细节的前提下,最后确定最佳答案。56、57题都可以直接在文中找到有关信息。 2、间接信息题 做这类题目时,考生需要对原文信息进行加工处理,然后再进行进一步的推理或鉴别,这是介于事实细节题和推理判断题之间的一种题型,对考生来说有一定难度。 3、数字计算题 数字计算题也是近几年高考中常考的内容,此类试题是在文章中直接表现出来细节事实,有的要经过具体的计算才能够得出正确的答案。具体的计算题可以是对年代的计算、月份的计算或比例的计算等。文章中经常会出现许多数字,它们对解题产生一定的影响。解答此类试题的方法是先来理解文章的大意,然后经过对比、分析、计算等就能够得出正确的答案。 4、排列顺序题 这种试题要求考生根据事件发生的先后顺序和句子之间的逻辑关系,找出事件发生的正确顺序。做这种题时可采用“首尾定位法”,即最先找出第一个事件和最后一个事件,迅速缩小选择范围,从而快速选出正确答案。 5、图表图画题 在有图表图画的阅读理解中,有的图表图画出现在阅读理解文章中,有的出现在选项中,这些图片的出现增加了试题的直观性,同时也暗含着和文章内容相关的信息。在解答此类试题的时候,一定要把握图表图画中所暗含的信息,特别是有些事实是通过图片来叙述的,我们可以采用按图寻找正确答案的方法。‎ ‎8.【2019·北京】D Why College Is Not Home ‎ The college years are supposed to be a time for important growth in autonomy(自主性) and the development of adult identity. However, now they are becoming an extended period of adolescence, during which many of today’s students and are not shouldered with adult responsibilities.‎ ‎ For previous generations, college was decisive break from parental control; guidance and support needed help from people of the same age and from within. In the past two decades, however, continued connection with and dependence on family, thanks to cellphones, email and social media, have increased significantly. Some parents go so far as to help with coursework. Instead of promoting the idea of college as a passage from the shelter of the family to autonomy and adult responsibility, universities have given in to the idea that they should provide the same environment as that of the home.‎ ‎ To prepare for increased autonomy and responsibility, college needs to be a time of exploration and experimentation. This process involves “trying on ” new ways of thinking about oneself both intellectually(在思维方面) and personally. While we should provide “safe spaces” within colleges, we must also make it safe to express opinions and challenge majority views. Intellectual growth and flexibility are fostered on debate and questioning.‎ ‎ Learning to deal with the social world is equally important. Because a college community(群体) differs from the family, many students will struggle to find a sense of belonging. If students rely on administrators to regulate their social behavior and thinking pattern, they are not facing the challenge of finding an identity within a larger and complex community.‎ ‎ Moreover, the tendency for universities to monitor and shape student behavior runs up against another characteristic of young adults: the response to being controlled by their elders. If acceptable social behavior is too strictly defined(规定) and controlled, the insensitive or aggressive behavior that administrators are seeking to minimize may actually be encouraged.‎ ‎ It is not surprising that young people are likely to burst out, particularly when there are reasons to do so. Our generation once joined hands and stood firm at times of national emergency. What is lacking today is the conflict between adolescent’s desire for autonomy and their understanding of an unsafe world. Therefore, there is the desire for their dorms to be replacement homes and not places to experience intellectual growth.‎ ‎ Every college discussion about community values, social climate and behavior should include recognition of the developmental importance of student autonomy and self-regulation, of the ‎ necessary tension between safety and self-discovery.‎ ‎67. What’s the author’s attitude toward continued parental guidance to college students?‎ A.Sympathetic B.Disapproving C.Supportive D.Neutral ‎68. The underlined word “passage” in Paragraph 2 means .‎ A.change B.choice C.text D.extension ‎69. According to the author, what role should college play?‎ A.to develop a shared identity among students B.to define and regulate students’ social behavior C.To provide a safe world without tension for students ‎ D.To foster students’ intellectual and personal development ‎70. Which of the following shows the development of ideas in the passage?‎ ‎【答案】‎ ‎67. B68. A69. D70. C ‎【解析】‎ 试题分析:大学并不是温暖的家庭,也不能成为学生的家庭。大学时期应该是培养自主性和自我同一性的重要时期。大学是孩子脱离父母控制的决定性时期,因此,学校不应该成为像家庭一样让孩子依赖的地方。‎ ‎69. D推理判断题。每个人的个性都是不同的,排除A;根据第四段“If students rely on administrators to regulate their social behavior and thinking pattern, they are not facing...”可知,作者不赞成大学管理者约束学生的社会行为,排除B;大学不是一个无忧无虑的场所,排除C;大学应该是学习知识和促进学生成长的地方,故选D。‎ ‎70. C篇章结构题。根据第四段“Learning to deal with the social world is equally important.”中的“equally important”和第五段的“moreover”可知,第四段和第五段是第三段的次要论点,故选C。‎ 考点:教育类短文阅读。 ‎ ‎【名师点睛】 I. 三类篇章结构题的出题方式 ‎1. 结构识辨: 此类组织结构题要求考生能够识别文章、段落的组织结构或行文方式。 结构识辨类组织结构题常见的题干表述方式如下: ‎ l How is the text organized?‎ l The author develops the passage mainly by________.‎ l The text is mainly developed in the order of________.‎ l Which of the following shows the structure of the passage? ‎ ‎2. 组织结构功能: 此类组织结构题要求考生判断文章或段落组织结构的功能、作用,其常见的题干表述方式如下:‎ The author quoted …words in...paragraph in order to make the article________.‎ ‎3. 结构推测: 此类组织结构题要求考生能够根据文章或段落的组织结构、行文方式或段落内容推测文章前后未呈现的写作内容。其常见的题干表述方式如下: ‎ What would the author probably discuss in the paragraph that follows?‎ Which of the following do you think has been discussed in the part before this selection?‎ II. 解题策略 考查组织结构的文章通常行文组织结构都比较清晰、规范,因此,具备文体、写作方式和文章组织结构方面相关知识对于有效解答此类题目具有至关重要的作用。‎ 1) 记叙文可以采取以时间为序、以地点为序、以故事情节发展为序(开端、发展、高潮、结局)等叙事方式进行写作。以故事情节为序又可以不按照时间的先后顺序叙述,比如,先写结局,再写其他, 最后写高潮,这种叙事手法叫做倒叙。‎ 2) 议论文总体上可以分为四类:‎ 第一类,“提出论点——分论点一——分论点二——分论点三……——结论”;‎ 第二类,“引入段——导出论点——分点论述——结论”;‎ 第三类,“提出问题——分析问题——解决问题”;‎ 第四类,“提出反面观点——批驳反面观点——提出正面观点”。‎ ‎ 3) 说明文往往采取下定义、举例子、列数字、对比或比较等说明方式进行写作,而这些说明方式都是为了使文章要说明的内容或问题更加清晰明了、真实可信,或者更具说服力。‎ ‎9.【2019·天津】C ‎ When John was growing up, other kids felt sorry for him. His parents always had him weeding the garden, carrying out the garbage and delivering newspapers. But when John reached adulthood, he was better off than his childhood playmates. He had more job satisfaction, a better marriage and was healthier. Most of all, he was happier. Far happier.‎ ‎ These are the findings of a 40-year study that followed the lives of 456 teenage boys from Boston. The study showed that those who had worked as boys enjoyed happier and more productive lives than ‎ those who had not. “Boys who worked in the home or community gained competence (能力) and came to feel they were worthwhile members of society,” said George Vaillant, the psychologist (心理学家) who made the discovery. “And because they felt good about themselves, others felt good about them.”‎ ‎ Vaillant’s study followed these males in great detail. Interviews were repeated at ages 25, 31 and 47. Under Vaillant, the researchers compared the men’s mental-health scores with their boyhood-activity scores with their boyhood-activity scores. Points were awarded for part-time jobs, housework, effort in school, and ability to deal with problems.‎ ‎ The link between what the men had done as boys and how they turned out as adults was surprisingly sharp. Those who had done the most boyhood activities were twice as likely to have warm relations with a wide variety of people, five times as likely to be well paid and 16 times less likely to have been unemployed. The researchers also found that IQ and family social and economic class made no real difference in how the boys turned out. Working----at any age----is important. Childhood activities help a child develop responsibility, independence, confidence and competence---the underpinnings (基础) of emotional health. They also help him understand that people must cooperate and work toward common goals. The most competent adults are those who know how to do this. Yet work isn’t everything. As Tolstoy once said, “One can live magnificently in this world if one knows how to work and how to love, to work for the person one loves and to love one’s work.”‎ ‎46. What do we know about John?‎ A. He enjoyed his career and marriage.‎ B. He had few childhood playmates.‎ C. He received little love from his family.‎ D. He was envied by others in his childhood.‎ ‎47. Vaillant’s words in Paragraph 2 serve as _____.‎ A. a description of personal values and social values B. an analysis of how work was related to competence C. an example for parents’ expectations of their children D. an explanation why some boys grew into happy men ‎48. Vaillant’s team obtained their findings by _____.‎ A. recording the boys’ effort in school B. evaluating the men’s mental health C. comparing different sets of scores D. measuring the men’s problem solving ability ‎49. What does the underlined word “sharp” probably mean in Paragraph 4?‎ A. Quick to react B. Having a thin edge C. Clear and definite D. sudden and rapid ‎50. What can be inferred from the last paragraph?‎ A. competent adults know more about love than work.‎ B. Emotional health is essential to a wonderful adult life.‎ C. Love brings more joy to people than work does.‎ D. Independence is the key to one’s success.‎ ‎【语篇解读】这是一篇说明文。文章通过John的例子结合一个研究发现说明童年时期帮助做一些家务的孩子长大后会更快乐,更成功。‎ ‎46.【答案】A 考点:考查细节理解。‎ ‎47.【答案】D ‎【解析】‎ 试题分析:根据第二段的句子Boys who worked in the home or community gained competence and came to feel they were worthwhile members of society,可知,第二段Vaillant的话是解释为什么一些男孩成长为快乐的人。故选D。‎ 考点:考查推理判断。‎ ‎48.【答案】C ‎【解析】‎ 试题分析:根据第三段的句子The researchers compared the men’s mental-health scores with their boyhood-activity scores,可知,Vaillant的团队获得他们的发现是通过对比不同的得分。故选C。‎ 考点:考查细节理解。‎ ‎49.【答案】C ‎ ‎【解析】‎ 试题分析:根据第四段的句子The link between what the men had done as boys and how they turned out as adults was surprisingly sharp(这些人在孩子的时候做的事情和他们成人后是怎样的人之间的关联是令人惊讶地明显),结合下面一句进一步可知,这里sharp的意思是“明确的,一定的”。故选C。‎ 考点:考查词义猜测。‎ ‎50.【答案】B ‎ ‎【解析】‎ 试题分析:根据最后一段的句子One can live magnificently in this world if one knows how to work and how too love, to work for the person one loves and to love one’s work,可知,情感健康对于好的成年人生活是基本的。故选B。‎ 考点:考查推理判断。‎ ‎【名师点睛】这篇文章内容和学生自己的生活密切相关,而且是通过调查研究的方式给出,更加有说服力,虽然是考试题,学生应该有兴趣阅读,文章篇幅适中。考查比较全面,除了理解细节题,还需要适当的推理,还有词义辨析类的题目,对学生能力考查比较全面。‎ 做词义辨析类的题目,要仔细阅读这个词所在的前后的句子,发现有没有关联的同义词,反义词或是这个词的定义,分析这篇阅读的49题,要求判断sharp的含义,这个词不是陌生的单词,高中教材上出现过,它的意思是“锋利的,尖锐的,敏捷的,聪明的”,但是代入后,发现这些意思都不符合,而结合所在句子,特别是下文的Those who had done the most boyhood activities were twice as likely to have warm relations with a wide variety of people, five times as likely to be well paid and 16 times less likely to have been unemployed.可知和上文的观点是一致的,也就是这些人在孩子的时候做的事情和他们成人后是怎样的人之间的关联是令人惊讶地明显。答案也不难得出。‎ ‎10.【2019·天津】D ‎ Failure is probably the most exhausting experience a person ever has. There is nothing more tiring than not succeeding.‎ We experience this tiredness in two ways: as start-up fatigue(疲惫) and performance fatigue. In the former case, we keep putting off a task because it has either too boring or too difficult. And the longer we delay it, the more tired we feel.‎ ‎ Such start-up fatigue is very real, even if not actually physical, not something in our muscles and bones. The solution is obvious though perhaps not easy to apply: always handle the most difficult job first.‎ Years ago, I was asked to write 102 essays on the great ideas of some famous authors. Applying my own rule, I determined to write them in alphabetical(按字母顺序), never letting myself leave out a tough idea. And I always started the day’s work with the difficult task of essay-writing. Experience proved that the rule works.‎ Performance fatigue is more difficult to handle. Though willing to get started, we cannot seem to do the job right. Its difficulties appear so great that, however hard we work, we fail again and again. In such a situation, I work as hard as I can-then let the unconscious take over.‎ ‎ When planning Encyclopaedia Britannica (《大英百科全书》), I had to create a table of contents based on the topics of its articles. Nothing like this had ever been done before, and day after dat I kept coming up with solutions, but none of them worked. My fatigue became almost unbearable.‎ One day, mentally exhausted, I wrote down all the reasons why this problem could not be solved. I tried to convince myself that the trouble was with the problem itself, not with me. Relived, I sat back in an easy chair and fell asleep.‎ ‎ An hour later, I woke up suddenly with the solution clearly in mind. In the weeks that followed, the solution which had come up in my unconscious mind provided correct at every step. Though I worked as hard as before, I felt no fatigue. Success was now as exciting as failure had been depressing.‎ Human beings, I believe must try to succeed. Success, then, means never feeling tired.‎ ‎51. People with start-up fatigue are most likely to .‎ A. delay tasks B. work hard C. seek help D. accept failure ‎52. What does the author recommend doing to prevent start-up fatigue?‎ A. Writing essays in strict order.‎ B. Building up physical strength.‎ C. Leaving out the toughest ideas.‎ D. Dealing with the hardest task first.‎ ‎53. On what occasion does a person probably suffer from performance fatigue?‎ A. Before starting a difficult task.‎ B. When all the solutions fail.‎ C. If the job is rather boring.‎ D. After finding a way out.‎ ‎54. According to the author, the unconscious mind may help us .‎ A. ignore mental problems B. get some nice sleep C. gain complete relief D. find the right solution ‎55. What could be the best title for the passage?‎ A. Success Is Built upon Failure B. How to Handle Performance Fatigue C. Getting over Fatigue: A Way to Success D. Fatigue: An Early Sign of Health Problems ‎【语篇解读】这是一篇夹叙夹议的文章。文章主要介绍每个人都希望成功,但是人们在实施任务的时候会出现启动疲惫和表现疲惫,这是造成失败的两种原因,文章具体介绍如何克服这两种疲惫。‎ ‎51.【答案】A ‎【解析】‎ 试题分析:根据第一段的句子In the former case, we keep putting off a task,可知,有启动疲惫的人更可能拖延任务。故选A。‎ 考点:考查细节理解。‎ ‎52.【答案】D 考点:考查细节理解。‎ ‎53.【答案】B ‎【解析】‎ 试题分析:根据第五段的句子Performance fatigue is more difficult to handle…we fail again and again,可知,当所有的解决方法都失败的时候,一个人可能会有表现疲惫。故选B。‎ 考点:考查细节理解。‎ ‎54.【答案】D ‎【解析】‎ 试题分析:根据第五段的最后一句话however hard we work, we fail again and again. In such a ‎ situation, I work as hard as I can-then let the unconscious take over.,可知,无意识的行为可能帮助我们发现正确的解决方法。故选D。‎ 考点:考查细节理解。‎ ‎55.【答案】C ‎【解析】‎ 试题分析:根据全篇文章和最后一段的内容,可知,文章主要介绍造成失败的两种疲惫,以及如何克服疲惫。故选C。‎ 考点:考查标题判断。‎ ‎【名师点睛】这篇文章内容不是学生非常熟悉的,篇幅适中较长,要求学生快速阅读材料,在掌握文章大意的前提下答题。考查题型以理解细节题为主,还要学生结合文章主旨给出适当的标题,这些不是某个句子可以体现的,要对文章有整体把握。 ‎ 这篇文章最后一题就是标题判断题。学生对标题判断题不能很好的把握,主要还是没有弄清主旨,分析四个选项都出现了关键词Fatigue,但是A项说:成功是建立在疲惫的基础上,显然和文章内容矛盾,B项是怎么处理表现疲惫,明显不够全面,D项虽然也讲的是疲惫,但是将它定义为健康问题的迹象,这和文章内容不符。通过全篇正确理解不难得出答案。‎ ‎11.【2019·四川】B ‎ If you could have one superpower, what would it be?‎ ‎ Dreaming about whether you would want to read minds, see through walls, or have superhuman strength may sound silly, but it actually gets to the heart of what really matters in your life.‎ ‎ Every day in our work, we are inspired by the people we meet doing extraordinary things to improve the world.‎ They have a different kind of superpower that all of us possess: the power to make a difference in the lives of others.‎ We’re not saying that everyone needs to contribute their lives to the poor. Your lives are busy enough doing homework, playing sports, making friends, seeking after your dreams. But we do think that you can live a more powerful life when you devote some of your time and energy to something much larger than yourself. Find an issue you are interested in and learn more. Volunteer or, if you can, contribute a little money to a cause. Whatever you do, don’t be a bystander. Get involved. You may have the opportunity to make your biggest difference when you’re older. But why not start now?‎ Our own experience working together on health, development, and energy the last twenty years has been one of the most rewarding parts of our lives. It has changed who we are and continues to fuel our optimism about how much the lives of the poorest people will improve in the years ahead.‎ ‎4. What does the underlined part in Paragraph 2 refer to?‎ A. Your life style.‎ B. Your life value.‎ C. Your trouble in life.‎ D. Your life experience.‎ ‎5. Why does the author say they are inspired every day?‎ A. They possess different kinds of superpowers.‎ B. They have got the power to change the world.‎ C. Some people around them are making the world better.‎ D. There are many powerful people in their life and work.‎ ‎6. What does the author stress in Paragraph 5?‎ A. Learning more and contributing more to a cause.‎ B. Rising above self and acting to help others.‎ C. Working hard to get a bigger opportunity.‎ D. Trying your best to help the poor.‎ ‎7. What can be inferred from the last paragraph?‎ A. The author believes the lives of the poorest will get better.‎ B. Much more progress will be made in the near future.‎ C. The work on health is the most valuable experience.‎ D. People’s efforts have been materially rewarded. ‎ ‎【答案】4.B 5.C 6.B 7.A ‎【解析】‎ 试题分析:本文以虚拟语气的问句形式开头,鼓励人们奉献自己的一些时间和精力去帮助别人,最穷人的生活将会变得更好。‎ ‎4. B 词义猜测题。根据文章开始提到的If you could have one superpower, what would it be?如果你有超能,它将会是什么呢?Dreaming about whether you would want to read minds, see through walls, or have ‎ superhuman strength may sound silly,梦想着是否你想读懂心思,看穿墙,或有超能的力量,这些可能听起来是愚蠢的,but it actually gets to the heart of what really matters in your life.但是它实际上是在你的生活中真正重要的核心,也就是你生活的价值。故选B。‎ ‎5.C 细节理解题。根据第三段Every day in our work, we are inspired by the people we meet doing extraordinary things to improve the world.每天在我们的工作中,我们会收到一些人的鼓舞,他们在做一些改善世界的事情。故选C。‎ ‎7.A 推理判断题。根据最后一段最后一句It has changed who we are and continues to fuel our optimism about how much the lives of the poorest people will improve in the years ahead.它已经改变了我们是谁,在以后的这些年里最穷人的生活将会得到改善。故选A。‎ ‎【名师点睛】‎ 推理判断题要求在理解原文表面文字信息的基础上,做出一定的推理判断,从而得到文章的隐含意义和深层意义。推理判断题所涉及的内容可能是文中的某一句话,也可能是某几句话,所以,推理题的答案只能是根据原文表面文字信息一步推出的答案:即对原文某一句话或某几句话所作的同义改写或综合。推理判断题的题干中通常含有infer, suggest, imply, conclude  indicate等标志性词语。 这种题型主要包括细节判断题、态度观点推断题、写作意图推断题、文章出处判断题和猜测想象推断题。比如第7小题A 推理判断题。根据最后一段最后一句It has changed who we are and continues to fuel our optimism about how much the lives of the poorest people will improve in the years ahead.它已经改变了我们是谁,在以后的这些年里最穷人的生活将会得到改善。故选A。‎ 考点:考查日常生活类阅读 ‎12.【2019·四川】C ‎ In the depths of the French Guianese rainforest, there still remain unusual groups of indigenous(土著的) people. Surprisingly, these people live largely by their own laws and their own social customs. And yet, people in this area are in fact French citizens because it has been a colony(殖民地) of the French Republic since 1946. In theory, they should live by the French law is often ignored or unknown, thus making them into an interesting area of “lawlessness” in the world.‎ The lives of these people have finally been recorded thanks to the effects of a Frenchman form Paris called Gin. Gin spent five months in early 2019 exploring the most remote corners of this area, which sits on the edge of the Amazon rainforest, with half its population of only 250,000 living in its capital, Cayenne.‎ ‎“I have a special love for the French Guianese people. I have worked there on and off for almost ten years,” says Gin. “I’ve been able to keep firm friendships with them. Thus I have been allowed to gain access to their living environment. I don’t see it as a lawless land. But rather I see it as an area of freedom.”‎ ‎“I wanted to show the audience a photographic record touching upon the uncivilized life,” continues Gin. “I prefer to work in black and white, which allows me to show different specific worlds more clearly.”‎ His black-and-white pictures present a world almost lost in time. These pictures show people seemingly pushed into a world that they were unprepared for. These local citizens now have to balance their traditional self-supporting hunting lifestyle with the lifestyle offered by the modern French Republic, which brings with it not only necessary state welfare, but also alcoholism, betrayal and even suicide.‎ ‎8. Why does the author feel surprised about the indigenous people in French Guiana?‎ ‎ A. They seldom follow the French law.‎ ‎ B. They often ignore the Guianese law.‎ C. They are separated from the modern world.‎ ‎ D. They are both Guianese and French citizens.‎ ‎9.Gin introduced the special world of the indigenous Guianese as _________.‎ A. a tour guide ‎ B. a geographer C. a film director ‎ D. a photographer ‎10. What is Gin’s attitude towards the lives of the indigenous Guianese?‎ A. Cautious.‎ ‎ B. Doubtful.‎ C. Uninterested.‎ ‎ D. Appreciative.‎ ‎11.What does the underlined word “it” in the last paragraph refer to?‎ A. The modern French lifestyle.‎ ‎ B. The self-supporting hunting.‎ C. The uncivilized hunting.‎ ‎ D. The French Republic.‎ ‎【答案】8.A 9.D 10.D 11.A ‎【解析】‎ 试题分析:本文通过一个摄影记者向人们展示了法国的Guianese人,和现代法国的生活风格给他们带来的影响。‎ ‎8.A推理判断题。根据第一段第二句Surprisingly, these people live largely by their own laws and their own social customs.令人吃惊的是,这些人主要靠自己的法律和他们的社会风俗生活。可知A项符合题意。‎ ‎9.D推理判断题。根据第四段第一句“I wanted to show the audience a photographic record touching upon the uncivilized life,” 我想展示给观众一个摄影记录,让他们触摸到不文明的生活。第五段第一句His black-and-white pictures present a world almost lost in time. 他的黑白照片呈现了一个几乎失去的世界。可知他是一个摄影师,故选D。‎ ‎10.D推理判断题。根据第三段第一、二句“I have a special love for the French Guianese people. I have worked there on and off for almost ten years,” 我对法国Guianese人有一个特殊的爱。我在那里工作了将近十年了。所以他对法国Guianese人是心存感激的,故选D。‎ ‎11.A词义猜测题。根据最后一段最后一句中the lifestyle offered by the modern French Republic,被法兰西共和国提供的生活风格,可知后面的定语从句中it指现代法国的生活风格。故选A。‎ 考点:考查风土人情类阅读。 ‎ ‎13.【2019·四川】D A warm drink of milk before bed has long been the best choice for those wanting a good night’s sleep. But now a study has found it really does help people nod off—if it is milked from a cow at night.‎ ‎ Researchers have discovered that “night milk” contains more melatonin(褪黑激素), which has been proven to help people feel sleepy and reduce anxiety.‎ ‎ The study, by researchers from Seoul, South Korea, involved mice being fed with dried milk powder made from cows milked both during the day and at night.‎ ‎ Those given night milk, which contained 10 times the amount of melatonin, were less active and less anxious than those fed with the milk collected during daytime, according to the study published in The Journal of Medicinal Food.‎ ‎ Night milk quickened the start of sleep and caused the mice to sleep longer.‎ ‎ While the effect of cows milk harvested at different time has not been tested on humans up to now, taking melatonin drugs has been suggested to those who are struggling to fall asleep at night.‎ ‎ Previous studies have also indicated that milk can be excellent for helping sleep because of the calcium content, which helps people to relax.‎ ‎ Milk is also sugar-free and additive-free with nutritionists recommending skimmed milk as the best choice before bed as it is the least fattening. The more fat you take in before bedtime, the greater burden you will put on your body at night.‎ ‎12. According to the text, the mice fed with daytime milk_______.‎ ‎ A. started sleep more easily ‎ B. were more anxious ‎ C. were less active ‎ D. woke up later ‎13. Which of the following is true of melatonin according to the text?‎ ‎ A. It’s been tested on mice for ten years ‎ ‎ B. It can make people more energetic ‎ C. It exists in milk in great amount ‎ D. It’s used in sleeping drugs ‎14. What can be a suitable title for the text?‎ A. Night Milk and Sleep ‎ ‎ B. Fat Sugar and Health C. An Experiment on Mice D. Milk Drinking and Health ‎15.How does the author support the theme of the text?‎ ‎ A. By giving examples.‎ ‎ B. By stating arguments.‎ ‎ C. By explaining statistical data.‎ ‎ D. By providing research results.‎ ‎【答案】12.B 13.D 14.A 15.D ‎【解析】‎ 试题分析:本文讲的是晚上牛奶和睡眠。‎ ‎12.B推理判断题。根据第四段第一句Those given night milk, which contained 10 times the amount of melatonin, were less active and less anxious than those fed with the milk collected during daytime,晚上喂牛奶的那些比白天喂牛奶的那些更不活跃,更不焦虑。可知白天喂牛奶的那些更焦虑。故选B。‎ ‎13.D推理判断题。根据第五段Night milk quickened the start of sleep and caused the mice to sleep longer.晚上牛奶加快睡眠的开始,使老鼠睡的时间更长。可知D项符合题意。‎ ‎14.A主旨大意题。根据第二段Researchers have discovered that “night milk” contains more melatonin(褪黑激素), which has been proven to help people feel sleepy and reduce anxiety. 研究者已经发现晚上牛奶包含更多的褪黑激素,这已经被证明能帮助人们感到困倦和减少焦虑。可知A项符合题意。‎ ‎【名师点拨】‎ 抓住文章主线和关键词语,归纳文章中心 要注意不是所有的段落都有主题句,有时主题句暗含在句中。阅读这样的文章,就需要自己根据文章的细节来分析,概括出段落的主题,从而推导出文章的主旨。分析的方法是:先弄清该段落主要讲了哪几方面的内容,这些内容在逻辑上有什么联系,然后加以归纳形成主题。比如第14小题A主旨大意题。根据第二段Researchers have discovered that “night milk” contains more melatonin(褪黑激素), which has been proven to help people feel sleepy and reduce anxiety. 研究者已经发现晚上牛奶包含更多的褪黑激素,这已经被证明能帮助人们感到困倦和减少焦虑。可知A项符合题意。‎ 考点:考查健康类阅读 ‎14. 【2019·浙江】 A ‎ “Did you hear what happened to Adam Last Friday?”Lindsey whipers to Tori.‎ ‎ With her eyes shining,tori brags,“You bet I did,Sean told me two days ago.”‎ ‎ Who are Lindsey and Tori talking about?It just happened to be yours truly,Adam Freedmam,I can tell you that what that what they are saying is (a) not nice and (b) not even true.Still,Lindsey and Tori aren’t very different from most students here at Linton Higt School,including me.Many of our conversations are gossip(闲话)。I have noticed three effects of gossip:it can hurt people,it can give gossipers a strange kind of satisfaction,and it can cause social pressures in a group.‎ ‎ An important negative effect of gossip is that it can hurt the person being talked about.Usually,gossip spreads information about a topic-breakups,trouble at home,even dropping ‎ out-that a person would rather keep secret.The more embarrassing or shameful the secret is,the juicier the gossip it makes.Probably the worst type of gossip is the absolute lie.People ofen think of gossipers as harmless,but cruel lies can cause pain.‎ ‎ If we know that gossip can be harmful,then why do so many of us do it?Tht answer lies in another effect of gossip:the satisfaction it gives us.Sharing the latest rumor(传言)can make a person feel important because he or she knows something that others don’t.Similarly,hearing the latest rumor can make a person feel like part of the “in group.”In other words,gossip is satisfying because it gives people a sense of belonging or even superiority(优越感).‎ ‎ Gossip also can have a third effect:it strengthens unwritten,unspoken rules about how people should act.Professor David Wilson explains that gossip is important in policing behaviors in a group.Translated into high school terms, this means that if everybody you hang around with is laughing at what John wore or what Jane said,then you can bet that wearing or saying something similar will get you the same kind of negative attention.The do’s and don’ts conveyed through gossip will never show up in any student handbook.‎ ‎ The effects of gossip vary depending on the situation.The next time you feel the urge to spread the latest news,thing about why you want to gossip and what effects your “juicy story”might have.‎ ‎41.The author uses a conversation at the beginning of the passage to .‎ A.introduce a topic B.present an argument ‎ C.describe the characters D.clarify his writing purpose ‎42.An important negative effects of gossip is that it .‎ A.breaks up relationships ‎ B.embarrasses the listener C.spreads information around D.causes unpleasant experiences ‎43.In the auther’s opinion,many people like to gossip because it .‎ A.gives them a feeling of pleasure B.help them to make more friends ‎ C.makes them better at telling stories D.enables them to meet important people ‎44.Professor David Wilson think that gossip can .‎ A.provide students with written rules B.help people watch their own behaviors C.force school to impove student handbooks D.attract the police’s attention to group behaviors ‎45.What advice does the author give in the passage?‎ A.Never become a gossiper B.Stay away from gossipers C.Don’t let gossip turn into lies D.Think twice before you gossip.‎ ‎【语篇解读】本文是一篇议论文。文章讨论了传言的危害和人们为什么喜欢传闲话。‎ ‎41. 【答案】A ‎【解析】‎ 试题分析:根据文章第三段的Many of our conversations are gossip. (我们的话题很多都是闲话),可知开头的对话是为了介绍本文的主题, 故选A。‎ 考点:考查推理判断。‎ ‎42. 【答案】D ‎【解析】‎ 试题分析:根据文章第四段的An important negative effect of gossip is that it can hurt the person being talked about.,可知主要危害是他会给被讨论的人带来不愉快的经历,故选D。‎ 考点:考查细节理解。‎ ‎43. 【答案】A ‎【解析】‎ 试题分析:根据文章第五段的The answer lies in another effect of gossip: the satisfaction it gives us,可知人们散播传言是因为它会带来满足感,故选A。‎ 考点:考查细节理解。‎ ‎44. 【答案】B ‎【解析】‎ 试题分析:根据文章第六段的Professor David Wilson explains that gossip is important in policing behaviours in a group.可知,他认为传言能帮助人们观察自身的行为,故选B。‎ 考点:考查细节理解。‎ ‎45. 【答案】D 考点:考查细节理解。‎ ‎【方法点拨】‎ 这是一篇议论文,题目设置以细节理解题为主。学生可以带着题目回到原文,能够在短文中画出解题依据是正确解题的关键,各位考生要谨记,在做阅读理解时要做到“不见依据不做题”。理由充分,依据凿实,这是不仅“知其然”,而且“知其所以然”的超高境界,是阅读水平提高的最大见证。那么如何能快速准确回到原文找到依据呢?‎ ‎1. 关键词定位法。‎ 这里说的关键词并不一定是中心词,而是在理解题干所问之后,在题干中能帮助你迅速回到原文的“特征词”。如43题的信息句是The answer lies in another effect of gossip: the satisfaction it gives us,可知人们散播传言是因为它会带来满足感,和选项A里面的a feeling of pleasure对应。还有44题,题干中的Professor David Wilson可以定位到第六段的Professor David Wilson explains that gossip is important in policing behaviours in a group.可知,他认为传言能帮助人们观察自身的行为,所以答案是B。‎ ‎2. 自然段定位法。‎ 即命题者往往按照信息点在文中出现的先后来依次命制各个小题,也就是说,各题的答案信息常常在短文中依次出现,排在后面的小题,文中对应的信息点也在后面。例如,第1题往往位于一、二自然段,第2题会在第1题的答案信息之后且与第一题的位置最接近,第3题会在第2题的答案信息之后,往往在文中较为中间的地方,而第4题则位于文中较后的位置。最后一题(偶尔可能是第一题)一般是主旨题(包括给文章加标题、归纳写作目的),这时,可能涉及全文内容。如41题,题目问文章的开头的作用,自然定位到第一段的内容,结合下文可以快速解答。‎ ‎2019社会生活类 ‎1.【2019·湖北卷】A ‎“I see you’ve got a bit of water on your coat,” said the man at the petrol station. “Is it raining out there?” “No, it’s pretty nice,” I replied, checking my sleeve. “Oh, right. A pony(马驹) bit me earlier.”‎ As it happened, the bite was virtually painless: more the kind of small bite you might get ‎ from a naughty child. The pony responsible was queuing up for some ice cream in the car park near Haytor, and perhaps thought I’d jumped in ahead of him.‎ The reason why the ponies here are naughty is that Haytor is a tourist-heavy area and tourists are constantly feeding the ponies foods, despite sighs asking them not to. By feeding the ponies, tourists increase the risk of them getting hit by a car, and make them harder to gather during the area’s annual pony drift(迁移).‎ The purpose of a pony drift is to gather them up so their health can be checked, the baby ones can be stooped from feeding on their mother’s milk, and those who’ve gone beyond their limited area can be returned to their correct area. Some of them are also later sold, in order to limit the number of ponies according to the rules set by Natural England.‎ Three weeks ago, I witnessed a small near-disaster a few mils west of here. While walking, I noticed a pony roll over on his back. “Hello!” I said to him, assuming he was just rolling for fun, but he was very still and, as I got closer, I saw him kicking his legs in the air and breathing heavily. I began to properly worry about him. Fortunately, I managed to get in touch with a Dartmoor’s Livestock Protection officer and send her a photo. The officer immediately sent a local farmer out to check on the pony. The pony had actually been trapped between two rocks. The farmer freed him, and he began to run happily around again.‎ Dartmoor has 1,000 or so ponies, who play a critical role in creating the diversity of species in this area. Many people are working hard to preserve these ponies, and trying to come up with plans to find a sustainable(可持续的) future for one of Dartmoor’s most financially-troubled elements.‎ ‎51.Why are tourists asked not to feed the ponies?‎ A. To protect the tourists from being bitten B. To keep the ponies off the petrol station C. To avoid putting the ponies in danger D. To prevent the ponies from fighting ‎52.One of the purposes of the annual pony drift is ______________.‎ A. to feed baby ponies on milk B. to control the number of ponies C. to expand the habitat for ponies D. to sell the ponies at a good price ‎53.What as the author’s first reaction when he saw a pony roll on its back?‎ A. He freed it from the trap B. He called a protection officer C. He worried about it very much D. He thought of it as being naughty ‎54.What does the author imply about the preservation of Dartmoor’s ponies?‎ A. It lacks people’s involvement.‎ B. It costs a large amount of money C. It will affect tourism in Dartmoor.‎ D. It has caused an imbalance of species ‎【解析】‎ 试题分析:这是一篇夹叙夹议文。本文记叙了作者在Haytor关于马驹的所见所闻,目睹了马驹的淘气与脆弱,了解了马驹迁徙的原因以及保护马驹面临的困难。‎ ‎51. C细节理解题。根据第三段尾句“By feeding the ponies, tourists increase the risk of them getting hit by a car, and make them harder to gather during the area’s annual pony drift(迁移).”可知不让游客给马驹喂食是为了避免使马驹处于危险中。故选C项。‎ ‎52. B细节理解题。根据第四段内容“The purpose of a pony drift is to …Some of them are also…, in order to limit the number of ponies…”可知马驹迁移目的之一是为了控制马驹的数量。故选B项。‎ ‎53. D细节理解题。根据第五段中的“…, assuming he was just rolling for fun,…”可知作者认为小马驹很淘气,滚着玩的。故选D项。‎ ‎54. B推理判断题。根据最后一段中尾句“Many people are working hard to preserve these ponies, and trying to come up with plans to find a sustainable(可持续的) future for one of Dartmoor’s most financially-troubled elements.”可知保护马驹是对Dartmoor来说是一笔很大的开销。故选B项。‎ ‎【考点定位】夹叙夹议文阅读 ‎【名师点睛】夹叙夹议文是近几年高考经常涉及的一类文体,文章一方面叙述事情,另一方面又对事情加以评论,此类文章一般先提出论点,然后以事例说明论点,最后得出结论,或者一边叙述事情经过,一边对事情加以分析评论,文章最后一段用一句概括性的话点明主题,将叙述和议论有机地结合起来,其中“叙”是“议”的基础,“议”是“叙”的渗透和延伸。考生要注意在通读全文的基础上弄清文章结构,阅读时重点要放在议论部分,因为该部分表达了作者的观点或看法,是文章的精髓。理解了作者的观点和看法,也就把握了全文的主要内容。本文难度不大,通过作者与小马的“亲密接触”介绍了英国达特穆尔荒原淘气的矮种马,散发出浓浓的英伦风。 ‎ ‎2.【2019·湖北卷】C Hilversum is a medium-sized city between the major cities of Amsterdam and Utrecht in the Gooi area of North Holland, the Netherlands. Unlike most of the Netherlands, Hilversum is actually in a hilly area with the soil mostly consisting of sand. Once called the Garden of Amsterdam, it still attracts travelers to come over to cycle and walk through the surrounding forests. They visit it for a relaxing day off from the urban madness. For Dutch people, Hilversum is all about textile (纺织) and media industries, and modern architecture.‎ In history, Hilversum was largely an agricultural area. Daily life was marked by farming, sheep raising and wool production. A railway link to Amsterdam in 1874 attracted rich traders from Amsterdam to Hilversum. They build themselves large villas (别墅) in the wooded surroundings of the town. One of the families moving in was the Brenninkmeijers, currently the wealthiest family of the Netherlands. They moved in after big success in the textile industry and aided a substantial textile industry in Hilversum. But the textile boom lasted only several decades. The last factory closed in the 1960s.‎ The change to a media economy started in 1920, when the Nederlandse Seintoestedllen Fabriek (NSF) established a radio factory in Hiversum. Most radio stations called in the large villas in the leafy areas of the town. Television gave another push to the local economy. Hilversum became the media capital of the Netherlands, and Dutch television stars moved into the leafy neighborhoods surrounding the town.‎ In the early 1900s, modern architcts W.M. Dudok and J. Duiker placed hundreds of remarkable buildings in Hilversum. These modern architectural masterpieces (杰作) are so many that Hilversum almost feels like an open air museum. Dudok alone shaped most 20th century Hilversum and approximately 75 buildings in 1928-1931. It has wide international fame and is included in many architecture textbooks. The building has a remarkable shape and looks like a combination of “blocks”. Actually, one may start his journey of modern architecture by walking or biking the W.M. Dudok Architectural Route in Hilversum.‎ ‎59.Hilversum is different from most of the Netherlands in that ______.‎ A. it has a large population B. it is cut off from big cities C. it has many beautiful gardens D. it is in a hilly area with sandy soil ‎60.What was the greatest contribution of the Brenninkmeijers to Hilversum?‎ A. Building a railway link to Amsterdam B. Helping its textile industry to develop ‎ C. Constructing large villas for the poor D. Assisting its agricultural industry ‎61.The beginning of the media industry in Hilversum was marked by the establishment of ______.‎ A. a radio factory B. the medial capital C. a radio station D. a TV station ‎62.What is known about W.M. Dudok’s Hilversum Town Hall?‎ A. It consists of approximately 75 buildings B. It looks like an open air museum in the city C. It is a classic example in architecture textbooks D. It has shaped most of 20th century Hilvesum.‎ ‎【解析】‎ 试题分析:本文是一篇说明文。介绍了荷兰Hilversum这一地区的地貌特征、工业发展和现代建筑。‎ ‎61.A细节理解题。根据第三段首句“The change to a media economy started in 1920, … established a radio factory in Hilversum”可知Hilversum传媒业兴起的兴起是以一家无线电厂的建立为标志的。故选A项。‎ ‎62.C推理判断题。根据最后一段倒数第三、四句“His master piece… It has wide international fame and is included in many architecture textbooks”可知W. M. Dudok’s Hilversum Town Hall是建筑教科书中的一个典范。故选C项。‎ ‎【考点定位】说明文阅读 ‎【名师点睛】说明文用平实的语言客观地解释或探讨各种问题,如机器的制造过程、自然或社会现象产生的原因、工程项目的规划或问题的解决方案等,介绍自然科学、社会科学领域的最新成果,社会经济发展过程中出现的新生事物等。本文虽短短几段,却囊括了荷兰小镇Hilversum的前世今生,把一个不怎么出名的小城栩栩如生地呈现在中国读者的眼前。‎ ‎3.【2019·江苏】D Freedom and Responsibility ‎ Freedom’s challenge in the Digital Age is a serious topic. We are facing today a strange new world and we are all wondering what we are going to do with it.‎ ‎ Some 2,500 years ago Greece discovered freedom. Before that there was no freedom. There were great civilizations, splendid empires, but no freedom anywhere. Egypt and Babylon were both tyrannies, one very powerful man ruling over helpless masses.‎ ‎ In Greece, in Athens (雅典), a little city in a little country, there were no helpless masses. And Athenians willingly obeyed the written laws which they themselves passed, and the unwritten, which must be obeyed if free men live together. They must show each other kindness and pity and the many qualities without which life would be very painful unless one chose to live alone in the desert.The Athenians never thought that a man was free if he could do what he wanted. A man was free if he was self-controlled. To make yourself obey what you approved was freedom. They were saved from looking at their lives as their own private affair. Each one felt responsible for the welfare of Athens, not because it was forced on him from the outside, but because the city was his pride and his safety. The essential belief of the first free government in the world was liberty for all men who could control themselves and would take responsibility for the state.‎ ‎ But discovering freedom is not like discovering computers. It cannot be discovered once for all. If people do not prize it, and work for it, it will go. Constant watch is its price. Athens changed. It was a change that took place without being noticed though it was of the extreme importance, a spiritual change which affected the whole state. It had been the Athenian’ s pride and joy to give to their city. That they could get material benefits from her never entered their minds. There had to be a complete change of attitude before they could look at the city as an employer who paid her citizens for doing her work. Now instead of men giving to the state, the state was to give to them. What the people wanted was a government which would provide a comfortable life for them; and with this as the primary object, ideas of freedom and self-reliance and responsibility were neglected to the point of disappearing. Athens was more and more looked on as a cooperative business possessed of great wealth in which all citizens had a right to share.‎ ‎ Athens reached the point when the freedom she really wanted was freedom from responsibility.‎ ‎ There could be only one result. If men insisted on being free from the burden of self-dependence and responsibility for the common good, they would cease to be free. Responsibility is the price every man must pay for freedom. It is to be had on no other terms. Athens, the Athens of Ancient Greece, refused responsibility; she reached the end of freedom and was never to have it again.‎ ‎ But, “the excellent becomes the permanent”, Aristotle said. Athens lost freedom forever, but freedom was not lost forever for the world. A great American, James Madison, referred to: “The capacity (能力) of mankind for self-government.” No doubt he had not an idea that he was speaking Greek. Athens was not in the farthest background of his mind, but once man has a great and good idea, it is never completely lost. The Digital Age cannot destroy it. Somehow in this or that man’s thought such an idea lives though unconsidered by the world of action. One can never be sure that it is not on the point of breaking out into action only sure that it will do so sometime.‎ ‎65. What does the underlined word “tyrannies” in Paragraph 2 refer to?‎ A. Countries where their people need help. B. Powerful states with higher civilization.‎ C. Splendid empires where people enjoy freedom. D. Governments ruled with absolute power.‎ ‎66.People believing in freedom are those who________ .‎ A. regard their life as their own business B. seek gains as their primary object C. behave within the laws and value systems D. treat others with kindness and pity ‎67.What change in attitude took place in Athens?‎ A. The Athenians refused to take their responsibility.‎ B. The Athenians no longer took pride in the city.‎ C. The Athenians benefited spiritually from the government.‎ D. The Athenians looked on the government as a business.‎ ‎68.What does the sentence “There could be only one result.” in Paragraph 5 mean?‎ A. Athens would continue to be free.‎ B. Athens would cease to have freedom.‎ C. Freedom would come from responsibility.‎ D. Freedom would stop Athens from self-dependence.‎ ‎69.Why does the author refer to Aristotle and Madison?‎ A. The author is hopeful about freedom.‎ B. The author is cautious about self-government.‎ C. The author is skeptical of Greek civilization.‎ D. The author is proud of man’s capacity.‎ ‎70. What is the author’s understanding of freedom?‎ A. Freedom can be more popular in the digital age.‎ B. Freedom may come to an end in the digital age.‎ C. Freedom should have priority over responsibility.‎ D. Freedom needs to be guaranteed by responsibility.‎ ‎【解析】‎ 试题分析:这是一篇历史文化类说明文。文章讲述了自由的起源、演变和它在古代文化下的形态和缺陷,以及数字化世界给自由带来的挑战。‎ ‎65. D词义猜测题。根据划线词所在句子后半句“one very powerful man ruling over helpless masses”可知tyrannies是指拥有绝对统治权的政府。故选D项。‎ ‎66.C细节理解题。根据第三段内容可知,雅典人之所以能自由地生活在一起,是因为他们能够自愿在法律规定和价值体系下下从事一起活动。故选C项。‎ ‎69.A细节理解题。根据最后一段第二句“Athens lost freedom forever, but freedom was not lost forever for the world.”及“Athens was not in the farthest background of his mind, but once man has a great and good idea, it is never completely lost. The Digital Age cannot destroy it.”可知作者并没有因此而失去信心,相反对自由充满了希望。故选A项。‎ ‎70.D观点态度题。作者通过雅典人对自由态度变化的对比,得出自由需要责任的保证这一结论,故选D项。‎ ‎【考点定位】历史文化类说明文阅读 ‎【名师点睛】 英语阅读属于跨文化交际活动,除语言因素外,它还受到中西文化差异的影响。考生既要扎实书本知识,也要广泛阅读,不断提高自己的知识广度,有意识地多了解文化背景知识,多关注社会热点,这样才能做到游刃有余。‎ ‎4.【2019·广东】D It was once common to regard Britain as a society with class distinction. Each class had unique characteristics.‎ In recent years, many writers have begun to speak the 'decline of class' and 'classless society' in Britain. And in modern day consumer society everyone is considered to be middle class. ‎ But pronouncing the death of class is too early. A recent wide-ranging society of public opinion ‎ found 90 percent of people still placing themselves in particular class; 73 percent agreed that class was still a vital part of British society; and 52 percent thought there were still sharp class differences. Thus, class may not be culturally and politically obvious, yet it remains an important part of British society. Britain seems to have a love of stratification.‎ One unchanging aspect of a British person's class position is accent. The words a person speaks tell her or his class. A study of British accents during 1970s found that a voice sounding like a BBC newsreader was viewed as the most attractive voice, Most people said this accent sounded 'educated' and 'soft'. The accents placed at the bottom in this study, on the other hand, were regional(地区的)city accents. These accents were seen as 'common' and 'ugly'. However, a similar study of British accents in the US turned these results upside down and placed some regional accents as the most attractive and BBC English as the least. This suggests that British attitudes towards accent have deep roots and are based on class prejudice.‎ In recent years, however, young upper middle-class people in London, have begun to adopt some regional accents, in order to hide their class origins. This is an indication of class becoming unnoticed. However, the 2019 pop song 'Common People' puts forward the view that though a middle-class person may 'want to live like common people' they can never appreciate the reality of a working-class life.‎ ‎41. A recent study of public opinion shows that in modern Britain ________. ‎ A. it is time to end class distinction B. most people belong to middle class C. it is easy to recognize a person’s class D. people regard themselves socially different ‎42. The word stratification in Paragraph 3 is closest in meaning to ________.‎ A. variety B. most people belong to middle class C. authority ‎ D. qualification ‎43. The study in the US showed that BBC English was regarded as _________.‎ A. regional B. educated C. prejudiced ‎ D. unattractive ‎44. British attitudes towards accent _________.‎ A. have a long tradition B. are based on regional status C. are shared by the Americans D. have changed in recent years ‎45. What is the main idea of the passage?‎ A. The middle class is expanding ‎ B. A person’s accent reflects his class C. Class is a key part of British society D. Each class has unique characteristics.‎ ‎【解析】‎ 试题分析:这篇短文给我们讲述了在英国社会中,阶级划分是重要的一部分。作者在短文中给我们介绍了两项调查,通过调查的结果我们可以看到,英国社会里的阶级划分没有消失,它是英国社会重要的一部分。‎ ‎41.D细节理解题。根据短文第三段的内容可知,关于“阶级消亡”的说法,在英国对大众进行了一个调查,结果发现90%的人们仍然把自己划分在一定的阶层中;73%的人认为阶级是英国社会一个必不可少的部分。由此可知人们仍然认为他们在社会中是有区别的,故选D。‎ ‎42.B词义猜测题。根据短文第三段的内容可知,根据一项关于英国社会阶级是否开始消亡的调查可知,绝大多数人仍然认为英国社会中存在不同的阶级,这是英国社会中重要的一部分。由此可知,英国人好像对阶级划分非常热衷。stratification划分,跟B选项是同义词,故选B。‎ ‎43.D细节理解题。根据第四段中However, a similar study of British accents in the US turned these results upside down and placed some regional accents as the most attractive and BBC English as the least.可知,美国关于英语口音的调查结果正好和英国的调查结果相反,他们认为一些地方口音是最吸引人的,而BBC的英语是最不吸引人的,故选D。‎ ‎44.D推理判断题。根据短文的最后一段的内容可知,In recent years, however, young upper middle-class people in London, have begun to adopt some regional accents,可见,现在英国人们对待口音的态度开始改变了。故选D。‎ ‎【考点定位】文化类短文阅读。‎ ‎【名师点睛】这篇短文介绍了英国社会中存在的阶级划分这一现象,通过调查数据,论证这一话题。短文属于中上难度,主要考查学生的语篇理解的能力,以及词义猜测、推理判断及概括主旨大意的能力。学生们在平时的英语学习中,要具备对一些说英语国家的文化背景知识,这对他们做题是有好处的。‎ ‎5.【2019·四川】C ‎ Across Britain, burnt toast will be served to mothers in bed this morning as older sons and daughters rush to deliver their supermarket bunches of flowers, But, according to a new study, we should be placing a higher value on motherhood all year.‎ ‎ Mothers have long known that their home workload was just as heavy as paid work. Now, the new study has shown that if they were paid for their parental labours, they would earn as much as$172,000 a year.‎ ‎ The study looked at the range of jobs mothers do, as well as the hours they are working, to determine the figure. This would make their yearly income £30,000 more than the Prime Minister earns.‎ ‎ By analysing the numbers, it found the average mother works 119 hours a week,40 of which would usually be paid at a standard rate and 79 hours as overtime. After questioning 1,000 mothers with children under 18,it found that ,on most days, mums started their routine work at 7am and finished at around 11pm.‎ ‎ To calculate just how much mothers would earn from that labour, it suggested some of the roles that mums could take on, including housekeeper, part-time lawyer, personal trainer and entertainer. Being a part-time lawyer, at £48.98 an hour, would prove to be the most profitable of the “mum jobs”,with psychologist(心理学家)a close second.‎ It also asked mothers about the challenges they face, with 80 percent making emotional(情感的) demand as the hardest thing about motherhood.‎ Over a third of mums felt they needed more training and around half said they missed going out with friends.‎ The study shows mothers matter all year long and not just on Mother’s Day. The emotional ,physical and mental energy mothers devote to their children can be never-ending, but children are also sources of great joy and happiness. Investing(投入)in time for parenting and raising relationships is money well spent.‎ ‎38.How much would a mother earn a year if working as the Prime Minister?‎ ‎ A.£30,000. B.£142,000.‎ ‎ C.£172,000. D.£202,000.‎ ‎39.The biggest challenge for most mothers is from .‎ ‎ A. emotional demand B. low pay for work ‎ C. heavy workload D. lack of training ‎40.What is stressed in the last paragraph?‎ ‎ A. Mothers’ importance shows in family all year long.‎ ‎ B. The sacrifices mothers make are huge but worthwhile.‎ ‎ C. Mothers’ devotion to children can hardly be calculated.‎ ‎ D. Investing time in parenting would bring a financial return.‎ ‎41.What can we conclude from the study?‎ ‎ A. Mothers’ working hours should be largely reduced.‎ ‎ B. Mothers should balance their time for work and rest.‎ ‎ C. Mothers’ labour is of a higher value than it is realised.‎ ‎ D .Mothers should be freed from housework for social life.‎ ‎【解析】:这是一篇写在母亲节的文章,在母亲节这一天,每个人都会对母亲做出一些事,比如一束花等等,但是作者认为,母亲这一职业值得全年的关注,因为它包含太多。‎ ‎38.B细节理解题。根据文章的they would earn as much as$172,000 a year.以及This would make their yearly income £3000 more than the Prime Minister earns.可知,总理的工资是172019-30000=142019。故选择B。‎ ‎39.A 细节理解题。根据文章倒数第三段的It also asked mothers about the challenges they face, with 80 percent making emotional(情感的) demand as the hardest thing about motherhood.可知,选择A。‎ ‎40.B 细节理解题。根据最后一段的The emotional ,physical and mental energy mothers devote to their children can be never-ending, but children are also sources of great joy and happiness.可知,选择B,指母亲付出的牺牲是巨大的,但是又是值得的。‎ ‎41.C 推理判断题。根据文章第一段的But, according to a new study, we should be placing a higher value on motherhood all year.可知,作者认为母亲的重要性应该得到更多的关注,故选择C。‎ ‎【考点定位】 考查说明文阅读 ‎【名师点睛】本篇是一篇说明文,说明了母亲这一职业的伟大。阅读说明文的重点在于读懂它说明的事理。说明文阅读中要求考生要特别注意一些数字等,如第38题。除此之外,还需要抓住关键词、中心词,迅速在文中进行定位,解决细节题,如39、40题。最后,对文章的内容要了解,通常主题在第一段最后出现,所以考生要善于把握主题。‎ ‎6.【2019·天津】D ‎ Once when I was facing a decision that involved high risk, I went to a friend. He looked at me for a moment, and then wrote a sentence containing the best advice I’ve ever had: Be bold and brave — and mighty (强大的) forces will come to your aid.‎ Those words made me see clearly that when I had fallen short in the past, it was seldom because I had tried and failed. It was usually because I had let fear of failure stop me from trying at all. On the other hand, whenever I had plunged into deep water, forced by courage or circumstance, I had always been able to swim until I got my feet on the ground again.‎ Boldness means a decision to bite off more than you can eat. And there is nothing mysterious about the mighty forces. They are potential powers we possess: energy, skill, sound judgment, creative ideas — even physical strength greater than most of us realize.‎ ‎ Admittedly, those mighty forces are spiritual ones. But they are more important than physical ones. A college classmate of mine, Tim, was an excellent football player, even though he weighed much less than the average player. “In one game I suddenly found myself confronting a huge player, who had nothing but me between him and our goal line,” said Tim. “I was so frightened that I closed my eyes and desperately threw myself at that guy like a bullet(子弹) — and stopped him cold.”‎ ‎ Boldness — a willingness to extend yourself to the extreme—is not one that can be acquired overnight. But it can be taught to children and developed in adults. Confidence builds up. Surely, there will be setbacks (挫折) and disappointments in life; boldness in itself is no guarantee of success. But the person who tries to do something and fails is a lot better off than the person who tries to do nothing and succeeds.‎ ‎ So, always try to live a little bit beyond your abilities—and you’ll find your abilities are greater than you ever dreamed.‎ ‎51. Why was the author sometimes unable to reach his goal in the past?‎ A. He faced huge risks.‎ B. He lacked mighty forces.‎ C. Fear prevented him from trying.‎ D. Failure blocked his way to success.‎ ‎52. What is the implied meaning of the underlined part?‎ A. Swallow more than you can digest.‎ B. Act slightly above your abilities.‎ C. Develop more mysterious powers.‎ D. Learn to make creative decisions.‎ ‎53. What was especially important for Tim’s successful defense in the football game?‎ A. His physical strength. B. His basic skill.‎ C. His real fear. D. His spiritual force.‎ ‎54. What can be learned from Paragraph 5?‎ ‎ A. Confidence grows more rapidly in adults.‎ B. Trying without success is meaningless.‎ C. Repeated failure creates a better life.‎ D. Boldness can be gained little by little.‎ ‎55. What is the author’s purpose in writing this passage?‎ A. To encourage people to be courageous.‎ B. To advise people to build up physical power.‎ C. To tell people the ways to guarantee success.‎ D. To recommend people to develop more abilities.‎ ‎【解析】‎ 试题分析:文章大意:本文是一篇议论文。作者联系自己的经历说明了大胆的重要性,并鼓励人们有勇气和胆量。 ‎ ‎53. D细节理解题。根据第四段首句中Admittedly, those mighty forces are spiritual ones.可知在于他的精神力量。选D。‎ ‎54. D 段落大意题。根据第五段首句Boldness — a willingness to extend yourself to the extreme—is not one that can be acquired overnight.可以知道大胆是一点点获得的。故选D。‎ ‎55. A 推理判断题。根据全文内容可知作者联系自己的经历说明了大胆的重要性,并鼓励人们有勇气和胆量。故选A。‎ ‎【考点定位】议论类短文阅读 ‎ ‎【名师点睛】本文是一篇夹叙夹议的议论文。整体难度中等偏上,考生阅读此文首先需要找出文章的论点,然后找出作者任何用论据来支撑其论点的。52题词句猜测题需要联系上下文及前后句来判断,不能从字面含义判断;54题需要抓住本段的中心句才能找到段落大意;55题作者写此文的目的需要从文章整体把握才可以选择正确答案。‎ ‎7.【2019·浙江】A From the very beginning of school we make books and reading a constant source of possible failure and public humiliation. When children are little we make them read aloud, before the teacher and other children, so that we can be sure they “know” all the words they are reading. This means that when they don’t know a word, they are going to make a mistake, right in front of everyone. After having taught fifth-grade classes for four years, I decided to try at all costs to rid them of their fear and dislike of books, and to get them to read oftener and more adventurously.‎ One day soon after school had started, I said to them, “Now I’m going to say something about reading that you have probably never heard a teacher say before. I would like you to read a lot of books this year, but I want you to read them only for pleasure. I am not going to ask you questions to find out whether you understand the books or not. If you understand enough of a book to enjoy it and want to go on reading it, that’s enough for me. Also I’m not going to ask you what words mean. The children sat stunned and silent. Was this a teacher talking? One girl, who had just come to us from a school where she had had a very hard time, looked at me steadily for a long time after I had finished. Then, still looking at me, she said slowly and seriously, Mr Holt, do you really mean that?” I said just as seriously, “I mean every word of it.‎ During the spring she really astonished me. One day, she was reading at her desk, From a glimpse of the illustrations I thought I knew what the book was. I said to myself, “It can’t be,” and went to take a closer look. Sure enough, she was reading Moby Dick , in edition with woodcuts. I said, “Don’t you find parts of it rather heavy going?” She answered, Oh, sure, but I just skip over those parts and go on to the next good part. “‎ This is exactly what reading should be and in school so seldom is, an exciting, joyous adventure. Find something, dive into it, take the good parts, skip the bad parts, get what you can out of it, go on to something else. How different is our mean-spirited, picky insistence that every child get every last little scrap of “understanding” that can be dug out of a book.‎ ‎41. According to the passage, children’s fear and dislike of books may result from________.‎ A. reading little and thinking little ‎ B. reading often and adventurously C. being made to read too much D. being made to read aloud before others ‎42. The teacher told his students to read______ .‎ A. for enjoyment B. for knowledge C. for a larger vocabulary D. for higher scores in exams ‎43. Upon hearing the teacher’s talk, the children probably felt that________.‎ A. it sounded stupid B. it was not surprising at all C. it sounded too good to be true D. it was no different from other teachers' talk ‎44. Which of the following statements about the girl is TRUE according to the passage?‎ A. She skipped over those easy parts while reading.‎ B. She had a hard time finishing the required reading tasks.‎ C. She learned to appreciate some parts of the difficult books.‎ D. She turned out to be a top student after coming to this school.‎ ‎45. From the teacher's point of view,_________ .‎ A. children cannot tell good parts from bad parts while reading B. children should be left to decide what to read and how to read C. reading is never a pleasant and inspiring experience in school D. reading involves understanding every little piece of information ‎【解析】‎ 试题分析:作者认为阅读应该是一种快乐,而不是强迫性的阅读,强调通过快乐阅读培养阅读习惯。‎ ‎41. D细节理解题 题目的意思是根据文段孩子们害怕或者是不喜欢读书是因____。根据第一段第四行“This means that when they don’t know a word, they are going to make a mistake, right in front of everyone”. 意为这意味着当他们不知道单词的时候,他们会犯错,而且是在所有的人面前犯错,这会给他们带来一些害怕。‎ ‎42. A 细节理解题 根据第二段作者的话“Now I’m going to say something about reading that you have probably never heard a teacher say before. I would like you to read a lot of books this year, but I want you to read them only for pleasure. I am not going to ask you questions to find out ‎ whether you understand the books or not. If you understand enough of a book to enjoy it and want to go on reading it, that’s enough for me. Also I’m not going to ask you what words mean.”“,but I want you to read them only for pleasure.” 意为“现在我要说一些关于阅读的事情,你可能从来没有听说过一个老师说。我希望你读很多书,但今年,我想让你读书只为乐趣。我不想问你问题,是否理解文章。如果你足够理解一个喜欢这本书,想去读它,这对我来说已经足够了。我也不想问你单词的意思。” 在这里enjoyment,enjoy和pleasure是同义词。所以答案选A ‎44. C细节理解题这种题目是判断正误题,答题时一定要细心选择是错误还是正确的。此题选择的是正确的,也就意味着有3个是错误,只有一个是正确的。根据第四段最后一句话“She answered, Oh, sure, but I just skip over those parts and go on to the next good part. “她回答说,哦,当然,但我只是跳过这些部分,继续读下一个好的部分。“可以得出答案是这个女孩知道如何欣赏这些文章。‎ ‎45. B细节理解题 四个选项的意思分别是A孩子在阅读的时候不能够区别好的部分以及不好的部分;B应该留给学生孩子自己去选择,读什么以及如何读;C在学校阅读永远不会是一个愉快和启发性的体验;D阅读需要理解每一部分的信息。其实根据常识可以判断C和D是错误的,然后再结合全文大意,作者认为孩子不应该是强迫去读,要让他们自己读,选择材料读。‎ ‎【考点定位】 这是一篇记叙文。‎ ‎【名师点睛】 记叙文的文章相对而言是比较容易做的,细节题占多数,但需要的是一个细心。答案都是在文章中可以找到,必须一个一个选项匹配排除,最终才能够得出最佳答案。当遇到答案比较接近的时候,尤其需要细心,比如说41题的CD选项,两者都用到了being made to read。其实这种表达也间接说明了答案就是在两个中选择一个。‎ ‎8.【2019·浙江】D ‎ In 2019 ,when my daughter Becky was ten , she and my husband ,Joe, were united in their desire for a dog . As for me , I shared none of their canine lust.‎ ‎ But why , they pleaded. “Because I don’t have time to take care of a dog.” But we’ll do it. ” Really? You’re going to walk the dog? Feed the dog? Bathe the dog?” Yes, yes , and yes .”I don’t believe you .” We will . We promise.‎ ‎ They didn’t . From day two (everyone wanted to walk the cute puppy that first day ) , neither thought to walk the dog . While I was slow to accept that I would be the one to keep track of her shots , to schedule her vet appointments , to feed and clean her , Misty knew this on day one . As she looked up at the three new humans in her life (small, medium, and large) , she calculated ,”The medium one is the sucker in the pack .”‎ ‎ Quickly, she and I developed something very similar to a Vulcan mind meld (心灵融合) . She’d look at me with those sad brown eyes of hers , beam her need , and then wait , trusting I would understand — which , strangely , I almost always did . In no time , she became my feet as I read , and splaying across my stomach as I watched television .‎ ‎ Even so , part of me continued to resent walking duty . Joe and Becky had promised. Not fair , I’d balk (不心甘情愿地做) silently as she and I walked . “Not fair , ” I’ d loudly remind anyone within earshot upon our return home .‎ ‎ Then one day — January 1, 2019 , to be exact — my husband ‘ s doctor uttered an unthinkable word : leukemia ( 白血病) .With that , I spent eight to ten hours a day with Joe in the hospital , doing anyXXXthing and everything I could to ease his discomfort. During those six months of hospitalizations, Becky, 12 at the time, adjusted to other adults being in the house when she returned from school. My work colleagues adjusted to my taking off at a moment's notice for medical emergencies. Every part of my life changed; no part of my old routine remained.‎ ‎ Save one: Misty still needed walking. At the beginning, when friends offered to take her through her paces, I declined because I knew they had their own households to deal with.‎ ‎ As the months went by,I began to realize that I actually wanted to walk Misty. The walk in the morning before I headed to the hospital was a quiet, peaceful time to gather my thoughts or to just be before the day's medical drama unfolded. The evening walk was a time to shake off the day's upsets and let the worry tracks in my head go to white noise.‎ ‎ When serious illness visits your household, it's , not just your daily routine and your assumptions about the future that are no longer familiar. Pretty much everyone you acts differently.‎ ‎ Not Misty. Take her for a walk, and she had no interest in Joe's blood counts or ’one marrow test results. On the street or in the park, she had only one thing on her mind: squirrels! She Was so joyous that even on the worst days, she could make me smile. On a daily basis she reminded me that life goes on.‎ ‎ After Joe died in 2009,Misty slept on his pillow.‎ I'm grateful一to a point. The truth is, after years of balking, I've come to enjoy m’ walks with Misty. As I watch her chase after a squirrel, throwing her whole being into the here-and-now of an exercise that has never once ended in victory, she reminds me, too, that no matter how harsh ‎ the present or unpredictable the future , there's almost always some measure of joy to be extracted from the moment.‎ ‎55. why didn't the writer agree to raise a dog at the beginning of the story?‎ A. She was afraid the dog would get the family, into trouble.‎ B. It would be her business to take care of the dog C. Her husband and daughter were united as one.‎ D. She didn't want to spoil he’ daughter.‎ ‎56. Which of the following is the closest in meaning to "The medium one is “he sucker in the pack.” (Paragraph 3)?‎ A. "The middle-aged person loves me most.”‎ B. ”The medium-sized woman is the hostess.”‎ C. "The man in the middle is the one who has the final say.”‎ D. "The woman is the kind and trustworthy one in the family.”‎ ‎57. It can be inferred from Paragraph 3 that_______.‎ A. Misty was quite clever B. Misty could solve math problems C. the writer was a slow learner D. no one walked Misty the first day ‎58. The story came to its turning point when________.‎ A. Joe died in 2009‎ B. Joe fell ill in 2019‎ C. the writer began to walk the dog D. the dog tired to please the writer ‎ ‎59. Why did the writer continue to walk Misty while Joe was in hospital?‎ ‎ A. Misty couldn’t live without her B. Her friends didn’t offer any help C. The walk provided her with spiritual comfort.‎ D. She didn't want Misty to ’others companion.‎ ‎60. What is the message the writer wants to convey in the passage?‎ ‎ A. One should learn to enjoy hard times.‎ ‎ B .A disaster can change everything in life.‎ ‎ C. Moments of joy suggest that there is still hope ahead.‎ ‎ D. People will change their attitude toward you when you are in difficulty.‎ ‎【解析】‎ 试题分析:这是一篇记叙文,讲述作者从遛狗的体验中悟出生活的哲理——尽管有时生活艰难、前途难测,但人生总有希望和快乐。‎ ‎55. B 细节理解题 根据第三段第二行“While I was slow to accept that I would be the one to keep track of her shots, to schedule her vet appointments, to feed and clean her“作者慢慢地意识到她要去接父女俩的手尾,比如说安排见宠物医生等。这意味着最终将由她来照顾这一只狗。‎ ‎56. D 猜词题,根据这3个单词的意思不难知道small指的是作者的女儿, medium指的是作者,一个woman; and large指的是作者的丈夫。根据后文他们有了心灵相通不难得出答案小狗认为这个妇女(也就是作者)是最信任的和最好心的。‎ ‎57. A 推理题,根据第三段“Misty knew this on day one它在第一天就知道“和 “she calculated它在计算”可以得出答案这只狗是非常聪明的。另外也可以根据意思排除选项BCD。B. Misty could solve math problems就算文章中出现了一个计算的词语,但也不能够得出能够解答数学题的结论,而且根据常识也可以把它排除。C. the writer was a slow learner这个内容没有提及,D. no one walked Misty the first day这个选项的答案,也原文“它在第一天就知道了”是不一致的。‎ ‎59. C 细节理解题,根据第八段第二行“quiet, peaceful time to gather my thoughts or to just be before the day’s medical drama unfolded安静,安静的时间来收集我的想法 “ ,所有的这一切都是精神上的一些安慰,所以答案选择The walk provided her with spiritual comfort. 这个散步给她带来了心灵的安慰。‎ ‎60. C 推理题,根据文章最后一句话no matter how harsh the present or unpredictable the future , there's almost always some measure of joy to be extracted from the moment.不管现在是多么困难或者将来是多么的难以预计,总是有一些提取快乐的方法。此题易错选A,虽然这句话是正确的。但是这篇文章并不是在说在困难时期享受快乐。‎ ‎【考点定位】记叙文 ‎【名师点睛】这种记叙文,一定要理清作者的思路。往往作者经历的事情是一波几折的,最初的想法和最后的想法是不一样的。比如说最初她不想养这只宠物狗,但是最后就是这一只狗陪伴了她,让她得到了一些体会。‎ ‎9.【2019·重庆】B In ancient Egypt, a shopkeeper discovered that he could attract customers to his shop simply ‎ by making changes to its environment. Modern businesses have been following his lead,with more tactics(策略).‎ One tactic involves where to display the goods. Foe example, stores place fruits and vegetables in the first section. They know that customers who buy the healthy food first will feel happy so that they will buy more junk food(垃圾食品)later in their trip. In department stores, section is generally next to the women’s cosmetics(化妆品) section:while the shop assistant is going back to find the right size shoe, bored customers are likely to wander over cosmetics they might want to try later. ‎ Besides, businesses seek to appeal to customers’ senses. Stores notice that the smell of baked goods encourages shopping, they make their own bread each morning and then fan the bread smell into the store throughout the day. Music sells goods, too. Researchers in Britain found that when French music was played, sales of French wine went up. ‎ When it comes to the selling of houses, businesses also use highly rewarding tactics. They find that customers make decision in the first few second upon walking in the door, and turn it into a business opportunity. A California builder designed the structure of its houses smartly. When entering the house, the customer would see the Pacific Ocean through the windows, and then the poll through an open stairway leading to the lower level. The instant view of water on both levels helped sell these $10 million houses.‎ ‎40. Why do stores usually display fruits and vegetables in the first section?‎ A. To save customers times.‎ B. To show they are high quality foods.‎ C. To help sell junk food.‎ D. To sell them at discount prices.‎ ‎41. According to Paragraph 3, which of the following encourages customers to buy?‎ A. Opening the store early in the morning.‎ B. Displaying British wines next to French ones.‎ C. Inviting customers to play music.‎ D. Filling the store with the smell of fresh bread.‎ ‎42. What is the California builder’s story intended to prove?‎ A. The house structure is a key factor customers consider.‎ B. The more costly the house is, the better it sells.‎ C. An ocean view is much to the customers’ taste.‎ D. A good first impression increases sales.‎ ‎43. What is the main purpose of the passage?‎ A. To explain how businesses turn people into their customers.‎ B. To introduces how businesses have grown from the past.‎ C. To report researches on customer behavior.‎ D. To show dishonest business practices.‎ ‎【解析】‎ 试题分析: 本文在解释为什么要水果放要店里的最显眼的位置,目的是促进后面的商业交易 ‎40.C 推断题。根据第二段提到they know that customers who buy the healthy food first will feel happy so that they will buy some more junk food later in their trip为了是能够在稍后卖出那些垃圾食品,故选C项。‎ ‎41.D 推断题。根据第三段提到Stores notice that the smell of baked goods encourages shopping ,so they make their own bread each morning and then fan the bread smell into the store throughout the day可知烤面包的味道能促进购物,因此他们让面包味充满着整个商店,故选D项。‎ ‎42.D 推断题。根据最后一段提到When entering the house,the customer would see the Pacific Ocean throught the windows, and the the pool throught an open stairway leading to the lower level. The instant of water on both levels helped sell these $10million houses当人们进入房间,看到窗外的太平洋和游泳池时,销售业绩会增长,故选D项。‎ ‎43.A 推断题。本文在解释为什么要水果放要店里的最显眼的位置,目的是促进后面的商业交易,故选A项。‎ ‎【考点定位】社会生活类阅读。‎ ‎【名师点睛】对于社会生活类阅读,大多数是细节理解或推断题,考生要把握题干的大意,准备的定位,来找到相关信息。注意文章中的所给的汉语注释,不要由于紧张而对于 while the shop assistant is going back to find the right size shoe, bored customers are likely to wander over cosmetics they might want to try later.这个词意有所忽视从而影响对于文章整体的理解。‎ ‎10.【2019·重庆】C Join the discussion…‎ LakeLander ·2 hours ago ‎ Today, a man talked very loud on his phone on a train between Malvern and Reading, making many passengers upset. I wonder how he would react if I were to read my newspaperoutloudonthetrain, Ihave never had the courage to do it, though.‎ Pak50 ··· ·57 minutes ago Why not give it a try? Perhaps you should take lessons on a ‎ musical instrument. The late musician Dennis Brian is said to have ‎ asked a fellow train passenger to turn off his radio. When his ‎ request was refused, he took out his French horn(号) and started ‎ to practice.‎ Angie O’Edema·42 minutes ago I don’t see how musical instruments can help improve manners in public. Don’t do to others what you wouldn’t like to be done to yourself. Once, a passenger next to me talked out loud on his mobile phone. I left my seat quietly, giving him some privacy to finish his conversation. He realized this and apologised to me. When his phone rang again later, he left his seat to answer it. You see, a bit of respect and cooperation can do the job better.‎ Taodas ·29 minutes ago I did read my newspaper out loud on a train, and it turned out well. The guy took it in good part, and we chatted happily all the way to Edinburgh.‎ Sophie 76 ·13minutes ago I have not tried reading my newspaper out loud on a train, but ,several years ago, I read some chapters from Harry Porter to my bored and noisy children. Several passengers seemed to appreciate what I did.‎ ‎44. The passenger made an apology to Angie O’Edema because____.‎ A. he offered his seat to someone else B. he spoke very loudly on his phone C. he refused to talk with Angie D. he ignored Angie’s request ‎45. Who once read a newspaper out loud on a train?‎ A. Pak50 B. Angie O’Edema C. Taodas D. Sophie76‎ ‎46. What is the discussion mainly about?‎ A. How to react to bad behavior. B. How to kill time on a train.‎ C. How to chat with strangers. D. How to make a phone call.‎ ‎47. Where is the passage most probably taken from?‎ A. A webpage. B. A newspaper. C. A novel. D. A report.‎ ‎【解析】‎ 试题分析: 主要讲述了Lake Lander在论坛上描述了自己遇到的不文明现象后,众人发表的关于自己经历的 不文明现象的描述和看法。‎ ‎44.B 推断题。根据第三段提到Once, a passenger next to me talked out loud on his mobile phone. I left my seat quietly, giving him some privacy to finish his conversation. He realized this and apologised to me. 可知因为他在火车上大声讲电话,并向我道歉,故选B项。‎ ‎【考点定位】社会现象类阅读。‎ ‎【名师点睛】本文是由论坛上截取的关于不文明现象的讨论,难度中等,注意题干的提问方式,找到问题的切入点,很容易得出答案。在第44题中题干中运用made an apology to而文章中运用的是apologised to,所以注意平时词汇的积累,会让这类的题目更有目标性; 在47题中提到文章的来源,根据段落的设计格式,显示的时间和各自所而对的问题,来推断出来自网络论坛,所以注意扩大阅读的范围,多加运用网络而不是局限于文体。‎ ‎11.【2019·重庆】E The values of artistic works, according to cultural relativism(相对主义), are simply reflections of local social and economic conditions. Such a view, however, fails to explain the ability of some works of art to excite the human mind across cultures and through centuries.‎ History has witnessed the endless productions of Shakespearean plays in every major language of the world. It is never rare to find that Mozart packs Japanese concert halls, as Japanese painter Hiroshige does Paris galleries, Unique works of this kind are different from today’s popular art, even if they began as works of popular art. They have set themselves apart in their timeless appeal and will probably be enjoyed for centuries into the future.‎ In a 1757 essay, the philosopher David Hume argued that because“the general principles of taste are uniform(不变的) in human nature,”the value of some works of art might be essentially permanent. He observed that Homer was still admired after two thousand years. Works of this type, he believed, spoke to deep and unvarying features of human nature and could continue to exist over centuries.‎ Now researchers are applying scientific methods to the study of the universality of art. For example, evolutionary psychology is being used by literary scholars to explain the long-lasting themes and plot devices in fiction. The structures of musical pieces are now open to experimental analysis as never before. Research findings seem to indicate that the creation by a great artist is as permanent an achievement as the discovery by a great scientist.‎ ‎52. According to the passage, what do we know about cultural relativism?‎ A. It introduces different cultural values.‎ B. It explains the history of artistic works.‎ C. It relates artistic values to local conditions.‎ D. It excites the human mind throughout the world.‎ ‎53. In Paragraph 2, the artists are mentioned in order to show that _____ .‎ A. great works of art can go beyond national boundaries B. history gives art works special appeal to set them apart C. popular arts are hardly distinguishable from great arts D. great artists are skilled at combining various cultures ‎54. According to Hume, some works of art can exist for centuries because_____ .‎ A .they are results of scientific study B. they establish some general principles of art C. they are created by the world’s greatest artists D. they appeal to unchanging features of human nature ‎55. Which of the following can best serve as the title of the passage?‎ A. Are Artistic Values Universal? B. Are Popular Arts Permanent?‎ C. Is Human Nature Uniform? D. Is Cultural Relativism Scientific?‎ ‎【解析】‎ 试题分析:‎ ‎ 本文是说明文,主要介绍了根据文化相对主义,艺术作品在永恒受欢迎的原因是人类的审美具有共同性,并且就此展开说明。‎ ‎52.C 推断题。根据第一段第一句话可知根据相对主义,艺术作品就是把艺术的价值与当地的社会经济条件结合在一块,故选C项。‎ ‎53.A 推断题。根据第二段提到第二行和第三行提到可知不会把莫扎特与日本的音乐进行比较,正如日本的画作与巴黎的画廊进行比较,艺术作品是没有国界的,故选A项。‎ ‎54.D 推断题。根据第三段第一句话提到可知Davide曾辩论到风格的原则上是没有变化的,所以一些艺术是可能会永生的,故选D项。‎ ‎55.A 主题标题。本文主要从相对论的角度来说,艺术的价值只能反应当时的社会经济,但作者提出不同的观点,故选A项。‎ ‎【考点定位】文化类阅读。‎ ‎【名师点睛】文章内容较为简单,集中考查细节题,对此类题型,考生可以首先从问题中找到关键词然后以此为线索,运用略读及查阅的技巧在文中迅速寻找这一细节找到后再把这一部分内容仔细阅读一遍,仔细比较所给选项与文中细节的细微区别。52.53.54这三道题分别属于第一段第二段和第三段这个递进的原则,所以定位段落就比较容易而最后一题属于主旨标题,把握整体,提取中心,所以这个阅读理解整体来说目标性比较强。 ‎ ‎12.【2019·安徽】C ‎ As Internet users become more dependent on the Internet to store information, are people remember less? If you know your computer will save information, why store it in your own personal memory, your brain? Experts are wondering if the Internet is changing what we remember and how.‎ ‎ In a recent study, Professor Betsy Sparrow conducted some experiments. She and her research team wanted to know the Internet is changing memory. In the first experiment, they gave people 40 unimportant facts to type into a computer. The first group of people understood that the computer would save the information. The second group understood that the computer would not save it. Later, the second group remembered the information better. People in the first group knew they could find the information again, so they did not try to remember it.‎ ‎ In another experiment, the researchers gave people facts to remember, and told them where to find the information on the Internet. The information was in a specific computer folder (文件夹). Surprisingly, people later remember the folder location (位置) better than the facts. When people use the Internet, they do not remember the information. Rather, they remember how to find it. This is called "transactive memory (交互记忆)".‎ ‎ According to Sparrow, we are not becoming people with poor memories as a result of the Internet. Instead, computer users are developing stronger transactive memories; that is, people are learning how to organize huge quantities of information so that they are able to access it at a later date. This doesn't mean we are becoming either more or less intelligent, but there is no doubt that the way we use memory is changing.‎ ‎64. The passage begins with two questions to ______. ‎ ‎ A. introduce the main topic B. show the author's altitude C. describe how to use the Interne. D. explain how to store information ‎65. What can we learn about the first experiment?‎ A. Sparrow's team typed the information into a computer. ‎ B. The two groups remembered the information equally well.‎ C. The first group did not try to remember the formation. ‎ D. The second group did not understand the information.‎ ‎66. In transactive memory, people ______.‎ A. keep the information in mind B. change the quantity of information ‎ C. organize information like a computer D. remember how to find the information ‎67. What is the effect of the Internet according to Sparrow's research? ‎ A. We are using memory differently. B. We are becoming more intelligent.‎ C. We have poorer memories than before. D. We need a better way to access information.‎ ‎【答案】‎ ‎64. A ‎65. C ‎66. D ‎67. A ‎【解析】‎ 试题分析:文章主要讲的是互联网给人类的记忆带来的影响。互联网使用者越来越依赖互联网储存信息,人们这样做是否会丧失记忆事物的能力呢?专家怀疑互联网可能正在改变人们的记忆内容和记忆的方式。‎ ‎64. A 写作意图题。根据第一段“Experts are wondering if the Internet is changing what we remember and how.”可知,作者使用两个问句是为了引出要讨论的话题。‎ ‎【考点定位】心理类短文阅读 ‎【名师点睛】一般来说,举例的目的是为了引出即将讨论的话题,可以从例子后面找到总结性的话语。“Experts are wondering if the Internet is changing what we remember and how.”就是举例的意图。后三道题都属于细节理解题,可以直接从文章找到提示性的语句;最后一道题有一定的难度,需要进行一些推理和排除。‎ ‎13.【2019·安徽】E ‎ Food serves as a form of communication in two fundamental ways. Sharing bread or other foods is a common human tradition that can promote unity and trust. Food can also have a specific meaning, and play a significant role in a family or culture's celebrations or traditions. The foods we eat—and when and how we eat them—are often unique to a particular culture or may even differ between rural (农村的) and urban areas within one country.‎ ‎ Sharing bread, whether during a special occasion (时刻) or at the family dinner table, is a common symbol of togetherness. Many cultures also celebrate birthdays and marriages with cakes that are cut and shared among the guests. Early forms of cake were simply a kind of bread, so this tradition hits its roots in the custom of sharing bread.‎ ‎ Food also plays an important role in many New Year celebrations. In the southern United States, pieces of corn bread represent blocks of gold for prosperity (兴旺) in the New Year. In Greece, people share a special cake called vasilopita. A coin is put into the cake, which signifies (预示) success in the New Year for the person who receives it.‎ ‎ Many cultures have ceremonies to celebrate the birth of a child, and food can play a significant role. In China, when a baby is one month old, families name and welcome their child in a celebration that includes giving red-colored eggs to guests. In many cultures, round foods such as grapes, bread, and moon cakes are eaten at welcome celebrations to represent family unity.‎ ‎ Nutrition is necessary for life, so it is not surprising that food is such an important part of different cultures around the world.‎ ‎72. According to the passage, sharing bread______.‎ A. indicates a lack of food B. can help to develop unity C. is a custom unique to rural areas D. has its roots in birthday celebrations ‎73. What does the coin in vasilopita signify for its receiver in the New Year?‎ A. Trust. B. Success. C. Health. D. Togetherness.‎ ‎74. The author explains the role of food in celebrations by______.‎ A. using examples B. making comparisons C. analyzing causes D. describing processes ‎75. What is the passage mainly about?‎ A. The custom of sharing food. B. The specific meaning of food.‎ C. The role of food in ceremonies. D. The importance of food in culture.‎ ‎【答案】‎ ‎72. B ‎73. B ‎74. A ‎75. D ‎【解析】‎ 试题分析:文章主要讲的是食物和文化的关系。不同的文化里包含着不同的饮食传统和庆祝方式。食物通过两种基本的方式在扮演着沟通的角色。‎ ‎72. B 细节理解题。根据第一段“Sharing bread or other foods is a common human tradition that can promote unity and trust.”可知,分享食物可以促进团结和建立信任。‎ ‎73. B 细节理解题。根据第三段“A coin is put into the cake, which signifies (预示) success in the New Year...”可知,硬币预示着来年的成功。‎ ‎74. A 细节理解题。根据倒数第二段“In China, when a baby is one month old...In many cultures, round foods”可知,作者主要使用了举例的方法来解释食物在庆祝中的角色。‎ ‎75. D 主旨大意题。根据第一段“and play a significant role in a family or culture's celebrations or traditions”和第四段“Many cultures have ceremonies to celebrate”及最后一段“food is such an important part of different cultures around the world.”可知,文章主要讲的是食物在不同文化中的作用。‎ ‎【考点定位】社会类短文阅读 ‎【名师点睛】本篇题目的设置主要考查考生对细节的理解能力和对文章大意的把握。做细节理解题,需要快速从文章中到找有用的信息。第四题是主旨大意题,考生容易误选A项,A项太片面,它只是文章前两段所讲的内容。‎ ‎14.【2019·湖南】A Forget Cyclists, Pedestrians are Real Danger We are having a debate about this topic. Here are some letters from our readers.‎ ‎ ■Yes, many cyclists behave dangerously. Many drivers are disrespectful of cyclists. But pedestrians are probably the worse offenders.‎ ‎ People of all ages happily walk along the pavement with eyes and hands glued to the mobile phone, quite unaware of what is going on around them. They may even do the same thing while crossing a road at a pedestrian crossing or elsewhere. The rest of us have to evade (避让) them or just stand still to wait for the unavoidable collision.‎ ‎ The real problem is that some pedestrians seem to be, at least for the moment, in worlds of their own that are, to them, much more important than the welfare of others.‎ ‎ ——Michael Horan ‎ ■I love the letter from Bob Brooks about cyclists (Viewpoints, May 29). I am afraid they seem to think they own the roads.‎ ‎ I was walking across Altrincham Road one morning when a cyclist went round me and on being asked what he was doing he shouted at me.‎ ‎ The government built a cycle lane on the road but it is hardly used.‎ ‎ The police do nothing. What a laugh they are!‎ ‎ The cyclists should all have to be made to use the cycle lanes and wear helmets, fluorescent (发荧光的) jacket and lights at night and in the morning they should pay some sort of tax and be fined for not wearing them.‎ ‎ ——Carol Harvey ‎ ■Cyclists jump on and off pavements (which are meant for pedestrians), ride at speed along the pavements, and think they have a special right to go through traffic lights when they are on red.‎ ‎ I was almost knocked down recently by a cyclist riding on the pavement when there was a cycle lane right next to him.‎ ‎ Other road users, including horse riders, manage to obey the rules so why not cyclists?‎ ‎ It's about time they had to be registered and insured, so when they do hit a pedestrian or a vehicle, or cause an accident, at least they can be treated and there might be an opportunity to claim.‎ ‎ ——JML ‎ Write to Viewpoints of the newspaper.‎ ‎56.Michael Horan wrote the letter mainly to show that _______.‎ ‎ A. drivers should be polite to cyclists ‎ B. road accidents can actually be avoided ‎ C. sine pedestrians are a threat to road safety ‎ D. walking while using phones hurts one's eyes ‎57.Carol Harvey suggests that cyclists should _______.‎ ‎ A. be provided with enough roads ‎ B. be asked to ride on their own lanes ‎ C. be made to pay less tax for cycling ‎ D. be fined for laughing at policemen ‎58.What is a complaint of JML? ‎ ‎ A. Very few drivers are insured.‎ ‎ B. Cyclists ride fast on pavements.‎ ‎ C. Pedestrians go through red traffic lights.‎ ‎ D. Horse riders disrespect other road users.‎ ‎59.The underlined word "they" in the third letter refers to ______.‎ ‎ A. accidents B. vehicles ‎ C. pedestrians D. cyclists ‎60.The three letters present viewpoints on _______.‎ ‎ A. real source of road danger ‎ B. ways to improve road facilities ‎ C. measures to punish road offences ‎ D. increased awareness of road rules ‎【答案】 ‎ ‎56.C ‎57.B ‎58.B ‎59.D ‎60.A ‎【解析】‎ 试题分析:本文围绕一个主题展开一场争论——谁才是马路安全隐患的威胁。‎ ‎56.C;细节理解题。根据第一段中的最后一句话:But pedestrians are probably the worse offenders.(但是,路人可能才是更严重的罪犯)可知Michael Horan的观点是,路人对道路安全存在威胁,故选C ‎ ‎57.B;细节理解题。根据第八段的开头:The cyclists should all have to be made to use the cycle lanes and wear helmets.可知Carol Harvey建议骑自行车的人应该在他们应该走的车道去骑。故选B ‎59.D;细节推理题。最后一段说到:It’s about time they had to be registered and insured, so when they do hit a pedestrian or a vehicle, or cause an accident, at least they can be tranced and there might be an opportunity to claim.JML希望对自行车拥有者登记,当他们撞到行人或者车辆、制造出车祸的时候,他们能够被追查到,所以They在此指的是cyclists。故选D ‎60.A;主旨大意提。根据文章的标题和所有读者的来信不难发现这篇文章主要是围绕到底谁(自行车或者行人)才是道路安全存在的隐患。故选A ‎【考点定位】社会现象类短文阅读。‎ ‎【名师点睛】本文考查社会现象类短文阅读,要求考生根据作者的细节描述掌握这一社会现象的起因、结果及影响,然后做题,进行归纳。这篇文章旨在给出了一个议题,让人们给出讨论,意见以及看法,要求学生能够通过字里行间的细节描写找出人们对这个议题想法,例如第一段的第二句:But pedestrians are probably the worse offenders.就给出Michael Horan的看法,因此方便了我们做出第56题,所以在做这种文章时一定得抓住表达作者或者他人态度或观点的句子。‎ ‎15.【2019·湖南】C ‎ Have your parents ever inspected your room to see if you cleaned it properly? Imagine having your entire houses, garage, and yard inspected at any time -- with no warning. Inspections were a regular part of lighthouse (灯塔) living, and a keeper's reputation depended on results. A few times each year, an inspector arrived to look over the entire light station. The inspections were supposed to be a surprise, but keeper sometimes had advance notice.‎ Once lighthouses had telephones, keepers would call each other to warn that the inspector ‎ was approaching. After boats began flying special flags nothing the inspector aboard, the keeper's family made it a game to see who could notice the boat first. As soon as someone spotted the boat, everyone would do last-minute tidying and change into fancy clothes. The keeper then scurried to put on his dress uniform and cap. Children of keepers remember inspectors wearing white gloves to run their fingers over door frames and windowsills looking for dust.‎ Despite the serious nature of inspections, they resulted in some funny moments. Betty Byrnes remembered when her mother did not have time to wash all the dishes before an inspection. At the time, people did not have dishwashers in their homes. In an effort to clean up quickly, Mrs. Byrnes tossed all the dishes into a big bread pan, covered them with a cloth and stuck them in the oven. If the inspector opened the oven door, it would look like bread was baking. he never did.‎ One day, Glenn Furst's mother put oil on the kitchen floor just before the inspector entered their house. Like floor wax, the oil made the floors shiny and helped protect the wood. This time, though, she used a little too much oil. When the inspector extended his hand to greet Glenn's mother, he slipped on the freshly oiled surface. "He came across that floor waving his arms like a young bird attempting its first flight," Glenn late wrote. After he steadied himself, he shook Glenn's mother's hand, and the inspection continued as though nothing had happened.‎ ‎66.What does Paragraph I tell us about the inspection at the light station?‎ ‎ A. It was carried out once a year.‎ ‎ B. It was often announced in advance.‎ ‎ C. It was important for the keeper's fame.‎ ‎ D. It was focused on the garage and yard.‎ ‎67.The family began making preparations immediately after ______.‎ ‎ A. one of the members saw the boat ‎ B. a warning call reached the lighthouse ‎ C. the keeper put on the dress uniform and cap ‎ D. the inspector flew special flags in the distance ‎68.Mrs. Byrnes put the dishes in the oven because this would ______.‎ ‎ A. result in some fun ‎ B. speed up washing them ‎ C. make her home look tidy ‎ D. be a demand from the inspector ‎ ‎69.If the inspector had opened the oven door, he would have seen _______. ‎ ‎ A. an empty pan ‎ B. many clean dishes ‎ C. pieces of baked bread ‎ D. a cloth covering something ‎70.The inspector waved his arms ______.‎ ‎ A. to try his best to keep steady ‎ B. to show his satisfaction with the floor ‎ C. to extend a warm greeting to Glenn's mother ‎ D. to express his intention to continue the inspection ‎【答案】 ‎ ‎66.C ‎67.A ‎68.C ‎69.D ‎70.A ‎【解析】‎ 试题分析:本文讲述了如何利用各种方法应对巡视员的突袭。‎ ‎66.C;细节理解题。根据第一段的倒数第四行和倒数第三行:and a keeper’s reputation depended on the results.(看守人的名声取决于结果的好坏)可知,故选C ‎67.A;细节理解题。根据第二段的第三句话:As soon as someone spotted the boat, everyone would do last-minute tidying and change into fancy clothes.可知在建庭成员中一旦有人发现船的到来,全家人就开始快速准备。故选A ‎68.C;细节理解题。根据倒数第二段的第二句话:Betty Byrnes remembered when her mother did not have time to wash all the dishes before an inspection. 可知没有时间在检查之前去洗所有的碗,所以Mrs.Byrnes将所有的碗都丢进了烤箱避免被inspector发现。故选C ‎69.D;细节理解题。根据倒数第二段的倒数第三行:Mrs.Byrnes toss all the dishes into a big bread pan, covered them with a cloth, and stuck them in the oven.可知当inspector打开烤箱的时候他只会看到一块盖着布的某物,故选D ‎【考点定位】社会生活类短文阅读。‎ ‎【名师点睛】社会生活类的文章相对而言是比较容易做的,细节题占多数,需要的是一个细心。注意事情发展的先后顺序和发展的情节,理清作者的思路,以此得出答案。有时可以找出原句,有时可能需要理解,根据前后文推断,进行一个一个选项匹配排除,最终得出最佳答案。 ‎ ‎16.【2019·新课标全国II】C More students than ever before are taking a gap year(间隔年) before going to university. It used to be the “year off” between school and university. The gap-year phenomenon originated(起源) with the months left over to Oxbridge applicants between entrance exams in November and the start of the next academic year.‎ This year, 25,310 students who have accepted places in higher education institutions have put off their entry until next year, according to statistics on university entrance provided by the University and College Admissions Service (UCAS).‎ That is a record 14.7% increase in the number of students taking a gap year. Tony Higgins from UCAS said that the statistics are good news for everyone in higher education. “Students who take a well-planned year out are more likely to be satisfied with, and complete, their chosen course. Students who take a gap year are often more mature and responsible,” he said.‎ But not everyone is happy. Owain James, the president of the National Union of Students (NUS), argued that the increase is evidence of student hardship – young people are being forced into earning money before finishing their education. “New students are now aware that they are likely to leave university up to£15,000 in debt. It is not surprising that more and more students are taking a gap year to earn money to support their study for the degree. NUS statistics show that over 40% of students are forced to work during term time and the figure increases to 90% during vacating periods,” he said.‎ ‎29. What do we learn about the gap year from the text?‎ A. It is flexible in length.‎ B. It is a time for relaxation C. It is increasingly popular D. It is required by universities ‎30. According to Tony Higgins. students taking a gap year ____.‎ A. are better prepared for college studies A. know a lot more about their future job B. are more likely to leave university in debt C. have a better chance to enter top universities ‎31. How does Owain James feel about the gap-year phenomenon?‎ A. He’s puzzled B. He’s worried C. He’s surprised D. He’s annoyed ‎32. What would most students do on their vacation according to NUS statistics?‎ A. Attend additional courses.‎ B. Make plans for the new term C. Earn money for their education D. Prepare for their graduate studies ‎【答案】‎ ‎29. C ‎30. A ‎31. B ‎32. C ‎【解析】‎ 试题分析:本文提到大学的学费高使得很多大学生在上大学学前就要先去挣钱然后再上大学学习。这样的好处是让学生更珍惜大学的时光,更成熟有责任感。但是也给人们带来了思考。‎ ‎29.C 推理判断题。由短文第一段More students than ever before are taking a gap year(间隔年) before going to university.可知,过间隔年学生比以前多了,第三段第一句话That is a record 14.7% increase in the number of students taking a gap year.可知体验间隔年数量上升了14.7%,说明越来越受到欢迎,因此选C。‎ ‎30.A 细节理解题。由人名定位到短文第三段。Students who take a well-planned year out are more likely to be satisfied with, and complete, their chosen course. Students who take a gap year are often more mature and responsible,” he said.第三段在说它的优点,有间隔年之后再去上大学,会让人更成熟有责任感,对大学的学习有帮助。故选A。‎ ‎31.B 细节理解题。根据文章最后一段But not everyone is happy. 然后提到Owain,Owain James, the president of the National Union of Students (NUS), argued that the increase is evidence of student ‎ hardship,这证明学生很难;…more and more students are taking a gap year to earn money to support their study for the degree.他说越来越多的学生会在gap year期间是要去挣大学的费用。故选B。‎ ‎32. C细节理解题。根据文章最后一句话可知:NUS statistics show that over 40% of students are forced to work during term time and the figure increases to 90% during vacating periods,” 数据表明,40%多的学生在学校兼职,90%的学生要在假期打工挣学费。‎ ‎【考点定位】社会社会类短文阅读 ‎ ‎【名师点睛】本题以大学生做兼职或专职去挣钱为话题,通过对相关数据的分析可知,大学学生的学费给他们的学习生活带来了深刻的影响。我们考生应多关注社会生活方面的文章,才能更好地理解文章,解决相关的问题。‎ ‎17.【2019·福建】A Food festivals around the world Stilton Cheese Rolling May Day is a traditional day for celebrations, but the 2,000 English villagers of Stilton must be the only people in the world who include checsc rolling in their annual plans. Teams of four,dressed in a variety of strange and funny clothes , roll a complete cheese along a 50-metre course. On the way, they must not kick or throw their cheese, or go into their competitors' lane. Competition is fierce and the chief prize is a complete Stilton cheese weighing about four kilos (disappointingly, but understandably the cheeses used in the race are wooden ones). All the competitors arc served with beer or port wine, the traditional accompaniment for Stilton cheese. Fiery Foods Festival—The Hottest Festival on Earth Every year more than 10,000 people head for the city of Albuquerque, New Mexico. They come from as far away as Australia, the Caribbean and China, but they all share a common addiction—food that is not just spicy ,but hot enough to make your mouth burn, your head spin and your eyes water. Their destination is the Fiery Eood and BBQ Festival which is held over a period of three clays every March. You might like to try a chocolate-covered habanero pepper—officially the hottest pepper in the world—or any one of the thousands of products that are on show. But one thing's for sure—if you don't like the feeling of a burning tongue, this festival isn't for you! La Tomatina—The World's Biggest Food Fight On the last Wednesday of every August, the Spanish town of Bunol hosts Ea Tomatina—the world's largest food fight. A week-long celebration leads up to an exciting tomato battle as the highlight ‎ of the week's events. The early morning sees the arrival of large trucks with tomatoes—official fight-starters get things going by casting tomatoes at the crowd.‎ The battle lasts little more than half an hour, in which time around 50,000 kilograms of tomatoes have been thrown at anyone or anything that moves, runs, or fights back. Then everyone heads down to the river to make friends again—and for a much-needed wash!‎ ‎56. In the Stilton cheese rolling competition, competitors on each team must .‎ A. wear various formal clothes B. roll a wooden cheese in their own lane C. kick or throw their cheese D. use a real cheese weighing about four kilos ‎57.Where is the Fiery Food and BBQ Festival held?‎ A. In New Mexico. B. In the Caribbean. C. In Australia. D. In China.‎ ‎58.The celebration of La Tomatina lasts .‎ A. three days B. seven days C. less than three days D. more than seven days ‎59.Which of the following is TRUE according to the passage?‎ A. The chief prize for the Stilton cheese rolling competition is beer or port wine.‎ B.More than 10,000 Chinese take pail in the Fiery Food and BBQ Festival.‎ C. Thousands of spicy foods are on show in the Fiery Food and BBQ Festival. D. An exciting tomato battle takes place at the beginning of La Tomatina.‎ ‎【解析】‎ 试题分析:本文是一篇说明文,分别讲述了英国、新墨西哥以及巴西的几个关于食物的传统节日。‎ ‎57.A细节理解题。根据第二段第一句Every year more than 10,000 people head for the city of Albuquerque, New Mexico.可以知道这是在新墨西哥发生的事情,从段落中第三句Their destination is the Fiery Eood and BBQ Festival可以知道人们到新墨西哥是为了参加Fiery food festival。故选A ‎58.B细节理解题。根据文章第三段第二句A week-long celebration leads up to an exciting tomato battle as the highlight of the week's events.可以知道,西红柿大战这个西班牙的节日的持续时间是一周,故选B ‎59.C细节理解题。根据文章第二段倒数第二个破折号后面的文字or any one of the thousands of products that are on show.可以知道你可以任意品尝展出的上千种的食物。故选C ‎【考点定位】说明文阅读。‎ ‎【名师点睛】说明文的文章相对而言是比较容易做的,细节题占多数,但需要的是一个细心。答案都是在文章中可以找到,必须一个一个选项匹配排除,最终才能够得出最佳答案。当遇到答案比较接近的时候,尤其需要细心,因为很有可能有些选项会有一些迷惑性。需要考生认真判断。‎ ‎18.【2019·福建】D Life can be so wonderful, full of adventure and joy. It can also be full of challenges, setbacks and heartbreaks. Whatever our circumstances, we generally still have dreams, hopes and desires—that little something more we want for ourselves and our loved ones. Yet knowing we can have more can also create a problem, because when we go to change the way we do things, up come the old patterns and pitfalls that stopped us from seeking what we wanted in the first place.‎ This tension between what we feel we can have and "what wre're seemingly able to have is the niggling suffering, the anxiety we feel. This is where we usually think it's easier to just give up. But we're never meant to let go of the part of us that knows we can have more. The intelligence behind that knowing is us—the real us. It's the part that believes in life and its possibilities. If you drop that, you begin to feel a little "dead" inside because you're dropping "you".‎ So, if we have this capability but somehow life seems to keep us stuck, how do we break these patterns? Decide on a new course and make one decision at a time. This is good advice for a new adventure or just getting through today's challenges.‎ While, deep down, we know we can do it, our mind—or the minds of those close to us—usually says we can't. That isn't a reason to stop, it's just the mind, that little man or woman on your shoulder, trying to talk you out of something again. It has done it many times before. It's all about starting simple and doing it now.‎ Decide and act before overthinking. When you do this you may feel a little, or large, release from the jail of your mind and you'll be on your way.‎ ‎68.It can be inferred from the first two paragraphs that we should .‎ A.slow down and live a simple life B.be careful when we choose to change C.stick to our dreams under any circumstances D.be content with what we already have ‎69.What is the key to breaking the old patterns? A. To focus on every detail. B. To decide and take immediate action. C. To listen to those close to us. D. To think twice before we act.‎ ‎70.Which of the following best explains the underlined part in the last paragraph? A. Escape from your punishment B. Realization of your dreams. C. Freedom from your tension. D. Reduction of your expectations.‎ ‎71.What does the author intend to tell us?‎ A.It's easier than we think to get what we want.‎ B. It's important to learn to accept sufferings in life.‎ C. It's impractical to change our way of thinking.‎ D. It's harder than we expect to follow a new course. ‎ ‎【解析】‎ 试题分析:本文是一篇说明文,作者介绍了在我们想要放弃的时候改变这种模式的一种方法就是重新选择目标并且迅速作出决定。‎ ‎68.C细节理解题。文章第一段第三句Whatever our circumstances, we generally still have dreams, hopes and desires说不管在什么情况下,我们仍然有着梦想、希望和欲望。以及第二段最后一句If you drop that, you begin to feel a little "dead" inside because you're dropping "you".如果你放弃了,你的内在会变得了无生机,因为你放下了你自己。故选C ‎69.B细节理解题。根据倒数第三段第一句Decide on a new course and make one decision at a time.可以知道为了打破旧格局需要决定一个新目标并且立刻做出决定。故选B ‎70.C分析推断题。根据划线句子可知其本意是逃出你内心的牢笼。结合上文中在过多思考之前快速决定和行动可以推测这个划线句子是指要从过多考虑的焦虑中解放自己。故选C ‎71.A分析推断题。从全文来看,作者主要是想要强调果断的行动而不是空想来实现自己的梦想和欲望,故选A.比起想要成功,行动更加容易。‎ ‎【考点定位】说明文阅读。‎ ‎【名师点睛】说明文 的文章更相对而言是比较容易做的,答案都是在文章中可以找到或者通过文中的线索分析出来,必须一个一个选项进行分析匹配排除,最终才能够得出最佳答案。结合上下文中的联系,只要掌握了每一段的大意,做题就不会太难。‎ ‎19.【2019·福建】E Group exercise is one of the most effective ways to improve physical fitness and sustain(保持) a healthy lifestyle. Group exercise is challenging, yet fun and empowering! Of course everyone knows that exercise is good for the body. However, studies have shown that when exercise is performed in groups, it's not only great for improving physical health but for psychological health. It's an opportunity to be social, release endorphins(内啡肽), and improve your strength. Additionally, group exercise creates a community feel and the shared common goal motivates participants to work hard. The instrumental support of taking on a fitness journey with others proves more effective than going to the gym alone. ‎ Another beneficial aspect of group exercise is the informational support participants receive from the instructor. Many people fear the gym because they feel lost and don't want to embarrass themselves. If you feel you can relate, then group training is an even better option for you. It's a great opportunity to learn more about fitness through the clear instruction and supervision(监管) of a fitness instructor. If you're tired of wandering around the gym wasting time and becoming bored, you can attend an upbeat group fitness class that'll keep your workout on track. Don't let fitness frighten you!‎ If you're serious about wanting to live a healthy lifestyle, it's extremely important to surround yourself with people who'll provide you with the proper emotional support. I wouldn't scold anyone for deciding to party on weekends and in turn I wouldn't expect anyone to give offence to me for focusing on my health. Surround yourself with people who uplift, encourage and understand you! Make fitness even more fun by trying something new or any group fitness class, with a friend. Plan to go for a jog together. Then try a fun healthy restaurant or fresh juice bar! Fitness can be both fun and social!‎ Surrounding yourself with people who'll provide you with respect support can be very beneficial while working towards reaching health and fitness goals. First, decide to do it for yourself and work towards staying positive. Then make sure the people you surround yourself with are supportive. Don't let negativity ruin your motivation.(360) 72. The first paragraph focuses on .‎ A. the greatest challenge of group exercise B. the most effective way to improve physical fitness C. the contribution of group exercise to psychological health D. the shared common goal in performing exercise in groups ‎73. The underlined word "upbeat" in the second paragraph probably means " ‎ ‎ A. cheerful B. average C. serious D. temporal*)'‎ ‎74. When it comes to emotional support, the author thinks it necessary ‎ A. to sustain a colorful lifestyle ‎ B. to party on weekends with positive people ‎ C. to try a fun healthy restaurant regularly ‎ D. to surround yourself with supportive people ‎75. What would be the best title for the passage?‎ ‎ A. Seeking Support B. Supporting Health ‎ C. Improving Your Strength D. Building Up Fitness ‎【解析】‎ 试题分析:本文是一篇说明文,主要是在介绍团体运动的好处。和其他人一起运动可以和他人交流从而获得支持,也能够得到一些无形的好处。‎ ‎72.C细节题。根据文章第一句Group exercise is one of the most effective ways to improve physical fitness and sustain a healthy lifestyle团体运动是一项最有效增加身体健康保持健康生活的的方式,故选C ‎74.D细节理解题。根据最后一段第一句Surrounding yourself with people who'll provide you with respect support can be very beneficial while working towards reaching health and fitness goals.在以健康为目标锻炼的时候和那些你能够给你尊敬和支持的人们在一起是很有益处的,故选D ‎75.B分析推断题。文章的主要内容是在说团体运动的好处,团体运动包含的好处主要是因为大家在一起运动,不仅生理上会有一些好处,而且团内其他成员会给你支持,教练也会给你们一些信息上的支持,故选B ‎【考点定位】说明文阅读。‎ ‎【名师点睛】说明文的文章更相对而言是比较容易做的,答案都是在文章中可以找到或者通过文中的线索分析出来,必须一个一个选项进行分析匹配排除,最终才能够得出最佳答案。结合上下文中的联系,只要掌握了每一段的大意,做题就不会太难。 ‎ ‎20.【2019·新课标全国I】B The freezing Northeast hasn’t been a terribly fun place to spend time this winter, so when ‎ the chance came for a weekend to Sarasota, Florida, my bags were packed before you could say “sunshine”. I left for the land of warmth and vitamin C(维生素C), thinking of beaches and orange trees. When we touched down to blue skies and warm air, I sent up a small prayer of gratefulness. Swimming pools, wine tasting, and pink sunsets(at normal evening hours, not 4 in the afternoon) filled the weekend, but the best part - particularly to my taste, dulled by months of cold- weather root vegetables- was a 7 a.m. adventure to the Sarasota farmers’ market that proved to be more than worth the early wake-up call.‎ ‎ The market, which was founded in 1979, sets up its tents every Saturday from 7:00 am to 1 p.m., rain or shine, along North Lemon and State streets. Baskets of perfect red strawberries; the red-painted sides of the Java Dawg coffee truck; and most of all, the tomatoes: amazing, large, soft and round red tomatoes.‎ Disappointed by many a broken, vine-ripened(蔓上成熟的) promise, I’ve refused to buy winter tomatoes for years. No matter how attractive they look in the store, once I get them home they’re unfailingly dry, hard, and tasteless. But I homed in, with uncertainty, on one particular table at the Brown’s Grove Farm’s stand, full of fresh and soft tomatoes the size of my fist. These were the real deal- and at that moment, I realized that the best part of Sarasota in winter was going to be eating things that back home in New York I wouldn’t be experiencing again for months.‎ ‎ Delighted as I was by the tomatoes in sight, my happiness deepened when I learned that Brown’s Grove Farm is one of the suppliers for Jack Dusty, a newly opened restaurant at the Sarasota Ritz Carlton, where - luckily for me - I was planning to have dinner that very night. Without even seeing the menu, I knew I’d be ordering every tomato on it.‎ ‎24. What did the author think of her winter life in New York?‎ A. Exciting. B. Boring. ‎ C. Relaxing. D. Annoying.‎ ‎25. What made the author’s getting up late early worthwhile?‎ A. Having a swim.‎ B. Breathing in fresh air.‎ C. Walking in the morning sun.‎ D. Visiting a local farmer’s market.‎ ‎26. What can we learn about tomatoes sold in New York in winter?‎ A. They are soft.‎ B. They look nice.‎ C. They taste great.‎ D. They are juicy.‎ ‎27. What was the author going to that evening?‎ A. Go to a farm. ‎ B. Check into a hotel.‎ C. Eat in a restaurant.‎ D. Buy fresh vegetables ‎【答案】‎ ‎24. B ‎ ‎25. D ‎ ‎26. B ‎ ‎27. C ‎ ‎【解析】‎ 试题分析:文章讲述了作者生活在冬天寒冷的纽约,当有一个到佛罗里达州萨拉索塔一个周的机会,作者在那里体验到完全不同的冬天。 ‎ ‎24. B推理判断题。根据文章第一段第一句The freezing Northeast hasn’t been a terribly fun place to spend time this winter,可以推测出作者在纽约冬天的生活是令人厌烦的。故选B。‎ ‎25. D 细节理解题。根据文章第一段最后一句dulled by months of cold- weather root vegetables- was a 7 a.m. adventure to the Sarasota farmers’ market that proved to be more than worth the early wake-up call.可知作者早上7点到农产品市场是很有意义的。可知选D。‎ ‎26. B 推理判断题。根据文章第三段第二句No matter how attractive they look in the store, once I get them home they’re unfailingly dry, hard, and tasteless.可知,纽约的西红柿在商店里看起来是很吸引人的。可知选B。‎ ‎27. C 推理判断题。根据文章最后一段尤其倒数第一二句可知,作者晚上计划在餐馆吃饭,并且点西红柿。故选C。‎ ‎【考点定位】记叙类短文阅读 ‎ ‎【名师点睛】‎ 本文是一篇记叙类短文,整体难度中等,难度较大的是24和26题,学生容易判断错误,需要通过一定的推理才能判断正确。学生首先要从倒数第二段最后一句判断出作者原来生活在纽约,才可理解第一段第一句描述的是纽约。第26题考生是不能根据作者前面描述的文字判断,真正答案要从后段文字描述去判断。这就要求考生做此类阅读时要注意前后之间的联系与对比,选择答案时切不可以以偏概全。‎ ‎21.【2019·上海】A Look to many of history’s cultural symbols, and there you’ll find an ancestor of Frosty, the snowman in the movie Frozen. It appeared on some of the first postcards, starred in some of the earliest silent movies, and was the subject of a couple of the earliest photos, dating all the way back to the 1800s. I discovered even more about one of humanity’s earliest forms of life art during several years of research around the world.‎ For example, snowmen were a phenomenon in the Middle Ages, built with great skill and thought. At a time of limited means of expression, snow was like free art supplies dropped from the sky. It was a popular activity for couples to leisurely walk through town to view the temporary works of chilly art. Some were created by famous artists, including a 19-year-old Michelangelo, who in 1494 was appointed by the ruler of Florence, Italy, to build a snowman in his mansion’s courtyard.‎ The Miracle of 1511 took place during six freezing works called the Winter of Death. The city of Brussels was covered in snowmen—an impressive scene that told stories on every street corner. Some were political in nature, criticizing the church and government. Some were a reflection of people’s imagination. For the people of Brussels, this was a defining moment of defining freedom. At least until spring arrived, by which time they were dealing with damaging floods.‎ If you fear the heyday of the snowman has passed, don’t worry: I’ve learned that some explosive snowman history is still being made today. Every year since 1818, the people of Zurich, Switzerland, celebrate the beginning of spring by blowing up a snowman. On the third Monday of April, the holiday Sechselauten is kicked off when a cotton snowman called the Boogg is stuffed with explosive and paraded through town by bakers and other tradesmen who throw bread to the crowds. The parade ends with the Boogg being placed on a 40-foot pile of firewood. After the bells of the Church of St. Peter have rung six times, representing the passing of winter, the pile is lit. When the snowman explodes, winter is considered officially over—the quicker it is burnt down, the longer summer is said to be.‎ ‎66. According to the passage, why did snowmen become a phenomenon in the Middle Ages?‎ A. People thought of snow as holy art supplies.‎ B. People longed to see masterpieces of snow.‎ C. Building snowmen was a way for people to express themselves.‎ D. Building snowmen helped people develop their skill and thought.‎ ‎67. “The heyday of the snowman” (paragraph 4) means the time when___________.‎ A. snowmen were made mainly by artists B. snowmen enjoyed great popularity C. snowmen were politically criticized D. snowmen caused damaging floods ‎68. In Zurich, the blowing up of the Boogg symbolizes__________________.‎ A. the start of the parade B. the coming of a longer summer C. the passing of the winter D. the success of tradesmen ‎69. What can be concluded about snowmen from the passage?‎ A. They were appreciated in history B. They have lost their value C. They were related to movies D. They vary in shape and size ‎【答案】‎ ‎66. C ‎67. B ‎68. C ‎69. A ‎【解析】‎ 试题分析:本文讲述历史的文化标志:雪人。‎ ‎67. B 细节理解题 根据黑体字后面的内容“has passed, don’t worry: I’ve learned that some explosive snowman history is still being made today. 已经过去了,不用担心:我了解到一些历史的雪人现在依然在制造。”这表明担心的是现在没有了这样的东西,换句话说就是现在不再流行,所以答案就是B。‎ ‎68. C 细节理解题 根据“celebrate the beginning of spring …. 庆祝春天的开始”, 可以知道答案应该是冬天的过去,也就是春天的开始。所以答案是C。‎ ‎69. A 推理题根据文章的第一句话look to many of history’s cultural symbols回顾许多的历史文化标志,另外整篇文章出现过很多的history,结合上下文可以知道答案是A (雪人在历史上是受欣赏的。)另外也可以根据排除法,排除其他选项。比如B选项,他们的价值现在依然存在,所以它的表达错误。C选项,雪人的确是与电影有关,但是这个答案不可以通过文章得出来。D选项在文章中没有提及这个内容。‎ ‎【考点定位】 这是一篇说明文 ‎【名师点睛】 细节题占多数,但需要的是一个细心。答案都是在文章中可以找到,而且往往答案都是在设问内容的附近。另外也可以通过排除法,一个一个选项匹配排除,最终得出最佳答案。当遇到答案比较接近的时候,尤其需要细心。‎ ‎2019年高考试题 社会生活类 ‎ ‎ [2019·安徽卷] ‎ C You are the collector in the gallery of your life. You collect. You might not mean to but you do. One out of three people collects tangible(有形的) things such as cats, photos and many toys.‎ These are among some 40 collections that are being shown at “The Museum Of”—the first of several new museums which, over the next two years, will exhibit the objects accumulated by unknown collectors. In doing so, they will promote a popular culture of museums, not what museums normally represent.‎ Some of the collections are fairly common—records, model houses. Others are strangely beautiful—branches that have fallen from trees, for example. But they all reveal(显露) a lot of things: ask someone what they collect and their answers will tell you who they are.‎ Others on the way include“The Museum of Collectors”and “The Museum of Me”. These new ones, it is hoped, will build on the success of “The Museum Of”. The thinkers behind the project want to explore why people collect, and what it means to do so. They hope that visitors who may not have considered themselves collectors will begin to see they, too, collect.‎ Some collectors say they started or stopped making collections at important points: the beginning or end of adolescence—“it's a growingup things; you stop when you grow up,” says one. Other painful times are mentioned, such as the end of a relationship. For time and life can seem so uncontrollable that a steady serial(顺序排列的) arrangement is comforting.‎ ‎64.How will the new museums promote a popular culture of museums?‎ A.By collecting more tangible things.‎ B.By showing what ordinary people have collected.‎ C.By correcting what museums normally represent.‎ D.By accumulating 40 collections two years from now.‎ ‎65.What can be learned about collectors from their collections?‎ A.Who they are.‎ B.How old they are.‎ C.Where they were born.‎ D.Why they might not mean to collect.‎ ‎66.Which of the following is an aim of the new museums?‎ A.To help people sell their collections.‎ B.To encourage more people to collect.‎ C.To study the significance of collecting.‎ D.To find out why people visit museums.‎ ‎67.According to the last paragraph, people may stop collecting when they ________.‎ A.become adults B.feel happy with life C.are ready for a relationship D.feel time to be uncontrollable ‎【文章大意】 本文为一篇说明文。每个人都是人生艺术走廊的收藏者。未来两年,The Museum Of将举办40多场无名收藏者的收藏展。‎ ‎64. B 细节理解题。第二段最后一句中的doing so指代的是上句的“…over the next two years, will exhibit the objects accumulated by unknown collectors.”故选B。‎ ‎65. A 细节理解题。根据第三段最后一句可知选A。‎ ‎66. C 细节理解题。根据第四段第三句可知选C。‎ ‎67. A 细节理解题。根据最后一段首句可知选A。‎ ‎[2019·安徽卷] ‎ D Should we allow modern buildings to be built next to older buildings in a historic area of ‎ a city? In order to answer this question, we must first examine whether people really want to preserve the historic feel of an area. Not all historical buildings are attractive. However, there may be other reasons—for example, economic(经济的) reasons—why they should be preserved. So, let us assume that historical buildings are both attractive and important to the majority of people. What should we do then if a new building is needed?‎ In my view, new architectural styles can exist perfectly well alongside an older style. Indeed, there are many examples in my own hometown of Tours where modern designs have been placed very successfully next to old buildings. As long as the building in question is pleasing and does not dominate(影响) its surroundings too much, it often improves the attractiveness of the area.‎ It is true that there are examples of new buildings which have spoilt(破坏) the area they are in, but the same can be said of some old buildings too. Yet people still speak against new buildings in historic areas. I think this is simply because people are naturally conservative(保守的) and do not like change.‎ Although we have to respect people's feelings as fellow users of the buildings, I believe that it is the duty of the architect and planner to move_things_forward. If we always reproduced what was there before, we would all still be living in caves. Thus, I would argue against copying previous architectural styles and choose something fresh and different, even though that might be the more risky choice.‎ ‎68.What does the author say about historical buildings in the first paragraph?‎ A.Some of them are not attractive.‎ B.Most of them are too expensive to preserve.‎ C.They are more pleasing than modern buildings.‎ D.They have nothing to do with the historic feel of an area.‎ ‎69.Which of the following is TRUE according to the author?‎ A.We should reproduce the same old buildings.‎ B.Buildings should not dominate their surroundings.‎ C.Some old buildings have spoilt the area they are in.‎ D.No one understands why people speak against new buildings.‎ ‎70.By “move things forward” in the last paragraph, the author probably means “________”.‎ A.destroy old buildings B.put things in a different place C.choose new architectural style D.respect people's feelings for historical buildings ‎71.What is the main purpose of the passage?‎ A.To explain why people dislike change.‎ B.To warn that we could end up living in caves.‎ C.To admit how new buildings have ruined their surroundings.‎ D.To argue that modern buildings can be built in historic areas.‎ ‎【文章大意】 本文为一篇议论文。保护古建筑与城市的发展是相互矛盾的,作者就如何解决这个矛盾谈了自己的看法。‎ ‎71. D 写作意图题。本文为议论文,作者就热门话题——古建筑保护与新建筑的建设是否矛盾发表了自己的看法。作者在第二段提出自己的观点“In my view, new architectural styles can exist perfectly well alongside an older style.” 接下来就此进行了论证。故选D。 ‎ ‎[2019·安徽卷] ‎ E You may not have heard of Ashoka, but for the past 27 years, this association,founded by Bill Drayton, has fought poverty(贫困) and sickness, promoted education and encouraged small businesses. To support these worthy causes, Ashoka provides money for the world's most promising“changemakers” seeking to solve(解决) urgent problems and would like to create a world in which every citizen is a changemaker.‎ Drayton believes that anyone can become an agent for change. The important thing is to simply give yourself permission. If you see a problem that you care about, you can help solve it. The young in particular are willing to accept this concept because at heart every child wants to grow into a happy, healthy contributing adult. In fact, it is many young people's ambition to set up programmes or businesses that improve social conditions. An excellent example is an Ashoka project started in 2019 in Dhaka,which handled the rubbish problem facing the city, helped local farmers and provided an income for poor people there.‎ When Masqsood and Iftekhar began to study the problem of all the uncollected rubbish that lay in Dhaka's streets, attracting rats and disease, they discovered that 80% of it was natural ‎ waste. So they educated the poor people in the city to compost(把……制成堆肥) this waste. They knew that they would have a market for the end product because local farmers were struggling with chemical fertilizers(化肥) which were expensive and had reduced the natural minerals in the soil over the years. At first, they were refused, but once they were able to persuade them that there was money to be made, the project took off. In 2009 sales were $14,000.‎ Drayton is optimistic that in ten years Ashoka will be making really serious, practical progress in bringing about social change by changing the way we look at economic development.‎ ‎72.Which of the following could be the best title for the passage?‎ A.Changemakers B.Businessmen C.Social conditions D.Rubbish problems ‎73.The underlined word “them” in Paragraph 3 probably refers to “________”.‎ A.the local farmers B.Masqsood and Iftekhar C.Drayton and his team D.the poor people in Dhaka ‎74.It can be concluded from the passage that anyone can become a changemaker if he________.‎ A.considers Drayton's concept B.gets permission from Ashoka C.tries to improve social conditions D.is a young, happy and healthy adult ‎75.The author's attitude towards Ashoka's programme can be described as “________”.‎ A.changing B.forgiving C.cautious D.positive ‎【文章大意】 本文为一篇说明文。文章介绍了Bill Drayton创办的名叫“阿育王”的扶贫济困机构。‎ ‎72. A 主旨大意题。本文主要介绍的是Ashoka这个扶贫济困机构,该机构旨在让每一个公民都成为changemaker。第一段两次出现changemaker,第二段首句为主题句。故选A。‎ ‎73. D 代词指代题。 根据第三段“At first, they were refused, but once they were able to persuade them…”中有三个人称代词,前两个为they,均指代Masqsood and Iftekhar。故画线的them指代上文的the poor people in the city。故选D。‎ ‎74. C 推理判断题。根据第二段中“In fact, it is many young people's ambition to set up programmes or businesses that improve social conditions.”可推知尽力改善社会环境的任何人都可以成为changemaker。‎ ‎75. D 观点态度题。文章对Ashoka's programme—changemakers自始至终充满着赞赏之词,显然作者对此是持积极态度的。故选D。‎ ‎ [2019·北京卷] ‎ D Store scent(香味)‎ What is the first thing you notice when you walk into a shop? The products displayed(展示) at the entrance? Or the soft background music?‎ But have you ever noticed the smell? Unless it is bad, the answer is likely to be no.But while a shop's scent may not be outstanding compared with sights and sounds, it is certainly there. And it is proving to be an increasingly powerful tool in encouraging people to purchase.‎ A brand store has become famous for its distinctive scent which floats through the fairly dark hall and out to the entrance, via scent machines. A smell may be attractive but it may not just be used for freshening air. One sports goods company once reported that when it first introduced scent into its stores, customers' intention to purchase increased by 80 per cent.‎ When it comes to the best shopping streets in Paris, scent is just as important to a brand's success as the quality of its window displays and goods on sale. That is mainly because shopping is a very different experience to what it used to be.‎ Some years ago, the focus for bread name shopping was on a few people with sales assistants' disproving attitude and don'ttouchwhatyoucan'tafford displays. Now the rise of electronic commerce (ecommerce)has opened up famous brands to a wider audience. But while eshops can use sights and sounds, only bricksandmortar stores(实体店) can offer a full experience from the minute customers step through the door to the moment they leave. Another brand store seeks to be much more than a shop, but rather a destination.And scent is just one way to achieve this.‎ Now, a famous store uses complex manmade smell to make sure that the soft scent of baby powder floats through the kid department, and coconut(椰子) scent in the swimsuit section. A department store has even opened a new lab, inviting customers on a journey into the store's windows to smell ‎ books,pots and drawers, in search of their perfect scent.‎ ‎67. According to the passage, what is an increasingly powerful tool in the success of some brand stores?‎ A. Friendly assistants.‎ B. Unique scents.‎ C. Soft background music.‎ D. Attractive window display.‎ ‎68. Eshops are mentioned in the passage to ________.‎ A. show the advantages of bricksandmortar stores B. urge shop assistants to change their attitude C. push stores to use sights and sounds D. introduce the rise of ecommerce ‎69. The underlined word “destination” in Paragraph 5 means ________.‎ A. a platform that exhibits goods B. a spot where travellers like to stay C. a place where customers love to go D. a target that a store expects to meet ‎70. The main purpose of the passage is to ________.‎ A. compare and evaluate B. examine and assess C. argue and discuss D. inform and explain ‎【文章大意】 本文主要介绍了香味对于传统实体店的影响。各大品牌店都加强了味觉对顾客消费欲求的刺激,并根据商品的不同而调节香味。‎ ‎67. B 细节理解题。根据第二段中“And it is proving to be an increasing powerful tool in encouraging people to purchase.”可知,此句中的it指的是前面的scent,故选B。‎ ‎68. A 推理判断题。根据第五段中“But while eshops…only bricksandmortar stores (实体店) can offer a full experience…”可知作者的倾向非常明显,认为实体店能提供完整的购物体验,即强调传统实体店的优势。故选A。‎ ‎ [2019·福建卷] ‎ E As has been all too apparent in recent days at Balcombe, few issues cause greater concern than energy policy. Many village communities feel their countryside is being ruined by the powerproducing machines of wind farms; yet they never take “direct action”, even though the planning laws put them at a severe disadvantage. And the generous subsidies(财政补贴), which encourage the expansion of wind power, are not favourable to the village communities and set landowners in conflict with other residents(居民).‎ Those who disagree with the rapid expansion of wind farms state that the damage they cause is out of proportion(比例) to the benefits they bring, because their energy output cannot match that of the carbonbased power stations they are supposed to replace. Supporters insist that wind must be part of a mix of renewables, nuclear and carbon, and that the country is committed to meeting EU (European Union) targets for noncarbon energy generation. ‎ Against this background, the fact that there is an argument within the government over whether to publish an official report on wind farms' impact on the countryside becomes even more extraordinary. The two parties in the coalition(联合) government are in disagreement over what it should say.‎ We have some advice for the two parties: publish the report, and let the country be the judge. Even if it contains evidence that wind farms are harmful, it will hardly be a pleasant surprise to people who do not like them. Equally, supporters must argue their case by acknowledging the concerns and explaining why they are either misplaced or worthy of much attention.‎ The suggestion that further negotiations are to take place to produce an “acceptable” report suggests that the politics of coalition government are doing the country harm in a certain way. Given the sensitivities involved, all the information should be available so that people can reach their own conclusions, rather than being left with the suspicion(猜疑) that facts are being replaced by political beliefs.‎ ‎72.We can learn from the first paragraph that________.‎ A.energy policy catches much attention of the public B.the residents are in favour of the expansion of wind farms C.many village communities are satisfied with the subsidies D.the planning laws offer great benefits to the residents ‎73.Supporters think that the expansion of wind power ________.‎ A.is more rapid than that of carbonbased power B.guarantees an increase in energy output C.is expected to be much better than that of nuclear power D.agrees with EU targets for noncarbon energy generation ‎74.It can be inferred from the passage that________.‎ A.an official report will settle the energy problem B.the two parties are divided over the issue of wind farms C.the two parties have agreed on a further negotiation D.political beliefs concerning energy issue go against facts ‎75.Which of the following reflects the author's opinion?‎ A.Increase political impact on energy policy.‎ B.Release a statement of supporters on wind farms.‎ C.Let the nation judge the facts about wind power.‎ D.Leave the two parties to reach their own conclusions.‎ ‎【文章大意】 本文是一篇社会生活类文章。文章介绍了人们特别关心能源政策,尤其是风力发电站引起了多方的关注和不同观点的探讨。‎ ‎72.A 细节理解题。根据文章第一段中的“…few issues cause greater concern than energy policy.”可知能源政策引起了公众更多的注意,因此A项正确。‎ ‎73.D 细节理解题。第二段主要列举了支持者和反对者的观点。支持者认为风能一定是可再生能源,也是核能和碳能混合体的一部分,国家一直致力于满足欧盟的非碳能源生产的目标。因此D项正确。‎ ‎74.B 推理判断题。根据第三段最后一句可知B项正确。 ‎ ‎75.C 推理判断题。根据第四段,尤其是其中的“…and let the country be the judge.”可知C项正确。‎ ‎ [2019·湖北卷] ‎ E For most city people, the elevator is an unremarkable machine that inspires none of the enthusiasm or interest that Americans afford trains, jets, and even bicycles. Dr Christopher Wilk is a member of a small group of elevator experts who consider this a misunderstanding. Without the elevator, they point out, there could be no downtown skyscrapers or tall buildings, and city ‎ life as we know it would be impossible. In that sense, they argue, the elevator's role in American history has been no less significant than that of cars. In fact, according to Wilk, the car and the elevator have been locked in a “secret war” for over a century, with cars making it possible for people to spread horizontally(水平地), and elevators pushing them towards life in close groups of towering vertical(垂直的) columns.‎ If we tend to ignore the significance of elevators, it might be because riding in them tends to be such a brief, boring, and even awkward experience—one that can involve unexpectedly meeting people with whom we have nothing in common, and an unpleasant awareness of the fact that we're hanging from a cable in a long passage.‎ In a new book, Lifted, German journalist and cultural studies professor Andreas Bernard directed all his attention to this experience, studying the origins of elevator and its relationship to humankind and finding that riding in an elevator has never been a totally comfortable experience. “After 150 years, we are still not used to it,” Bernard said. “We still have not exactly learnt to cope with the mixture of closeness and displeasure.” That mixture, according to Bernard, sets the elevator ride apart from just about every other situation we find ourselves in as we go about our lives.‎ Today, as the world's urban population explodes, and cities become more crowded, taller, and more crowded, America's total number of elevators—900,000 at last count, according to Elevator World magazine's “2019 Vertical Transportation Industry”—are a force that's becoming more important than ever. And for the people who really, really love them, it seems like high time that we looked seriously at just what kind of force they are. ‎ ‎67. What does the underlined word “this” in Paragraph 1 refer to?‎ A. The general view of elevators.‎ B. The particular interests of experts.‎ C. The desire for a remarkable machine.‎ D. The enthusiasm for transport vehicles.‎ ‎68. The author's purpose in mentioning cars is ________.‎ A. to contrast their functions with elevators'‎ B. to emphasize the importance of elevators C. to reveal their secret war against elevators D. to explain people's preference for elevators ‎69. According to Prof Bernard, what has made the elevator ride different from other life experiences?‎ A. Vertical direction.‎ B. Lack of excitement.‎ C. Little physical space.‎ D. Uncomfortable conditions.‎ ‎70. The author urges readers to consider ________.‎ A. the exact number of elevator lovers B. the serious future situation of elevators C. the role of elevators in city development D. the relationship between cars and elevators ‎【要点概述】 本文是一篇议论文。文章告诉我们,电梯在美国人的生活中是非常重要的,它使人们向垂直方向发展。随着城市规模的扩大,电梯的使用越来越多,人们要更加关注电梯的作用。‎ ‎68. B 推理判断题。作者在第一段中提到汽车,提到了它的作用“…with cars making it possible for people to spread horizontally(水平地)…”,然后提到电梯的作用“…elevators pushing them towards life in close groups of towering vertical(垂直的) columns.”,其目的是用汽车的作用说明电梯的作用及它的重要性。 所以B项正确。‎ ‎69. C 推理判断题。根据第三段中的“…that riding in an elevator has never been a totally comfortable experience.”以及“‘We still have not exactly learnt to cope with the mixture of closeness and displeasure.’”可知人们乘坐电梯的时候感觉是拥挤的、不愉快的,而这主要是因为电梯里的空间太小,给人压抑感。所以C项正确。‎ ‎70. C 推理判断题。根据文章最后一段第一句“Today, as the world's urban population explodes, and cities become more crowded, taller, and more crowded, America's total number of elevators—900,000 at last count, according to Elevator World magazine's ‘2019 Vertical Transportation Industry’—are a force that's becoming more important than ever.”可知,随着城市规模的扩大,电梯的使用越来越多,它起的作用比以往更重要了,所以人们要更加关注电梯的作用。所以C项正确。 ‎ ‎[2019·湖南卷] ‎ C The behaviour of a building's users may be at least as important as its design when it comes to energy use, according to new research from the UK Energy Research Centre (UKERC). The UK promises to reduce its carbon emissions(排放) by 80 per cent by 2050, part of which will be achieved by all new homes being zerocarbon by 2019. But this report shows that sustainable building design on its own—though extremely important—is not enough to achieve such reductions: the behaviour of the people using the building has to change too.‎ The study suggests that the ways that people use and live in their homes have been largely ignored by existing efforts to improve energy efficiency(效率),which instead focus on architectural and technological developments.‎ ‎“Technology is going to assist but it is not going to do everything,” explains Katy Janda, a UKERC senior researcher,“consumption patterns of building users can defeat the most careful design.”In other words,old habits die hard, even in the bestdesigned ecohome.‎ Another part of the problem is information. Households and billpayers don't have the knowledge they need to change their energyuse habits. Without specific information,it's hard to estimate the costs and benefits of making different choices. Feedback(反馈) facilities, like smart meters and energy monitors,could help bridge this information gap by helping people see how changing their behaviour directly affects their energy use; some studies have shown that households can achieve up to 15 per cent energy savings using smart meters.‎ Social science research has added a further dimension(方面),suggesting that individuals' behaviour in the home can be personal and cannot be predicted—whether people throw open their windows rather than turn down the thermostat(恒温器), for example.‎ Janda argues that education is the key. She calls for a focused programme to teach people about buildings and their own behaviour in them.‎ ‎66. As to energy use, the new research from UKERC stresses the importance of ________.‎ A. zerocarbon homes B. the behaviour of building users C. sustainable building design D. the reduction of carbon emissions ‎67. The underlined word “which” in Paragraph 2 refers to“________”.‎ A. the ways B. their homes ‎ C. developments D. existing efforts ‎68. What are Katy Janda's words mainly about?‎ A. The importance of changing building users' habits.‎ B. The necessity of making a careful building design.‎ C. The variety of consumption patterns of building users.‎ D. The role of technology in improving energy efficiency.‎ ‎69. The information gap in energy use ________.‎ A. can be bridged by feedback facilities B. affects the study on energy monitors C. brings about problems for smart meters D. will be caused by building users' old habits ‎70. What does the dimension added by social science research suggest?‎ A. The social science research is to be furthered.‎ B. The education programme is under discussion.‎ C. The behaviour of building users is unpredictable.‎ D. The behaviour preference of building users is similar.‎ ‎【文章大意】 英国能源研究中心新的研究表明加强对人们的教育对于减少二氧化碳的排放是非常重要的。‎ ‎66. B 细节理解题。文章第一句就提到了在能源使用方面,房子使用者的行为至少和设计一样重要,因此英国能源研究中心新的研究强调的是房子使用者的行为。‎ ‎67. D 词义猜测题。根据上下文可知这里which指代的就是existing efforts。‎ ‎68. A 推理判断题。文章第三段Katy的话强调的是房子使用者的消耗习惯所起的重要作用,因此A项正确。‎ ‎69. A 细节理解题。文章第四段的“Feedback(反馈) facilities…could help bridge this information gap by helping people see how changing their behaviour directly affects their energy use…”说明设施的反馈可以弥补信息差的局限性,因此A是正确答案。‎ ‎70. C 推理判断题。文章倒数第二段提到房子使用者的行为是不可预测的。‎ ‎ [2019·江苏卷] ‎ B However wealthy we may be, we can never find enough hours in the day to do everything we want. Economics deals with this problem through the concept of opportunity cost, which simply refers to whether someone's time or money could be better spent on something else.‎ Every hour of our time has a value. For every hour we work at one job we could quite easily be doing another, or be sleeping or watching a film. Each of these options has a different opportunity cost—namely, what they cost us in missed opportunities.‎ Say you intend to watch a football match but the tickets are expensive and it will take you a couple of hours to get to and from the stadium. Why not, you might reason, watch the game from home and use the leftover money and time to have dinner with friends?This—the alternative use of your cash and time—is the opportunity cost.‎ For economists, every decision is made by knowledge of what one must forgo—in terms of money and enjoyment—in order to take it up. By knowing precisely what you are receiving and what you are missing out on, you ought to be able to make betterinformed, more reasonable decisions. Consider that most famous economic rule of all:there's no such thing as a free lunch. Even if someone offers to take you out to lunch for free, the time you will spend in the restaurant still costs you something in terms of forgone opportunities.‎ Some people find the idea of opportunity cost extremely discouraging: imagine spending your entire life calculating whether your time would be better spent elsewhere doing something more profitable or enjoyable. Yet, in a sense it's human nature to do precisely that—we assess the advantages and disadvantages of decisions all the time.‎ In the business world, a popular phrase is “value for money”.People want their cash to go as far as possible. However, another is fast obtaining an advantage:“value for time”. The biggest restriction on our resources is the number of hours we can devote to something, so we look to maximize the return we get on our investment of time. By reading this passage you are giving over a bit of your time which could be spent doing other activities, such as sleeping and eating. In return, however, this passage will help you to think like an economist, closely considering the opportunity cost of each of your decisions.‎ ‎58.According to the passage, the concept of “opportunity cost” is applied to ________.‎ A.making more money B.taking more opportunities C.reducing missed opportunities D.weighing the choice of opportunities ‎59.The “leftover…time” in Paragraph 3 probably refers to the time ________.‎ A.spared for watching the match at home B.taken to have dinner with friends C.spent on the way to and from the match D.saved from not going to watch the match ‎60.What are forgone opportunities?‎ A.Opportunities you forget in decisionmaking.‎ B.Opportunities you give up for better ones.‎ C.Opportunities you miss accidentally.‎ D.Opportunities you make up for.‎ ‎【文章大意】 本文为一篇说明文。在我们的生活中,无论是对金钱还是时间而言,我们都需要一直寻找更好的机会。本文从经济学的角度诠释了这些概念。‎ ‎58.D 细节理解题。根据第一段第二句“Economics deals with this problem through the concept of opportunity cost, which simply refers to whether someone's time or money could be better spent on something else.”可知机会成本,简单指某人的时间或者金钱是否可以被更好地用在其他方面,即权衡机会的选择,故D项正确。‎ ‎ [2019·江西卷] ‎ C ‎ Close your eyes for a minute and imagine what life would be like if you had a hundred dollars less. Also imagine what it would be like spending the rest of your life with your eyes closed. Imagine having to read this page, not with your eyes but with your fingertips.‎ With existing medical knowledge and skills, two thirds of the world's 42 million blind should not have to suffer. Unfortunately, rich countries possess most of this knowledge, while developing countries do not. ‎ ORBIS is an international nonprofit organization which operates the world's only flying teaching eye hospital. ORBIS intends to help fight blindness worldwide. Inside a DC8 aircraft, ‎ there is a fullyequipped teaching hospital with television studio and classroom. Doctors are taught the latest techniques of bringing sight back to people there. Project ORBIS also aims at promoting peaceful cooperation(合作) among countries. ‎ ORBIS tries to help developing countries by providing training during threeweek medical programmes. ORBIS has taught sightsaving techniques to over 35,000 doctors and nurses, who continue to cure tens of thousands of blind people every year. ORBIS has conducted 17 plane programmes in China so far. For the seven to ten million blind in China, ORBIS is planning to do more for them. At the moment an ORBIS team is working on a longterm plan to develop a training centre and to provide eye care services to Shanxi Province.ORBIS needs your help to continue their work and free people from blindness.‎ For just US$38, you can help one person see; for $380 you can bring sight to 10 people; $1,300 helps teach a doctor new skills; and for $13,000 you can provide a training programme for a group of doctors who can make thousands of blind people see again.Your money can open their eyes to the world.Please help ORBIS improve the quality of life for so many people less fortunate than ourselves.‎ ‎65. The first paragraph is intended to ________.‎ A. introduce a new way of reading B. advise the public to lead a simple life C. direct the public's attention to the blind D. encourage the public to use imagination ‎66. What do we learn about existing medical knowledge and skills in the world?‎ A. They are adequate.‎ B. They have not been updated.‎ C. They are not equally distributed.‎ D. They have benefited most of the blind.‎ ‎67. ORBIS aims to help the blind by ________.‎ A. teaching medical students B. training doctors and nurses C. running flying hospitals globally D. setting up nonprofit organizations ‎68. What does the author try to do in the last paragraph?‎ A. Appeal for donations.‎ B. Make an advertisement.‎ C. Promote training programmes.‎ D. Show sympathy for the blind.‎ ‎69. What can be the best title for the passage?‎ A. ORBIS in China ‎ B. Fighting blindness C. ORBIS flying hospital ‎ D. Sightsaving techniques ‎【文章大意】 本文为一篇说明文。文章主要介绍了一个名叫ORBIS的专门培训为盲人治眼的医生和护士的组织。‎ ‎65. C 推理判断题。根据第一段最后一句“Imagine having to read this page, not with your eyes but with your fingertips.”可知,第一段的主要目的是吸引公众对盲人的关注。 ‎ ‎66.C 细节理解题。根据第二段第二句“Unfortunately, rich countries possess most of this knowledge, while developing countries do not.”可知,现存的医疗知识和技能在世界的分布是不均的。‎ ‎ 67.B 细节理解题。根据第三段倒数第二句“Doctors are taught the latest techniques of bringing sight back to people there.”以及第四段第二句“ORBIS has taught sightsaving techniques to over 35,000 doctors and nurses…”可知,ORBIS的目的是通过培训医生和护士来帮助盲人。 ‎ ‎68. A 推理判断题。根据最后一段的内容,尤其是倒数第二句“Your money can open their eyes to the world.”可知,作者的目的是呼吁捐款。 ‎ ‎69. C 主旨大意题。综观全文可知,本文主要介绍了ORBIS飞行医院项目。故选C项。‎ ‎ [2019·新课标全国卷Ⅰ] ‎ C A typical lion tamer(驯兽师) in people's mind is an entertainer holding a whip(鞭) and a chair. The whip gets all of the attention, but it's mostly for show. In reality, it's the chair that does the important work. When a lion tamer holds a chair in front of the lion's face, the lion tries to focus on all four legs of the chair at the same time. With its focus divided, the lion becomes confused and is unsure about what to do next. When faced with so many options, the lion chooses to freeze and wait instead of attacking the man holding the chair.‎ How often do you find yourself in the same position as the lion? How often do you have something you want to achieve (e.g. lose weight, start a business, travel more)—only to end up confused by all of the options in front of you and never make progress?‎ This upsets me to no end because while all the experts are busy debating about which option is best, the people who want to improve their lives are left confused by all of the conflicting information. The end result is that we feel like we can't focus or that we're focused on the wrong things, and so we take less action, make less progress, and stay the same when we could be improving.‎ It doesn't have to be that way. Anytime you find the world waving a chair in your face, remember this: all you need to do is focus on one thing. You just need to get started. Starting before you feel ready is one of the habits of successful people. If you have somewhere you want to go, something you want to accomplish, someone you want to become…take immediate action. If you're clear about where you want to go, the rest of the world will either help you get there or get out of the way.‎ ‎28. Why does the lion tamer use a chair? ‎ A. To trick the lion. ‎ B. To show off his skills.‎ C. To get ready for a fight.‎ D. To entertain the audience.‎ ‎29. In what sense are people similar to a lion facing a chair?‎ A. They feel puzzled over choices. ‎ B. They hold on to the wrong things.‎ C. They find it hard to make changes. ‎ D. They have to do something for show.‎ ‎30. What is the author's attitude towards the experts mentioned in Paragraph 3?‎ A. Tolerant. B. Doubtful.‎ C. Respectful. D. Supportive.‎ ‎31. When the world is “waving a chair in your face”, you're advised to ________.‎ A. wait for a better chance B. break your old habits C. make a quick decision D. ask for clear guidance ‎【文章大意】 本文是一篇议论文。本文作者利用驯兽师对待狮子的例子说明了人们在面临多种选择感到困惑时该如何做。当我们面对多种选择时,我们常常不知该选哪一种,因此进展较少,或保持原样。实际上,我们应该专注于一件事,然后下决心做下去。‎ ‎29. A 推理判断题。根据首段中的“With its focus divided, the lion becomes confused and is unsure about what to do next.”可知,当关注的东西很分散时,狮子开始变得困惑,人们与狮子相似,在面临很多选择时也会感到困惑,不知道该怎么做。故选A。‎ ‎30. B 推理判断题。根据第三段中的“This upsets me to no end because while all the experts are busy debating about which option is best…”可知,专家们正忙于讨论哪一种选择是最好的,这令作者苦恼,由此推断作者对专家的态度是怀疑的。故选B。‎ ‎31. C 推理判断题。根据最后一段中的“Anytime you find the world waving a chair in your face, remember this: all you need to do is focus on one thing. You just need to get started.”可知,当世界在你面前挥舞着一把椅子时,你要集中精力做一件事。故选C。 ‎ ‎ [2019·新课标全国卷Ⅱ] ‎ B Since the first Earth Day in 1970, Americans have gotten a lot “greener” towards the environment.“We didn't know at that time that there even was an environment, let alone that there was a problem with it,” says Bruce Anderson, president of Earth Day USA.‎ But what began as nothing important in public affairs has grown into a social movement.Business people, political leaders, university professors, and especially millions of grassroots Americans are taking part in the movement. “The understanding has increased many, many times,” says Gaylord Nelson, the former governor from Wisconsin, who thought up the first Earth Day. ‎ According to US government reports, emissions(排放) from cars and trucks have dropped from 10.3 million tons a year to 5.5 million tons.The number of cities producing CO beyond the standard has been reduced from 40 to 9.Although serious problems still remain and need to be dealt with, the world is a safer and healthier place.A kind of “green thinking” has become part of practices.‎ Great improvement has been achieved.In 1988 there were only 600 recycling programmes; today in 2019 there are about 6,600.Advanced lights, motors, and building designs have helped save a lot of energy and therefore prevented pollution.‎ Twentyfive years ago, there were hardly any education programmes for environment.Today, it's ‎ hard to find a public school, university, or law school that does not have such a kind of programme.“Until we do that, nothing else will change!” says Bruce Anderson.‎ ‎25. According to Anderson, before 1970, Americans had little idea about ________.‎ A. the social movement ‎ B. recycling techniques C. environmental problems D. the importance of Earth Day ‎ ‎26. Where does the support for environmental protection mainly come from?‎ A. The grassroots level. ‎ B. The business circle. ‎ C. Government officials.‎ D. University professors. ‎ ‎27. What have Americans achieved in environmental protection?‎ A. They have cut car emissions to the lowest. ‎ B. They have settled their environmental problems. ‎ C. They have lowered their CO levels in forty cities.‎ D. They have reduced pollution through effective measures.‎ ‎28. What is especially important for environmental protection according to the last paragraph?‎ A. Education. B. Planning. ‎ C. Green living. D. CO reduction. ‎ ‎【文章大意】 本文作者通过把美国现在的环境问题和以前的环境问题进行比较,尤其是几项数据的对比,说明了人们的环境保护意识有了很大的提高。环境教育在这其中起了很大的作用。 ‎ ‎25. C 细节理解题。根据第一段的“‘We didn't know at that time that there even was an environment, let alone that there was a problem with it,’says Bruce Anderson…”可知当时美国人对于环境问题没有足够的认识。‎ ‎26. A 信息理解题。第二段第二句“Business people, political leaders, university professors, and especially millions of grassroots Americans are taking part in the movement.”强调了这项运动的主要支持者是那些美国的基层民众,因此答案为A。‎ ‎27.‎ ‎ D 细节理解题。根据文章的第三段第一句可知汽车和卡车的排放量从10,300,000吨降到了5,500,000吨,故A项错误;根据第三段第二句可知一氧化碳排放超标的城市的数量从40个降到了9个,故C项错误;根据第三段第三句可知环境问题依然存在,有待解决,故B项错误;本段列举的数据证明了美国人采取了有效措施,减少了环境污染,因此答案为D。‎ ‎28. A 推理判断题。根据文章最后一段中的“Twentyfive years ago, there were hardly any education programmes for environment.Today, it's hard to find a public school, university, or law school that does not have such a kind of programme.”可知,现在几乎每个学校都有环境保护方面的教育内容,这说明教育在环境保护方面起了重要的作用。故A正确。‎ ‎ [2019·四川卷] ‎ D With around 100 students scheduled to be in that 9 am Monday morning lecture, it is no surprise that almost 20 people actually make it to the class and only 10 of them are still awake after the first 15 minutes; it is not even a surprise that most of them are still in their pyjamas(睡衣). Obviously, students are terrible at adjusting their sleep cycles to their daily schedule.‎ All human beings possess a body clock. Along with other alerting(警报) systems, this governs the sleep/wake cycle and is therefore one of the main processes which govern sleep behaviour. Typically, the preferred sleep/wake cycle is delayed in adolescents, which leads to many students not feeling sleepy until much later in the evenings. This typical sleep pattern is usually referred to as the “night owl” schedule of sleep.‎ This is opposed to the “early bird” schedule, and is a kind of disorder where the individual tends to stay up much past midnight. Such a person has great difficulty in waking up in the mornings. Research suggests that night owls feel most alert and function best in the evenings and at night. Research findings have shown that about 20 per cent of people can be classified as “night owls” and only 10 per cent can be classified as “early birds”—the other 70 per cent are in the middle. Although this is clearly not true for all students, for the ones who are true night owls this gives them an excellent excuse for missing their lectures which unfortunately fall before midday.‎ ‎43. What does the author stress in Paragraph 1?‎ A. Many students are absent from class.‎ B. Students are very tired on Monday mornings.‎ C. Students do not adjust their sleep patterns well.‎ D. Students are not well prepared for class on Mondays.‎ ‎44. Which of the following is TRUE according to Paragraph 2?‎ A. Most students prefer to get up late in the morning.‎ B. Students don't sleep well because of alerting systems.‎ C. One's body clock governs the sleep/wake cycle independently.‎ D. Adolescents' delayed sleep/wake cycle isn't the preferred pattern.‎ ‎45. Which of the following is closest in meaning to the underlined word “classified”? ‎ A. Criticized. B. Grouped. ‎ C. Organized. D. Named.‎ ‎46. What does the text mainly talk about?‎ A. Functions of the body clock. ‎ B. The “night owl” phenomenon.‎ C. Human beings' sleep behaviour.‎ D. The school schedule of “early birds”.‎ ‎【文章大意】 本文是一篇说明文。文章开头以学生星期一早上上课时很多学生难以将他们的睡眠周期调整到正常模式为例,解释了“night owl”这一睡眠周期现象。‎ ‎43. C 推理判断题。文章开头通过举例说明了星期一早上上课时很多学生难以调整他们的睡眠模式。又根据首段的主旨句“Obviously, students are terrible at adjusting their sleep cycles to their daily schedule.”可知答案为C。‎ ‎ [2019·天津卷] ‎ A A guide to the university Food The TWU Cafeteria is open 7 am to 8 pm. It serves snacks(小吃), drinks, ice cream bars and meals. You can pay with cash or your ID cards. You can add meal money to your ID cards at the Front Desk. Even if you do not buy your food in the cafeteria, you can use the tables to eat your lunch, to have meetings and to study.‎ If you are on campus in the evening or late at night, you can buy snacks, fast food, and drinks in the Lower Café located in the bottom level of the Douglas Centre. This area is often used for entertainment such as concerts, games or TV watching.‎ Relaxation The Globe, located in the bottom level of McMillan Hall, is available for relaxing, studying,‎ ‎ cooking, and eating. Monthly activities are held here for all international students. Hours are 10 am to 10 pm, closed on Sundays.‎ Health Located on the top floor of Douglas Hall, the Wellness Centre is committed to physical, emotional and social health. A doctor and nurse is available if you have health questions or need immediate medical help or personal advice. The cost of this is included in your medical insurance. Hours are Monday to Friday, 9 am to noon and 1:00 to 4:30 pm.‎ Academic support All students have access to the Writing Centre on the upper floor of Douglas Hall. Here, qualified volunteers will work with you on written work, grammar, vocabulary, and other academic skills. You can sign up for an appointment on the signup sheet outside the door two 30minute appointments per week maximum. This service is free.‎ Transportation The TWU Express is a shuttle(班车) service. The shuttle transports students between campus and the shopping centre, leaving from the Mattson Centre. Operation hours are between 8 am and 3 pm, Saturdays only. Round trip fare is $1.‎ ‎36. What can you do in the TWU Cafeteria?‎ A. Do homework and watch TV.‎ B. Buy drinks and enjoy concerts. ‎ C. Have meals and meet with friends.‎ D. Add money to your ID and play chess.‎ ‎37. Where and when can you cook your own food?‎ A. The Globe, Friday.‎ B. The Lower Café, Sunday.‎ C. The TWU Cafeteria, Friday.‎ D. The McMillan Hall, Sunday.‎ ‎38. The Guide tells us that the Wellness Centre ________.‎ A. is open six days a week B. offers services free of charge C. trains students in medical care D. gives advice on mental health ‎39. How can you seek help from the Writing Centre?‎ A. By applying online.‎ B. By calling the centre.‎ C. By filling in a signup form.‎ D. By going to the centre directly.‎ ‎40. What is the function of TWU Express?‎ A. To carry students to the lecture halls.‎ B. To provide students with campus tours C. To take students to the Mattson Centre.‎ D. To transport students to and from the stores.‎ ‎【文章大意】 本文是一篇说明文。 主要介绍了在大学里的生活指南。涉及食物、休闲、健康、学业帮助以及交通等方面的内容。‎ ‎36. C 细节理解题。本题问的是在In the TWU Cafeteria学生可以做的事,因此将注意力放在Food栏目下的第一段。根据这一段中的最后一句“…you can use the tables to eat your lunch, to have meetings and to study.”可知C项正确。‎ ‎37. A 细节理解题。根据题干中的cook your own food,可知它与第二个栏目Relaxation中的cooking对应,这是发生在the Globe,且时间是周一到周六的早上10点到晚上10点,由此可知A正确。‎ ‎38. D 细节理解题。根据题干中的the Wellness Centre,可知此题涉及的是Health部分。根据第一句中的“…is committed to physical, emotional and social health.”可知D项正确。‎ ‎39. C 细节理解题。根据题干中的the Writing Centre,可知此题涉及的是Academic Support部分。根据倒数第二句中的“You can sign up for an appointment on the signup sheet outside the door…”可知,申请时只要填写报名表格就行了,可知C项正确。‎ ‎40. D 细节理解题。根据题干中的TWU Expression可知此题涉及的是Transportation部分。根据第二句“The shuttle transports students between campus and the shopping centre…”可知TWU Express的功能是在商店和商店之间运送学生,可知D项正确。 ‎ ‎ [2019·天津卷] ‎ C ‎“Dad,” I say one day,“let's take a trip. Why don't you fly out and meet me?” My father had just retired after 27 years as a manager for IBM. His job filled his day, his thoughts, his ‎ life. While he woke up and took a warm shower, I screamed under a freezing waterfall in Peru. While he tied a tie and put on the same Swiss watch, I rowed a boat across Lake of the Ozarks.‎ My father sees me drifting aimlessly, nothing to show for my 33 years but a passport full of funny stamps. He wants me to settle down, but now I want him to find an adventure.‎ He agrees to travel with me through the national parks. We meet four weeks later in Rapid City.‎ ‎“What's our first stop?” asks my father.‎ ‎“What time is it?”‎ ‎“Still don't have a watch?”‎ Less than an hour away is Mount Rushmore. As he stares up at the four Presidents carved in granite(花岗岩), his mouth and eyes open slowly, like those of a little boy.‎ ‎“Unbelievable,” he says. “How was this done?”‎ A film in the information centre shows sculptor Gutzon Borglum devoted 14 years to the sculpture and then left the final touches to his son.‎ We stare up and I ask myself, “Would I ever devote my life to anything?”‎ No directions,no goals. I always used to hear those words in my father's voice. Now I hear them in my own.‎ The next day we're at Yellowstone National Park, where we have a picnic.‎ ‎“Did you ever travel with your dad?” I ask.‎ ‎“Only once,” he says. “I never spoke much with my father. We loved each other—but never said it. Whatever he could give me, he gave.”‎ That_last_sentence—it's_probably_the_same_thing_I'd_say_about_my_father._And_what_I'd_want_my_child_to_say_about_me.‎ In Glacier National Park, my father says, “I've never seen water so blue.” I have, in several places of the world, I can keep travelling, I realize—and maybe a regular job won't be as dull as I feared.‎ Weeks after our trip, I call my father. ‎ ‎“The photos from the trip are wonderful,” he says.“We've got to take another trip like that sometime.”‎ I tell him I've decided to settle down, and I'm wearing a watch.‎ ‎46. We can learn from Paragraphs 2 and 3 that the father ________.‎ A. followed the fashion B. got bored with his job C. was unhappy with the author's lifestyle D. liked the author's collection of stamps ‎47. What does the author realize at Mount Rushmore?‎ A. His father is interested in sculpture.‎ B. His father is as innocent as a little boy.‎ C. He should learn sculpture in the future.‎ D. He should pursue a specific aim in life.‎ ‎48. From the underlined paragraph, we can see that the author ________.‎ A. wants his children to learn from their grandfather B. comes to understand what parental love means C. learns how to communicate with his father D. hopes to give whatever he can to his father ‎49. What could be inferred about the author and his father from the end of the story?‎ A. The call solves their disagreements.‎ B. The Swiss watch has drawn them closer.‎ C. They decide to learn photography together.‎ D. They begin to change their attitudes to life.‎ ‎50. What could be the best title for the passage?‎ A. Love nature, love life ‎ B. A son lost in adventure C. A journey with dad D. The art of travel ‎【文章大意】 本文是一篇夹叙夹议文。“我”喜欢旅游,而我的父亲喜欢工作。在父亲退休后,“我”邀请父亲去旅游,在旅游的过程中父子二人对生活的态度发生了变化。父亲想去旅游,而“我”却想静下来工作。‎ ‎46.‎ ‎ C 推理判断题。第二段中讲到作者过着流浪的生活,而父亲忙于自己的工作;第三段父亲要作者安顿下来,但作者想让他进行一次冒险。由此可推知父子二人的生活轨道是完全不一样的,父亲不喜欢作者的生活方式。故选C项。‎ ‎49. D 推理判断题。根据文章最后的结尾,父亲说:“我们再去旅游吧。”而我告诉他“我已经决定安定下来。”可知旅游过后,父子二人对生活的态度发生了很大的变化。故D项正确。‎ ‎50. C 主旨大意题。总览全文,可知作者叙述了一次与父亲的旅游经历,而在这次旅游中,父子二人的生活发生了根本的变化。C项中的关键词journey与dad简练完整地将文章的主题归纳了出来。‎ ‎ [2019·天津卷] ‎ D People aren't walking any more—if they can figure out a way to avoid it.‎ I felt superior about this matter until the other day I took my car to mail a small parcel. The journey is a matter of 281 steps. But I used the car. And I wasn't in any hurry, either. I had merely become one more victim of a national sickness: motorosis.‎ It is an illness to which I had thought myself immune(免疫的), for I was bred in the tradition of going to places on my own two legs. At that time, we regarded 25 miles as good day's walk and the ability to cover such a distance in ten hours as a sign of strength and skill. It did not occur to us that walking was a hardship. And the effect was lasting. When I was 45 years old I raced—and beat—a teenage football player the 168 steps up the Stature of Liberty.‎ Such enterprises today are regarded by many middleaged persons as bad for the heart. But a wellknown British physician, Sir Adolphe Abrhams, pointed out recently that hearts and bodies need proper exercise. A person who avoids exercise is more likely to have illnesses than one who exercises regularly. And walking is an ideal form of exercise—the most familiar and natural of all.‎ It was Henry Thoreau who showed mankind the richness of going on foot. The man walking can learn the trees, flowers, insects, birds and animals, the significance of seasons, the very feel of himself as a living creature in a living world. He cannot learn in a car.‎ The car is a convenient means of transport, but we have made it our way of life. Many people don't dare to approach Nature any more; to them the world they were born to enjoy is all threat. To them security is a_steel_river thundering on a concrete road. And much of their thinking takes place while waiting for the traffic light to turn green.‎ I say that the green of forests is the mind's best light. And none but the man on foot can evaluate what is basic and everlasting.‎ ‎51. What is the national sickness?‎ A. Walking too much.‎ B. Travelling too much.‎ C. Driving cars too much.‎ D. Climbing stairs too much.‎ ‎52. What was life like when the author was young?‎ A. People usually went around on foot.‎ B. People often walked 25 miles a day.‎ C. People used to climb the Statue of Liberty.‎ D. People considered a tenhour walk as a hardship.‎ ‎53. The author mentions Henry Thoreau to prove that________.‎ A. middleaged people like getting back to nature B. walking in nature helps enrich one's mind C. people need regular exercise to keep fit D. going on foot prevents heart disease ‎54. What is compared to “a steel river” in Paragraph 6?‎ A. A queue of cars.‎ B. A ray of traffic light.‎ C. A flash of lightning.‎ D. A stream of people.‎ ‎55. What is the author's intention of writing this passage?‎ A. To tell people to reflect more on life.‎ B. To recommend people to give up driving.‎ C. To advise people to do outdoor activities.‎ D. To encourage people to return to walking.‎ ‎【文章大意】 本文是一篇议论文。作者认为全国的人们都患了一种病motorosis,即人们过多地使用汽车,而不走路。作者把现在的自己和年轻时的自己走路做了对比,同时又用专家的观点来证明自己的观点以此鼓励人们要回归走路。‎ ‎51. C 细节理解题。根据第二段中的最后一句中的a national sickness: motorosis以及前文的 “The journey is a matter of 281 steps. But I used the ‎ car.”可知作者提到的全国通病就是人们开车太多了。可知C项正确。‎ ‎52. A 细节理解题。根据第三段中的内容,作者年轻时靠自己的双腿去某个地方,一天走25英里路,从没感觉走路是一种痛苦。由此可知,作者年轻时人们通常靠自己的双脚到处走动。可知A项正确。‎ ‎53. B 推理判断题。根据第五段中的第一句“It was Henry Thoreau who showed mankind the richness of going on foot.”可知作者提到Henry Thoreau是为了证明用脚走路可以丰富人类的大脑。可知B项正确。‎ ‎54. A 词义猜测题。根据画线词组后面的“…while waiting for the traffic light to turn green.”可知是车在等交通灯变绿,所以这里是指一排车。可知A项正确。‎ ‎55. D 推理判断题。总览全文,可知“我”用自己的亲身经历告诉读者现在很多人都依赖车,不自己走路了,而走路使人身体健康,丰富人的内心。作者同时引用名人名言,进一步证明自己的观点,其目的都是在鼓励人们重新回归走路。所以D项正确。‎ ‎ [2019·浙江卷] ‎ B Here is some mustknow information from a handbook on how people behave in doing business in some countries.‎ In Brazil Brazilians are warm and friendly.They often stand close when talking and it is common for them to touch the person on the shoulder.People often greet each other (particularly women) with light cheek kisses.Schedules tend to be flexible,with business meetings sometimes starting later than planned.But to be safe, be on time.Meals can stretch for hours—there's no such thing as rushing a meal in Brazil.Lunches also can start in the mid to late afternoon.Brazilians are social,preferring facetoface communication over emails or phone calls.‎ In Singapore Singaporeans shake hands when they meet and often also greet each other with a small,polite bow.Business cards should be offered and received with two hands.Arriving late is considered disrespectful.So be on time. Efficiency(效率) is the goal, so meetings and dealings often are fastpaced.Singaporeans are direct in their discussions,even when the subject is about money.Rank is important and authority is respected.This determines how people interact in meetings.For example,people avoid disagreeing outright with someone of a higher rank.‎ In the United Arab Emirates In the UAE,status is important, so the most senior or oldest should be greeted first with their titles.The handshake seems to be longer than elsewhere. So do not pull away from the handshake. Women should cover themselves when it comes to dress.Men also tend to be covered from neck to elbows(肘部) and down to the knees.People do not avoid entertaining in their homes,but they also hold business meals at restaurants.Touching or passing food or eating with your left hand is to be avoided.When meetings are onetoone,if your host offers you coffee,you should refuse.It might seem odd,but it is a cultural tradition.Coffee should only be accepted if it is already set out or presented.‎ In Switzerland The Swiss tend to be formal and address each other by last name.They also are respectful of private lives.You should be careful not to ask about personal topics.Punctuality (守时) in vital,something that comes from a deep respect for others' time.Arrive at any meeting or event a few minutes early to be safe.They also have clear structure in their companies.Higherups make the final decisions,even if others might disagree.Neat,clean dress is expected.The Swiss follow formal table manners.They also keep their hands visible at the table and their elbows off the table.It is polite to finish the food on your plate.‎ ‎46.The passage is mainly about ________.‎ A.communication types B.the workplace atmosphere C.customs and social manners D.living conditions and standards ‎47.Why do Singaporeans avoid arguing with the boss?‎ A.They put efficiency in the first place.‎ B.They dislike facetoface communication.‎ C.They want to finish meetings as quickly as possible.‎ D.They are supposed to obey the person of a higher rank.‎ ‎48.In the UAE,when should you refuse the coffee if it is offered?‎ A.When greeting seniors.‎ B.When meeting the host alone.‎ C.When attending a presentation.‎ D.When dining with business partners.‎ ‎49.In which country do people care about where to put their hands at the dinner table?‎ A.In Brazil. ‎ B.In Singapore.‎ C.In the United Arab Emirates.‎ D.In Switzerland.‎ ‎【文章大意】 本文是一则应用文,主要向读者介绍了不同国家的生意场上有不同的社交礼仪。‎ ‎48. B 细节理解题。 从第四段的“When meetings are onetoone, if your host offers you coffee, you should refuse…”可知,当单独与主人一起时,他提出喝杯咖啡时,最好的礼仪是委婉地拒绝。故选B项。 ‎ ‎49. D 细节理解题。 从最后一段中的“The Swiss follow formal table manners. They also keep their hands visible at the table…”知,在瑞士人们吃饭的时候让双手放到桌上,这是瑞士传统的餐桌礼仪。故选D项。‎ ‎[2019·重庆卷] ‎ D One moment it was quiet and calm in the forest, the next, the air was charged with tension. The elephant had heard the distant alarm calls of animals and her mood suddenly changed. I urged the elephant deeper into the forest. We sounded like a forest fire—crackling, snapping, trailblazing. But through all the noise came a sharp warning cry. The elephant stopped and we heard it again—the telltale call of a spotted deer.‎ I looked quickly around the shadows of the forest. Rays of sunlight shone through tree branches,beneath which the patchwork(交错) of green plants and shadowwithinshadows would make tiger stripes(条纹) look more attractive. Apart from an occasional noise from the elephant's stomach, the forest was silent.‎ Gradually, the tension slipped from our bodies. The elephant seized a nearby branch and put it into her mouth. I reached forward and gently moved my hand over the elephant's neck; there was a soft part, free of wrinkles and hairs, behind her ear.‎ This was my fourth time to_sense_the_aura of the forest in Corbett, although I saw no tigers in the end. Located at the foot of the Himalayan mountains, Corbett is home to about 135 Bengal tigers, but the forest seemed to be guarding their whereabouts(出没处), a silent reminder of their ‎ secrecy and rarity. Still, I was happy enough touching the elephant behind the ear. If I had so desperately wanted to see a tiger, I could have gone to a zoo. After all, spotting tigers merely confirms their beauty;tracking them can make you aware of something more.‎ ‎48. Which of the following was a clear signal of alarm?‎ A. The elephant stopped.‎ B. A spotted deer called.‎ C. The elephant seized a branch.‎ D. The forest was silent for a while.‎ ‎49. The author begins his account of the tour in the forest mainly by________‎ A. describing various sounds B. comparing different animals C. listing different activities D. introducing various plants ‎50. What does the underlined part “to sense the aura” most probably mean?‎ A. To see the diversity.‎ B. To enjoy the scenery.‎ C. To feel the atmosphere.‎ D. To experience the freedom.‎ ‎51. How does the author feel after several visits to Corbett?‎ A. Seeing a Bengal tiger is quite thrilling.‎ B. It is very timeconsuming to travel in Corbett.‎ C. It is really worthwhile to study the animals in Corbett.‎ D. The process of finding Bengal tigers is most appealing.‎ ‎【文章大意】 该文介绍了作者和他的一头大象在森林中差点遇见孟加拉虎的经历。‎ ‎48.B 细节理解题。从第一段最后两句可知答案。‎ ‎49.A 写作方法题。第一段中有“We sounded like a forest firecrackling, snapping, trailblazing.”的详尽叙述,forest firecracking(森林大火燃烧的声音),snapping(鞭子噼啪作响), trailblazing(开路先锋的吆喝)等都是对声音的描述。‎ ‎50.C 词义猜测题。全文都在描写作者和大象在Corbett森林里的各种感觉和感受。作者对森林环境的描写是其对森林气氛的感知的结果。故选C。‎ ‎51.D 推理判断题。从短文最后一段中的“After all, spotting tigers merely confirms their beauty; tracking them can make you aware of something more.”(总之,看见老虎仅仅可以确定其美丽,而寻找老虎的踪迹却能让你感受更多)可知,寻找孟加拉虎很有吸引力。‎ ‎ [2019·重庆卷] ‎ E It's generally believed that people act the way they do because of their personalities and attitudes.They recycle their garbage because they care about the environment. They pay $5 for a caramel brulée latte because they like expensive coffee drinks.‎ It's undeniable that behaviour comes from our inner dispositions(性情), but in many instances we also draw inferences about who we are, as suggested by the social psychologist Daryl Bem, by observing our own behaviour. We can be strangers to ourselves. If we knew our own minds, why would we need to guess what our preferences are from our behaviour? If our minds were an open book, we would know exactly how much we care about the environment or like lattes. Actually, we often need to look to our behaviour to figure out who we are.‎ Moreover, we don't just use our behaviour to learn about our particular types of character—we infer characters that weren't there before. Our behaviour is often shaped by little pressures around us,which we fail to recognize. Maybe_we_recycle_because_our_wives_and_neighbours_would_disapprove_if_we_didn't. Maybe we buy lattes in order to impress the people around us. We should not mistakenly believe that we always behave as a result of some inner disposition.‎ Whatever pressures there can be or inferences one can make, people become what they do,though it may not be in compliance(符合) with their true desires. Therefore, we should all bear in mind Kurt Vonnegut's advice: “We are what we pretend to be, so we must be careful about what we pretend to be. ”‎ ‎52. According to the passage, personalities and attitudes are commonly believed to ________.‎ A. determine one's behaviour B. reflect one's taste C. influence one's surroundings D. result from one's habits ‎53. Which of the following would Daryl Bem most probably agree with?‎ A. The return of a wallet can indicate one's honesty.‎ B. A kind person will offer his seat to the old.‎ C. One recycles plastics to protect the environment.‎ D. One buys latte out of true love of coffee.‎ ‎54. What can be inferred from the underlined sentence in the passage?‎ A. We fail to realize our inner dispositions.‎ B. We can be influenced by outside pressures.‎ C. Our behaviour is the result of our true desires.‎ D. Our characters can shape our social relationships. ‎ ‎55. What does the author mainly discuss in the passage?‎ A. Personalities and attitudes.‎ B. Preferences and habits.‎ C. Behaviour and personalities.‎ D. Attitudes and preferences.‎ ‎【文章大意】 本文论述了人之所以成为某种性格的人是因为他或她在不断受到环境和他人的影响下而成为他或她假装的那个人,这是一个悖论,但事实果真如此吗?读完该文,读者会有更多的思考空间。‎ ‎52.A 细节理解题。由文章第一句及该题题干中的commonly believed可知由性格决定行为的见解颇为常见。而后文则是对这种观点的反相论证。‎ ‎55.C 主旨大意题。全文通过对行为和性格的深刻分析得出结论。故选C。 ‎
查看更多

相关文章

您可能关注的文档